Download as pdf or txt
Download as pdf or txt
You are on page 1of 218

Department of Electrical Power and Machines Engineering

Zagazig University
Department of Electrical Power and Machines Engineering

Zagazig University
Contents
Chapter one: Circuit Variables
1) Potential and Potential Difference……………..……………7
2) The Electric Current:-...............................................................8
3) The Electric Circuit:- ................................................................9
Energy Sources:-....................................................................9
Power and energy:-..............................................................10
The load:-.............................................................................12
4- Kirchhoff's laws:- ....................................................................19
a)The first law, or current law. ............................................19
b) The second law or voltage law........................................20
Problems .......................................................................................29
CHAPTER 2:The Natural and Forced Responses of Simple
Circuit
I - First order systems:-
Circuit Containing Inductance And Resistance:- ................33
The natural response:- .........................................................35
Responses considered as solution of the differential
equation:- .............................................................................37
1-The forced responses....................................................37
2-The natural response.....................................................37
The step function response:-................................................42
Circuits Containing Capacitance And Resistance:-.............44
Problems .......................................................................................52
II - Second order systems:-..........................................................56
Circuit Containing Resistance, Inductance And
Capacitance:- .......................................................................56
Natural response of a parallel RLC circuit..........................56
Determining the constants A1 and A2for the following cases;57
a)Over – damped case.......................................................57
b)Under – damped case. ...................................................58
c)Critically damped case. .................................................60
Natural response of a series RLC circuit with a step function
source...................................................................................64
The Sinusoidal Forcing Function; .......................................72
The Laplace Transformation......................................................74
Some Laplace transforms evaluated;...................................75
Expansion by partial fraction...............................................76
1- Simple and non- repeated................................................76
2-Conjugate complex...........................................................76
3- Repeated roots. ................................................................77
The differentiation theorem. ................................................77
The integral theorem............................................................78
Circuit elements in the S domain.........................................78
Problems.........................................................................................88
CHAPTER 3: The sinusoidal Steady State………..…………92
The sinusoidal response:- ............................................................92
Power due to a sinusoidal varying current:- .............................93
The Phasor diagram:-..................................................................95
Addition and subtraction of phasors....................................96
Phasor relationships between voltage and current in the circuit
elements:- ......................................................................................98
a) Resistance case. ...............................................................98
b) Inductance case. ..............................................................98
c) Capacitance case..............................................................99
Kirchhoff's Laws in The Phasor Domain ................................101
a ) Kirchhoff's voltage law in the phasor domain:- ...........101
b ) Kirchhoff's current law in the phasor domain:- ...........101
Sinusoidal Steady State Power Calculation.............................102
Complex Power ..........................................................................103
Series Parallel and Delta To Star Simplifications...................105
a) Series combination:-......................................................105
b) Parallel combination:- ...................................................106
c) Delta to Star Transformation and vice versa.................108
I) Delta to star transformation:-.........................................108
II) Star to delta transformation:-........................................109
Problems .....................................................................................113
Parallel and Series Resonance ..................................................116
a) Parallel Resonance:- ......................................................116
Band width and quality factor:-...................................117
Describing the circuit behavior in terms of ωo ,
β and Q...........................................................................119
Effect of frequency variation on the phase shift:- .........122
b) Series Resonance:- ........................................................123
Describing the circuit behavior in terms of ωo ,
β and Q...........................................................................125
More in parallel resonance:-..........................................127
Power Factor Improvement ......................................................128
Z and Y Locus ............................................................................132
Case 1 Series circuit with variable X constant R. .............132
Case 2 Series circuit with variable R constant X. .............133
Case 3 Parallel circuit with variable B constant G............133
Problems .....................................................................................138
CHAPTER 4: Circuit analysis……………………………..…..141
Mesh current: -...........................................................................141
Node Voltage:- ............................................................................149
Problems .....................................................................................155
Mutual inductance .....................................................................159
Self inductance:- ................................................................159
Mutual inductance:-...........................................................159
Coupling coefficient K :-...................................................161
Analysis of coupling circuits:-...........................................161
Natural current:- ................................................................162
Dot rule for coupled coils:-................................................163
The dot rule:- .....................................................................163
Problems .....................................................................................171
Circuit theorems.........................................................................173
1- Superposition theorem:- ................................................173
2- Reciprocity theorem:-....................................................174
3- Maximum power transfer theorem:- .............................176
Case 1:The load is a variable resistor RL:- ........................176
Case 2:The load is a variable impedance ZL:-...................177
Case 3:The load impedance is a variable resistance RL and
fixed reactance XL:- ...........................................................178
4- Thevenin’s and Norton’s theorems:-.............................180
4-1) Thevenin’s theorem:- .................................................180
4-2) Norton’s theorem:-.....................................................182
a) By Thevenin’s theorem. ................................................185
b) By Norton’s theorem.....................................................186
Problems .....................................................................................190
CHAPTER 5: Poly phase Systems…………………………………194
Principles of star and mesh connections:- ...............................194
Advantage of polyphase systems:-............................................197
Two and three phase's systems:- ..............................................200
a) Two phase star connected system:- ...............................200
b) Three phase star connected system:-.............................200
c) Three phase mesh connected system:- ..........................202
Three phase power system with unbalanced load. .................205
The measurement of power and power factor in a three phase
circuits. ........................................................................................210
Connection of the wattmeter in a three phase system .......210
Case 1 : Four wires system ( Star point available)..........210
a) Unbalanced load. ......................................................210
b) Balanced load. ..........................................................211
Case 2 : Three wires system (Star point not available). ..211
Case 3 : Two wattmeter method......................................212
Power factor measurement for balanced loads.......................214
Problems .....................................................................................217
Chapter 1 Circuit Variables

Chapter 1

CIRCUIT VARIABLES

1) Potential and Potential Difference:-

Physical theory supposes that all matter consists of particles.


Each particle has a mass and electric charge. Two kinds of
charge are known (positive and negative). Two like charges exert
a mutual force of repulsion while unlike charges exert a force of
attraction. Therefore there is a force associated with the charges;
this force is referred to an electric field. The smallest known
charge is that of a single electron and, since this is much too
small to adopt as a unit for all but a few special purpose, a unit,
the COULOMB, equal to 6.24 × 1018 electron. Any system of
charges has a potential energy, without constrained the
individual charges will move and the energy will be released. So
every point in the field at a level of energy, the magnitude of
which is depending upon the charges and their relative positions.
When one of the charges is of unit magnitude, and positive, the
energy at the point defining its position is referred to as the
POTENTIAL at that point, and is measured in joules per
coulomb (J/C) or volts. The POTENTIAL DIFFERENCE
between two points in a field is therefore the difference in
energy, per unit charge, at the two points.

Example (1-1).
The potential energy of a charge of + 5 coulomb at point A in a
field is 15 J, and at point B is 45 J as shown in Fig. (1-1).
Determine in volt the potential difference VAB.
A B
+5 C • •
45 J
15 J
Fig. 1-1

7
Chapter 1 Circuit Variables

Assume the energy in joules is W, the charge in coulombs is q


dW Δ W
and the potential difference in volt is V then V = = ,
dq Δ q
∆W = 15- 45 = - 30 J and ∆ q = 5
− 30
∴V= = - 6 J/C = - 6 volt.
5
Negative sign means that the energy is required from an external
source to move the charge from A to B.

2) The Electric Current:-

dq
It depends on the rate of charge flow, i = C/S (Ampere).
dt
Where i the electric current in Ampere (A), q is the electric
charge in coulomb and t is the time in Sec.

Example (1-2).
The current at the terminals of the element shown in Fig. (1-2) is
as shown in the figure. Calculate the total charge in micro-
coulombs entering the element at its upper terminal.

i = 0 for t < 0 i
V
− 2000 t
i = 10 e for t ≥ 0

Fig. 1-2
dq
i= , ∴q = ∫ i dt
dt
∞ 10  − 2000 t ∞ 1
q = ∫ 10 e − 2000 t dt = e  = = 5×10 − 3 C = 5000 μ C
0 − 2000   0 200

Example (1-3).
Let the charge entering the upper terminal is,
1 t 1 −α t
q= −( + )e C.
α2 α α2

8
Chapter 1 Circuit Variables

Find the maximum value of the current entering the terminal if


α = 0.03679 S-1.

= − e− α t + α ( +
t 1 −α t
= t e −α t
dq 1
i= )e
dt α α α2
di
The maximum current may be at =0,
dt
∴ = e − α t − α t e − α t = e − α t (1− α t ) = 0
di
dt
1 1 1 −1 0.3679
∴t = = , ∴ im = e = =10 A
α 0.03679 0.03679 0.03679

3) The Electric Circuit:-

The electric circuit is concerned with the transmission of energy


from one point to another through the use of devices such as
generators, wires, batteries, motors and transformers. Thus the
electric circuit may contain generator, wire and load, Fig. (1-3)
shows an electric circuit.

Wire
Generator or
Load
Energy source

Fig. 1-3
Energy Sources:-

The energy sources may be one of many forms, electro chemical,


electro magnetic, thermo electric, photo electric……..etc. But for
the purpose of circuit analysis only two idealized forms are
recognized, these are Voltage sources and Current sources.
The voltage source maintains a constant terminal voltage
regardless to the current supplied to the load (i. e. voltage
constant with respect to variable load). While the current source,
maintains a constant current in the load, regardless to the

9
Chapter 1 Circuit Variables

terminal voltage at its terminal Fig. (1-4) shows a symbolic


diagram for both.

• i
i
Ri
n Rld I Ri E Rld
E
n
V

Voltage Current
Source Source
Fig. 1-4

Where Rin is the internal resistance for each source, Rld the load
resistance, V is the voltage source magnitude, I is the current
source magnitude and E is the terminal voltage from each
source.
For the voltage source Rin << Rld
∴E = V − i . R , ∴E ≅ V Regardless Rld value.
in
For the current source Rin >> Rld
R
i =I in ≅ I , ∴I ≅ constant regrdless R value .
R +R ld
in ld

Power and energy:-

Our purpose now is to relate the power and the energy to the
circuit variable of voltage and current.
dW
The power is the time rate of absorbing energy P =
dt
Where P is the power in watts, W is the energy in joule and t is
the time in second.
dW dW dq
P= = . = V .i
dt dq dt

10
Chapter 1 Circuit Variables

Example (1-4).
For the circuit shown in Fig.(1-5) assume V = 0 for t < 0 and
V = 50e − 2000 t V for t ≥ 0.
And i = 0 for t < 0 & i =10e − 2000 t A for t ≥ 0 . Calculate the
total energy in mill joules delivered to the circuit element.

Fig. 1-5

∞ ∞
W = ∫ P d t = ∫ V i d t = ∫ 50 ×10 e − 4000 t d t
0 0
−1 − 4000 t ∞ 500
= 500{ e } = =125 m J
4000 0 4000
N.P. If the potential difference V in the same direction as the
current i then the element may supply power, while if V and i are
anti phase then the element absorb power.

Example (1-5).
The voltage and current at the terminals of an automobile battery
during a charge cycle are shown in Fig. (1-6), calculate the total
charge and the total energy transferred to the battery.
i
V
15

10
12

9 6

14.8 t (Ks) 4 12 14.8 t (Ks)


Fig. 1-6

11
Chapter 1 Circuit Variables

4 5 12 4 14.8 6
q = ∫ (15 − t) dt + ∫ (10 − (t − 4)) dt + ∫ (6 − (t −12)) dt
0 4 4 8 12 2.8
2
5t 4 2
t 12 6t 2 14.8
= {15t − } + {12t − } + {31.71t − } = 122.304 coulomb
8 0 4 4 5.6 12

W = ∫ P dt = ∫ V i dt

3
V =9+ t
14.8
3 5
∴ P = V .i = ( 9 + t ) . (15 − t ) =135 − 8.21 t − 0.25 t 2
1 1 14.8 4
3 1
P = V .i = ( 9 + t ) . (12 − t ) =108 − 2.07 t − 0.1 t 2
2 2 14.8 2
3 6
P = V .i = ( 9 + t ) . ( 31.71− t ) = 285.39 −12.86 t − 0.434 t 2
3 3 14.8 2.8

4 12
W = ∫ (135 − 8.21 t − 0.25 t 2 ) dt + ∫ (108 − 2.07 t − 0.1 t 2 ) dt
0 4
14.8
+ ∫ ( 285.39 −12.86 t − 0.434 t 2 ) dt = 1499.838 Joules
12

The load:-

The load may be defined by the ratio between the terminal


voltage and the current. There are three distinct idealized types.

1- The resistance parameter.


It's the element that dissipates energy and not stores.

i(t)
V (t)
V(t)
R= , this equation is known as Ohm's law. Where R is the
i(t)
resistance in ohms.

12
Chapter 1 Circuit Variables

1
The reciprocal of a resistance is a conductance G, thus G =
R
∴ i (t) = V (t) . G and,
V (t) 2
P (t) = i (t) . V (t) ={i (t)}2 R = = G .{ V (t)}2
R
If the applied voltage and the current are constants; V (t) = V and
i (t) = I
Then, P = I 2. R = V 2. G

2- The inductance parameter.

It is the element which able to store magnetic energy, its


property is defined by the relationship.
L

i (t) V (t)
d i (t)
V ( t ) = L. Where L is the inductance in henrys if V in volts
d (t)
, i in amperes and t in seconds.
1 d i (t)
∴i (t) = ∫ V (t) . dt & P (t) = i (t) . V (t) = i (t) . L
L dt
d i (t) 1
W = ∫ P (t) dt = L . ∫ i (t) . = L . ( i (t)) 2
dt 2

Example (1-6).
For the circuit shown in Fig. (1-7) determine the following.
a) Sketch the current waveform.
b) At what instant the current is maximum.
c) Express the voltage across the terminals of the inductor as a
function of time.
d) Sketch the voltage waveform.

13
Chapter 1 Circuit Variables

i (t) = 0 for t < 0


−5t L = 100 mH
i (t) = 10t e for t ≥ 0

Fig. 1-7

a - Fig. (1-8) shows the current waveform.

i (t)
A
0.736

0 0.2 t (s)

Fig. 1-8

b -  i (t) =10t e − 5 t For maximum current then (


di
may equal
dt
to zero).
∴ =10 e − 5t − 50t e − 5t = 10 e − 5t (1− 5t ) = 0
di
dt
− 5× 0.2
t = 0.2, and im = 10 × 0.2 × e = 0.736 A

= 100 10- 3 { 10 e − 5t − 50t e − 5t }


di
c - V (t) = L .
dt
−5t
V (t) = (1- 5t) e

1
d - At t = 0 V (t) = 1 & at t = = 0.2 s, V (t) = 0
5
and at t = ∞ V (t) = 0.
Fig. (1-9) shows a sketch for the voltage waveform.

14
Chapter 1 Circuit Variables

V(t)
1
0.2
t (s)

Fig. 1-9

Example ( 1- 7).
For the problem that had been discussed in the previous
example, Define.
a) Plot P(t) and W (t) versus time.
b) In what time interval is the energy being stored in the
inductor, and in any interval it's extracted from it?
c) What is the maximum energy stored in the inductor?
0.2 ∞
d) Evaluate the integrals ∫ P dt & ∫ P dt .
0 0.2

−5t −5t
a - P (t) = V (t). i (t) = ( 1 – 5t ) e . 10 t e
−10 t
= (10 t - 50 t 2 ) e
P (t) = 0 at t = 0, t = 0.2 and t = .
For obtaining the points at which the power is maximum
dP
then; =0
dt
= (10 −100 t ) e −10 t −10 (10t − 50 t 2 ) e −10 t = 0
dP
dt
10 – 200 t + 500 t 2 = 0
∴ t = 0.06 and 0.34 sec.
P (0.06) = 0.428 and P (0.34) = - 0.079 watts.
Fig. (1-10) shows P (t) against t.

15
Chapter 1 Circuit Variables

P (t)

0.428
0.2
0.06 0.34 t (s)
- 0.079

Fig. 1-10

W (t) = ∫ P (t) dt = ∫ (10t − 50 t 2 ) e −10 t dt


e −10t
(10 t − 50 t 2 ) + ∫ e −10t (10 −100 t ) dt
1
W (t) =
−10 10
−10 t e −10t
=- e (t – 5 t 2 ) + −10 ∫ t e −10t dt
−10
−10 t −10t
e −10t dt ]
e 1
=- e (0.1 + t – 5 t 2 ) – 10 [ ∫ t+
−10 10
−10 t −10 t
=- e (0.1 + t – 5 t 2 – t – 0.1) = 5 t 2 . e
−2 −2
W (0.2) = 5 × 0.04 × e = 0.2 e = 0.02707 J = 27.07 m J

And W ( ∞ ) = ∫ P (t) dt = [ 5 t 2 e − 10 t ]∞ = − 27.07 m J
0.2
0.2
1 1
Or W (t) = L i 2 So W ( 0.2 ) = ×100 × ( 0.736 ) 2 = 27.08 mJ
2 2
1
And W (∞ ) = ×100 × 0 = 0
2
Fig. (1-11) shows a sketch for W (t) in mill joules against t in
seconds.

b- The energy stored in the inductor at the time interval 0<t≤0.2


and is extracted from it in the time interval 0.2<t≤∞.

16
Chapter 1 Circuit Variables

W (t) mJ
27.07

0.2 t (s)

Fig. 1-11

c- Wmax. = 27.07 mJ

0.2
d- W (0.2) = ∫ P (t) dt ={5 t 2 e −10 t }0.2 = 27.07 mJ
0
0
∞ ∞
 2 −10 t 
W( ∞ ) = ∫ P (t) dt = 5 t e  = − 27.07 m J
0.2   0.2

3 - The capacitance parameter.

A circuit that is able to store electrostatic energy is said to


process capacitance.
q (t) = C . V (t), where C is the capacitance in farads when V in
volt and q in coulomb.

i (t) C

V(t)
dq dV 1
i (t) = =C Or V (t) = ∫ i (t) dt
dt dt C
d V (t) t
P (t) = i (t). V (t) = C . V (t) & W (t) = ∫ P (t) dt
dt 0
t d V(t) 1
= C ∫ V (t) dt = C V 2
0 dt 2
The following table summarizes the relations between the
different types of the loads & the voltage and the current.

17
Chapter 1 Circuit Variables

R V (t) i (t) G = i (t) V (t)


V(t) 1
L = i ( t ) ∫ V (t) dt
d i (t) dt L
i (t) 1
C = V( t ) ∫ i (t) dt
d V (t) dt C

Example (1-8).
For the circuit shown in Fig. (1-12) find.
a) An expression for the capacitor current, power and energy.
b) Sketch the voltage, current, power and energy as function of
time.

V (t) = 0 t≤0
V (t) = 4t 0≤t≤1 V (t)
C = 0.5 µ f
− t +1
V (t) = 4 e 1≤t≤∞

Fig. 1-12

= 0.5× 10 − 6 × 4 = 2 μ A
d V (t)
a- i (t) = C. for 0 ≤ t ≤ 1
dt
= 0.5 × 10 − 6 × ( − 4 e − t +1 ) = − 2.0 e − t +1µ A for 1 ≤ t ≤ ∞
−6
P (t) = V (t) . i (t) = 4 t × 2 × 10 =8tµW for 0 < t ≤ 1
− t +1 − t +1 −6
=-4 e ×2 e × 10
−2t +2
=-8 e µW for 1 < t ≤ ∞
1 1
 W (t) = ∫ P (t) dt , Then W(1) = ∫ 8 t dt = 4 t 2  = 4 μ J
0  0
∞ e −2t +2 ∞
W ( ∞ ) = 4 + ∫ − 8 e − 2 t + 2dt = 4 − 8[ ] = 4 − 4 = 0 μJ
1 − 2 1

b- Fig. (1-13 - a, b, c and d) shows the voltage, current, power


and energy sketch's.

18
Chapter 1 Circuit Variables

i (t) µ A
V (t) volt
4
2

1 t
1 t -2
a b

W (t) µ J
P (t) µ W
4
8

1 t 1 t
-8
c d
Fig 1- 13

4- Kirchhoff's laws:-

a)The first law, or current law.

The algebraic sum of currents meeting at a node or a junction of


a network is zero.
Applying this law on the circuit which has been shown in Fig.
(1-14) then,
i (t) = i1 (t) + i2 (t) + i3 (t) + i4 (t)

i (t) i1 (t) i2 (t) i3 (t) i4 (t)


R1 R2 R3 R4

Fig. 1- 14

19
Chapter 1 Circuit Variables

b) The second law or voltage law.

In any closed mesh "circuit" the algebraic sum of all the potential
drops due to current flow is equal to the algebraic sum of all the
e.m.f in the mesh. One direction round the mesh is taken to be
positive.
Applying the second Kirchhoff's law on the circuit which had
been shown in Fig (1-15).
Then in mesh 1;
di (t )
V (t) + V (t) = V (t) So, i ( t ) . R + L 2 = V (t)
R L 1 2 dt 1
And in mesh 2;
V (t) + V (t) − V (t) = V (t)
R L C 2

i1(t)
R VR(t) V2(t)
V1(t) 1 i2(t) 2
i3(t)
L VL(t) C
VC(t)

Fig. 1-15
di (t) 1
Then, i (t). R + L 2 − ∫ i (t) dt = V (t)
2 dt C 3 2

Example (1-9).
For the circuit which has been shown in Fig. (1- 16) find.
a) The power supplied by each source.
b) The power dissipated in the resistors.

20
Chapter 1 Circuit Variables

i1(t) 2 Ω

i2
585 V {i1(t) – i2 (t)}
20 Ω
(t)
4Ω
40 Ω
{i2 (t) – i3 (t)} i3
585 V 5Ω
(t)
2Ω
{i1(t) – i2(t) + i3 (t)}

Fig. 1-16

From the circuit which had been shown in Fig. (1-16) it's
possibol todeduce that;
4{i2(t) – i3 (t)} + 20 i2 (t) + 2 i1 (t) = 585 .. 1
2 {i1 (t) - i2 (t) + i3 (t)} + 5 i3 (t) – 4 {i2 (t) – i3 (t)} = 585 .. 2
Also 5 i3 (t) + 20 i2 (t) – 40 {i1 (t) – i2 (t)} = 0 .. 3
From equations 1 & 2 and 3, then
i1 (t) = 52.5, i2 (t) = 30 and i3 (t) = 60 A

a- The power supplied from the sources = i1 (t) × 585 +


585 × ( i1 (t) – i2 (t) + i3 (t)) = 585 ( 52.5 + 82.5 ) = 78975 W

b- The power dissipated in the resistors = ∑ [ i ( t )]2 . R


∴The power dissipated = (52.5)2 × 2 + (30)2 × 20 + (22.5)2 × 40
+ (60)2 × 5 + (30)2 × 4 + (82.5)2 × 2 = 78975 W

Example (1- 10).


For the circuit shown in Fig. (1-17) calculate the terminals
voltage of the source and sketch the waveform for one complete
cycle.

21
Chapter 1 Circuit Variables

10 Ω i (t)
A
20
i (t) 0.5 H 0.8 1.5 1.6
0.2 0.7 1.0 t
-20

Fig. 1-17

There are six intervals of time;


Δ i (t)
While i (t) = io (t) ± ( t − t o ) , then i (t) at any interval may
Δt
be;
20
Interval 1 i1 (t) = t = 100 t for 0.0 ≤ t ≤ 0.2
0.2
Interval 2 i2 (t) = 20 for 0.2 ≤ t ≤ 0.7
20
Interval 3 i3 (t) = 20 − ( t − 0.7 ) = 160 - 200 t for 0.7 ≤ t ≤ 0.8
0.1
20
Interval 4 i4 (t) = 0.0 − ( t − 0.8 ) = 80 - 100 t for 0.8 ≤ t ≤ 1.0
0.2
Interval 5 i5 (t) = - 20 for 1.0 ≤ t ≤ 1.5
20
Interval 6 i6 (t) = - 20 + ( t −1.5 ) = - 320+200 t for 1.5 ≤ t ≤ 1.6
0.1
d i (t) d i (t)
 VR (t) = R × i (t) = 10 × i (t) & VL (t) = L = 0.5
dt dt
And Vs (t) =VR (t) + VL (t)
Then Fig. (1- 18 a, b, & c) shows VR(t) , VL(t) and Vs(t)
respectively.

22
Chapter 1 Circuit Variables

VR (t)
200
0.8 1.6
0.2 0.7 1.0 1.5 t
-200
-a-
VL(t)
100
50 0.8

-50 0.2 0.7 1.0 1.5 1.6 t


-100
-b-
Vs(t) = VR(t) + VL(t)

250
100 0.8 1.0
0.2 0.7 1.5 1.6 t
-100

-250
-c-

Fig. 1-18

Example (1-11).
For the circuit shown in Fig. (1-19) given i1(t) = at +b . Evaluate
the constants a and b and hence determine the current i1(t), the
voltage VAB (t) and the current i2(t).

23
Chapter 1 Circuit Variables

i1 (t) 2 Ω A

{i1 (t) – i2 (t)} 0.2 H


V(t) = 12t V 0.1 f i2 (t)
1Ω
B
Fig. 1-19

d i (t) 1
=12t − 2 (at + b) = i (t) + 0.2 2 = (at + b − i (t)) dt
0.1 ∫
V
AB 2 dt 2
Differentiating the previous equation with the time then;

12 – 2a = 10 ( at + b – i2 (t))
10 at +10b + 2a −12 di (t)
2 =a
∴i (t) = , Then
2 10 dt
10 at +10b + 2a − 12
∴t (12 − 2a) − 2b = + 0.2a
10
10t (12 – 2a) – 20b = 10 at + 10b + 2a – 12 + 2a
Comparing the t coefficients then, 120 – 20a = 10a, then the
120
parameter a = =4
30
Also comparing the scalar magnitudes
then, - 20b = 10 b + 4a – 12 , -30b = 16 – 12 = 4, then the
4
parameter b = − = − 0.1333
30
∴ i1 (t) = 4t – 0.1333 A
then VAB (t) = t ( 12 – 8 ) + 0.2666 = 4t + 0.2666 V
40 t −1.333 + 8 −12
& i2 (t)= = 4t -0.5333 A
10

Example (1-12).
For the circuit shown in Fig. (1-20) if the current source supply
the circuit with a current i(t) = 3 t2 + 4 t + 1. Then calculate.
a) The input power to the coil at t = 2 sec.

24
Chapter 1 Circuit Variables

b) The energy stored and dissipated up to 3 sec.


c) The energy released if the current fall to zero after 5 sec.
2Ω

i (t) 10 H

Fig. 1-20

d i (t)
a- PL (t) = VL (t). i (t) = L . × i (t)
dt
= 10 × (6 t + 4) × (3 t2 + 4 t + 1) = 180 t 3 + 360 t 2 + 220 t + 40
∴ PL (2) = 1440 + 1440 + 440 + 40 = 3360 W

3
b- The energy stored at 3 second WL (3) = W(0) + ∫ P (3) dt
0 L
1 1
W (0) = L ( i(0) ) 2 = × 10 × ( 1 )2 = 5 Joule
2 2
3
WL (3) = 5 + ∫ (180 t 3 + 360 t 2 + 220 t + 40) dt
0
180 t 4 360 t 3 220 t 2
WL (3) = 5 + { + + + 40 t }3
4 3 2 0
= 5 + 7995 = 8000 Joule
1
Or Wstored (3) = L ( i(3) ) 2
2
i (3) = 3 × (3)2 + 4 × 3 + 1 = 40 A
1
∴ Wstored (3) = × 10 × (40)2 = 8000 J
2
3 3
Wdissipated (3) = ∫ P (t) dt = ∫ 2 (3t 2 + 4t +1) 2 dt
R
0 0
3
= 2 ∫ (9 t 4 + 24 t 3 + 22 t 2 + 8 t +1) dt
0

25
Chapter 1 Circuit Variables

9 t 5 24 t 4 22 t 3 8 t 2
Wdissipated (3) = 2 { + + + + t }3 = 2320.8 J
5 4 3 2 0

c- The energy released after 5 Sec. if the current falls to zero


may be equal to the energy stored in 5 Sec.
180 t 4 360 t 3 220 t 2
Wstored (5) = W (0) + { + + + 40 t }5
4 3 2 0
= 5 + 46075 = 46080 J
1
Or = Wstored (5) = L ( i(5) ) 2
2
2
i (5) = 3 × (5) + 4 × 5 + 1 = 96
1
∴Wstored (5) = × 10 × (96)2 = 46080 J
2

Example (1-13).
The given series circuit shown in Fig. (1-21) passes a current i(t)
of waveform shown in Fig. (1-22). Find the voltage across each
element and sketch each voltage to same time scale. Also sketch
the charge on the capacitor q(t).
2Ω 2 mH 500 µ f

Fig. 1-21

i (A)

10
6
3 t ms
-10
Fig 1-22

Across the resistor: VR =R × i


The plot of VR is a duplicated of the current function plot, with a
peak value of 2 × (10) = 20 V.
Fig. (1-23) shows a sketch for VR(t).

26
Chapter 1 Circuit Variables

VR (Volts)
20

3 6 t ms
-20
Fig. 1-23

di
Across the inductor: VL=L
dt
(1) 0 < t < 1 ms i (t) = 10 × 103 t amperes
-3 × 3
VL = (2 × 10 ).(10 10 ) = 20 V
(2) 1 < t < 2 ms i = 10 A VL = (2 × 10-3)(0) = 0
Etc. Fig. (1-24) shows a sketch for the voltage across the
inductor terminals.
VL (Volts)
20
2 4
1 5 6 t ms
-20

Fig. 1-24

1
Across the capacitor: Vc = ∫ i dt
C
1 t (10 ×103 t ) dt = 10 ×106 t 2
(1) 0 < t < 1ms Vc = ∫
500 ×10 − 6 0
(2) 1 < t < 2ms
Vc = 10 +
1 t 3 −3
−6 ∫ − 3 (10) dt = 10 + 20 ×10 (t − 10 )
500 ×10 10
3
= -10 + 20 × 10 t etc.
Fig. (1-25,a) shows the voltage across the capacitor terminals
The plot of q is easily made using the relationship q = C Vc.
Note that when i(t) is positive, both q and Vc increase, i.e. both
the charge on the capacitor and the voltage across the capacitor

27
Chapter 1 Circuit Variables

increase; when i(t) is negative, both decrease . Fig. (1-25, b)


represents the charge on the capacitor.

VC (t)
q Coul.

40
20 x 10- 3

6 t ms 6 t ms

-a- -b-

Fig. 1-25

28
Chapter 1 Circuit Variables

Problems

1-a) what current is supplied by the source in the circuit shown


in Fig. (1-26) below?
b) Find the power dissipated in each resistor.
c) Show that the power delivered by the source is equal to the
power absorbed by all the circuit elements

5Ω

2Ω
8Ω 5Ω 3Ω
12Ω
50 V

Fig. 1- 26

2- A series RL circuit with R = 100 Ω and L = 0.5 H has a


current function as described below. Determine VR(t) and VL
(t) in each interval.
− 2000 t
a) 0 < t ≤ 10 × 10 - 3 s, i (t) = 5 { 1 – e } A.
− 2000 ( t −10 x 10 − 3)
b) 10 × 10 -3 < t s , i (t) = 5 e A.

− 500 t
3- The current in RC circuit is i (t) = 10 e A. There was
no initial charge on the capacitor. After the current transient
the capacitor has a charge of 0.02 coulombs. If the applied
− 500 t
voltage is V(t) = 100 { 1- e } volts, find C and VR(t) .

4- The parallel RL circuit has a square wave of voltage as shown


in Fig. (1-27). Determine the total current it (t).

29
Chapter 1 Circuit Variables

V(t) volts
it (t)
20
V (t) 4 Ω 10 mH
5 10 15 20 t (ms)
- 20

Fig. 1-27

5- The parallel RC circuit has an applied voltage of waveform


shown in Fig. (1-28) below. Determine the total current it (t).

it (t)
Vt (t) 5Ω 400 µ f

v(t)
v
50 o
l
t 2 4 6
-50 t ms
s

Fig. 1-28

6- The voltage at the terminals of the capacitor is known to be,


−1000 t
V (t) = 40 − ( 50 cos 500 t + 20 sin 500 t ) e V for t ≥ 0.
If C = 0.8 µ f find q (t), i (t), w ( ∞ ) and q ( ∞ ).

7- A series RL circuit with R = 5 Ω and L = 0.004 H contains a


current with a waveform as shown in Fig. (1-29). Sketch VR
(t) and VL (t).

30
Chapter 1 Circuit Variables

i (t)
A
5
2 4 6 8 t
-5
m
s
Fig. 1-29

8- A rectangular voltage pulse, of magnitude 50 V and duration


100 msec. is applied across a 2 H inductor. The initial current
is zero. Calculate the current at the end of 100 msec period. It
is desired to reduce the current to zero again in 5 msec.
Determine the magnitude and polarity of the required voltage
pulse.

9- A steady voltage of 100 V is applied to a 2 H inductor.


Determine the current flowing and the energy stored after 10
sec. given that the current at t = 0 was zero. Determine also
the voltage which must be applied in order to reduce the
current to zero in 2 sec. sketch the current and voltage
waveforms.

10- A steady current of 4 mA is fed into a 10 µf capacitor.


Determine the voltage to which the capacitor is charged after
5 sec. and the corresponding stored energy. The capacitor was
initially uncharged. Determine the current needed to
discharge the capacitor completely in 20 sec. sketch the
voltage and the current waveforms.

11- Determine the equivalent inductance of the three inductors


connected as shown in the diagram of Fig. (1-30).

31
Chapter 1 Circuit Variables

i2 (t) L2
i1 (t) L1
i3 (t)
V1 (t) L3
V2 (t)
V (t)

Fig. 1- 30

32
Chapter 2 The Natural and Forced Responses of Simple Circuit

CHAPTER 2

The Natural and Forced Responses of Simple Circuit

I - First order systems:-


Circuit Containing Inductance And Resistance:-
It had been discussed previously the behavior of ideal inductors
and capacitors. It had been noted that their ability for storing
energy. The currents and the voltages that arise when the energy
stored is released to a purely resistive network may be
determined in this chapter. For the circuit shown in Fig. (2-1).
i (t) R

V (t) L

Fig. 2-1

The relationship between voltage and current is expressed by;


d i (t)
R .i (t) + L. = V (t)
dt
This equation may be solved for the current if V (t) is known.
For natural response of the RL circuit (i.e. to release the energy
stored in the inductors) the supply voltages may be zero.
d i (t)
R .i (t) + L. =0
dt
d i (t) R
Then =− .d t
i (t) L
Integrating the previous equation, then
R R
{Lin i (t) }t = - t , So Lin i (t) − lin i (0) = − t
0 L L

i (t) −R t −R t
∴ = e L , So i (t) = i (0) e L = I es t
i (0)

33
Chapter 2 The Natural and Forced Responses of Simple Circuit

Where s is a real constant and I is a constant equal the current at


zero time (i.e. I = i (0)).
For the circuit shown in Fig. (2-2) let i (t) = I es t , then the
voltage across the circuit may be;

R i (t) L

V (t)
Fig 2-2

d i (t)
V (t) = R i (t) + L = R I es t + L S I es t
i (t)
= (R + L S) I es t = V es t
Where V = I (R + L S) = I. Z (s) and Z (s) = R + L S
Plotting Z(s) and V (t) for different values of S, hence Fig. (2-3)
shows these characteristics.
i(t) V(t)
Z
S - ve V(t) S + ve
(s V(t)
)R ∞
R i(t) i(t)
S= −
L
-∞ S t

Fig. 2-3

When Z (s) = ∞ i. e. S = ± ∞ this case is called pole of the


impedance.
Thus the function Z (s) = R + LS has one zero and one pole. The
admittance is defined as ;
1 R
Y(s) = has a pole at S = - and a zero at S = ± ∞ as
R + LS L
shown in Fig. (2-4).
1
The steady current at S = 0 gives Y (s) = =G
R

34
Chapter 2 The Natural and Forced Responses of Simple Circuit

Y (s)
1
G=
R R
S=-
L
0 S
s

Fig 2-4

The natural response:-


 I = V . Y (s) and V = I . Z (s)
For Y (s) = ∞ or Z (s) = 0 gives the condition for which the
circuit responds with no applied voltage.
R
For Y (s) = ∞ or Z (s) = 0, so S = -
L
− t
R − t
R
∴ i (t) = I e L = i (0) e L This expression is the natural
response.

Example (2-1).
For the circuit shown in Fig. (2-5) the switch is open at t = 0,
then
a) Find i1 (t), i2 (t) and i3(t) for t ≥ 0
b) Calculate the initial energy stored in the parallel circuit.
c) Determine how much energy is trapped in the inductors at
t → ∞.
d) The total energy delivered to the resistive network.
4Ω

5H 20H 40Ω 15Ω 10Ω


i1= 8 i2= 4 t=0 i3(t)

Fig. 2-5

35
Chapter 2 The Natural and Forced Responses of Simple Circuit

a- The equivalent circuit of the previous circuit is shown in


Fig. (2-6).
12 A

4H V (t) 8Ω

Fig. 2-6

−R t −8 t
i (t) = I e L = 12 e 4 = 12 e − 2 t A t ≥ 0
∴ V (t) = i (t) . R = 12 e − 2 t * 8 = 96 e − 2 t V t ≥ 0
1t 1t
∴ i1 (t) = ∫ V ( t ) d t - i1 (0) = ∫ 96 e − 2 t dt − 8
L0 50
= 1.6 – 9.6 e − 2 t A t ≥ 0
1 t
i2 (t) = ∫ 96 e − 2 t dt − 4 = - 1.6 - 2.4 e − 2 t A t ≥ 0
20 0
× 96 e − 2 t = 5.76 e − 2 t A t ≥ 0
V (t) 15 3
i3 (t) = × =
10 25 50

1 2
b- The initial energy stored in the parallel circuit = Li
2
1
= ( 5 × 82 + 20 × 42 ) = 320 J
2
c- At t → ∞ i1 = 1.6 and i2 = - 1.6, hence the energy trapped in
the inductors may be ;
1 1
W ( ∞ ) = L i 2 = ( 5 × 1.62 + 20 × (- 1.6)2 ) = 32 J
2 2

d- The total energy delivered to the resistive network may be;


∞ ∞
WR= ∫ P (t) . dt = ∫ V (t) .i (t) . dt
0 0
∞ ∞
= ∫ 96 e − 2 t .12 e − 2 t . dt = ∫ 1152 e − 4 t . dt =288 J
0 0

36
Chapter 2 The Natural and Forced Responses of Simple Circuit

Responses considered as solution of the differential


equation:-
d i(t)
R .i (t) + L . = V (t)
dt
The solution of this differential equation may be in two parts;

1- The forced responses.


For exponential waveforms, the impeding effect of resistance
and inductance taken together may be combined into a single
factor known as the impedance factor and denoted by Z (s). As
the current I flow as a result of the application of the voltage V
to the circuit, V and I are known as the Forcing response.

2- The natural response.


Since I × Z (s) = V, The value of S which makes Z (s) = 0 gives
the condition for which the circuit responds with no applied
voltage. Then the pole of Y (s) or the zero of Z (s) yields this
condition. This is called the Natural response.

Example ( 2-2).
−12 t
If the voltage V (t) = 100 e V is applied at t = 0 to a circuit
consisting of 10 Ω and 0.25 H calculate.
a) The forced current response.
b) The natural response.

St −12 t
a-  V (t) = V e = 100 e
∴ S = - 12
Z (s) = R + L S = 10 + 0.25 × – 12 = 7 Ω
V 100
I= = = 14.286 A
Z (s) 7
−12 t
∴ if (t) = 14.286 e A

b- For natural response the Z (s) = 0


Z (s) = R + L S = 0

37
Chapter 2 The Natural and Forced Responses of Simple Circuit

R 10
∴S = - =− = − 40
L 0.25
St − 40 t
in (t) = I e = I e A.
I Can be determined from the initial data.

Example ( 2-3).
For the circuit shown in Fig. (2-7) find an expression for the
current i (t).
i (t) 50 Ω

V (t) = 150 sin 500 t 0. 2 H

Fig. 2- 7

J500 t
 V (t) = 150 sin 500 t = J 150 e
Z (s) = R + L S = 50 + 0.2 × J 500 = 50 + J 100 = 111.8 ∠ 63.4o
150
If = ο = 1.34 ∠ - 63.4o A
111.8 ∠ 63.4
J(500 t - 63.4)
∴if (t) = J1.34 e = 1.34 sin ( 500 t – 63.4 ) A
Z (s) = R + L S = 0 for natural response
R 50
∴ S = - =− = − 250
L 0.2
-250 t
∴in (t) = I e
-250 t
i (t) = if (t) + in (t) = 1.34 sin ( 500 t – 63.4 ) + I e A
 At t = 0 , i (t) = 0
∴0 = 1.34 sin ( - 63.4 ) + I
I = - 1.34 sin ( - 63.4 ) = 1.34 × 0.894 = 1.2
-250 t
∴i (t) = 1.34 sin ( 500 t – 63.4 ) + 1.2 e A

Example (2-4).
For the circuit shown in Fig. (2-8), if I = 5A for t ≤ 0 , determine
the total current for S has a different values, its equals 20, -10,
-25 or -30.

38
Chapter 2 The Natural and Forced Responses of Simple Circuit

I 5Ω

St
V (t) = 100 e 0. 2 H

Fig. 2-8

For S = 20
Z (20) = 5 + 0.2 × 20 = 9 Ω
100
If = = 11.1 A
9
20 t
if (t) = 11.1 e A
For natural response;
R 5
S = - =− = − 25
L 0.2
-25 t
∴in (t) = I e A
20 t -25 t
i (t) = 11.1 e +I e A
At t = 0 i (0) = 5
∴5 = 11.1 + I
I = - 6.1
20 t -25 t
∴i (t) = 11.1 e - 6.1 e A

For S = - 10
Z (- 10) = 5 - 0.2 × 10 = 3 Ω
100
If = = 33.3 A
3
-10 t
if (t) = 33.3 e A
-25 t
in (t) = I e
-10 t -25 t
i (t) = 33.3 e + Ie A
For t = 0 i (0) = 5
∴5 = 33.3 + I
I = - 28.3
-10 t -25 t
i (t) = 33.3 e - 28.3 e A

39
Chapter 2 The Natural and Forced Responses of Simple Circuit

For S = -25
Z (- 25) = 5 - 0.2 × 25 = 0
100 S t
∴if (t) = e
Z (s)
-25 t
in (t) = I e
100 S t -25 t
i (t) = e +I e
Z (s)
100
At t = 0 5 = +I
Z (s)
100
∴I = 5 -
Z (s)
100 S t 100 -25 t
i (t) = e + (5 - )e
Z (s) Z (s)
-25 t 100 St -25 t
=5 e + (e -e )
Z (s)
0
At S = - 25 i (t) =
0
According to L'Hospital's theorem (Rule).
f (t) 0 f (t) f \ (t)
If lim = then, lim = lim
t → x  (t) 0 t → x  (t) t → x  \ (t)

1 S t − 25 t t e St -25 t
∴ lim . (e − e ) = lim =5t e
S → − 25 5 + 0.2 S S → − 25 0.2
-25 t -25 t -25 t
i (t) = 5 e + 500 t e = ( 5 + 500 t ) e A

For S = - 30
Z (-30) = 5 - 0.2 × 30 = -1 Ω
100
If = = - 100 A
-1
-30 t
if (t) = - 100 e A
For natural response;
-25 t
∴in (t) = I e A
-30 t -25 t
i (t) = - 100 e + Ie A

40
Chapter 2 The Natural and Forced Responses of Simple Circuit

At t = 0 i (0) = 5
∴5 = - 100 + I , So I = 105
-25 t -25 t -25 t -30 t
∴i (t) = -100 e -30t +105 e =5 e +100 ( e -e )A

Example (2-5).
For the circuit shown in Fig. (2-9) find the mesh currents i1 (t)
and i2 (t).
10 Ω

i (t)
5Ω
V1 (t) 5Ω
150 sin (1000 t) i1 (t)
i2 (t)
0.01 H

Fig. 2-9

5× ( 5 + 0.01S ) 25 + 0.05 S +100 + 0.1S 125 + 0.15 S


Z(s) = +10 = =
10 + 0.01S 10 + 0.01S 10 + 0.01S
For S = J 1000
125 + J150
∴Z ( J1000) = =13.81∠5.19ο
10 + J10
150
∴I = =10.86 ∠ − 5.19ο
f 13.81∠ 5.19
if (t) = 10.86 sin ( 1000 t – 5.19 ) A
125 + 0.15 S
 Z (s) = For natural response Z (s) = 0
10 + 0.01S
∴ S = - 833.3
in (t) = I e -833.3t
i (t) = 10.86 sin ( 1000 t – 5.19 ) + I e -833.3t
i (0) = 0 = 10.86 sin ( -5.19 ) + I
∴ I = 0.982
So i (t) = 10.86 sin (1000 t – 5.19) + 0.982 e -833.3t A
V1 (t) =150 sin (1000 t)
– 10 {10.86 sin (1000 t – 5.19) + 0.982 e -833.3t }

41
Chapter 2 The Natural and Forced Responses of Simple Circuit

V1 (t) = 42.987 sin (1000 t + 13.2) – 9.82 e -833.3t V


∴ i2 (t) = 8.59 sin (1000 t + 13.2) – 1.96 e -833.3t A
i1(t) = i (t) - i2 (t) =10.86 sin (1000 t - 5.19) + 0.982 e -833.3t
- { 8.59 sin ( 1000 t + 13.2) - 1.96 e -833.3t }
= 3.83 sin (1000 t - 50.2) + 2.94 e -833.3t A

The step function response:-


V (t) = 0 for t ≤ 0 & V (t) = V for t ≥ ∞.
The similarity of this function to the exponential function
V (t) ≡ V e S t when S = 0

Example ( 2-6).
For the circuit shown in Fig. (2-10). Find iR (t) and iL (t).

RLS
Z(s) = for S = 0 then Z (s) = 0 Hence the forced response
R + LS
does not exist.
The natural response is to have a voltage when the current is
zero.
i (t)

10 A R i (0) = 3A
L
iR(t) IL (t)
Fig. 2-10

R
i. e. Z(s) = ∞ , then R + LS = 0 , So S = -
L
−R t
∴ V (t) = V e L
 iL (0) = 3, ∴ iR (0) = 7 So VR (0) = 7R
−R t −R t −R t
V (t) = 7R e L , Then iR (t) = 7 e L & iL (t) = 10 - 7 e L

42
Chapter 2 The Natural and Forced Responses of Simple Circuit

Example ( 2-7).
In the circuit shown in Fig.(2-11) the switch on position a for a
long time, then it moves to position b at t = 0 find;
a) iL (t) for t ≥ 0
b) The time passed for VL = 24 V.
c) Plot iL (t) and VL (t).
2Ω b a

24 V 200 m H 10 Ω 8A

Fig. 2-11

a- iL (0) = - 8 A
For the switch on b then Z (s) = 2 Ω
24
If = =12 A
2
−R t
∴ iL (t) = 12 + I e L = 12 + I e -10t
At t = 0 iL (0) = - 8
Then - 8 = 12 + I , So I = - 20
∴ iL (t) = 12 - 20 e -10t

d i (t)
b – VL (t) = L = 0.2 ( 200 e -10t ) = 40 e -10t
dt
24
∴ 24 = 40 e -10t , So e -10t = ,
40
5
Then t = 0.1 lin ( ) = 51.08 m s.
3

c- Fig. ( 2- 12) shows a plotting for both iL(t) and VL(t).

43
Chapter 2 The Natural and Forced Responses of Simple Circuit

V (t) i (t)
40 V

24 V VL (t)
12 A
iL (t)
51.08 m t
s
-8A
Fig. 2-12

Circuits Containing Capacitance And Resistance:-


For the circuit shown in Fig. (2-13) let i (t) = I eSt
i (t)

V (t) C

Fig. 2- 13

1 1 St 1
∴ V ((t) = ∫ i (t) dt = I e = V eSt , where V = I
C SC SC
Hence V (t) varies exponentially with the time. In the circuit
1
shown in Fig. (2-14) V (t) = R i (t) + ∫ i (t) dt + VC (0)
C
St
i (t) = I e

VR (t)
V (t)
VC (t)

Fig. 2-14

44
Chapter 2 The Natural and Forced Responses of Simple Circuit

1 St
∴ R I eS t + I e = V eS t
SC
1
∴ V = (R + ) I = I . Z (s)
SC
1
Z (s) = 0, At S = - . And Z (s) = ∞, at S = 0.
RC
Fig. (2-15) shows a sketch for Z (s).

Z (s)

R
Zero

S
1
-
RC
Pole
Fig 2-15

Example (2-8).
In the circuit shown in Fig. (2-16) the switch in position A for a
long time. Then and at t = 0 the switch is moved to position b
calculate.
a) The initial value of VC.
b) The final value of VC.
c) An expression for iC (t).
d) Plot VC (t) and iC (t) versus t.

400 K Ω 20 Ω
b a
90 V 0.5 µ F 60 Ω 40 V

Fig. 2-16

45
Chapter 2 The Natural and Forced Responses of Simple Circuit

1
× 60
SC 80 +1200 S C
a - At t < 0 , Z (s) = + 20 =
1
+ 60 1+ 60 S C
SC
40
For S = 0 , Z (s) = 80, so if = = 0.5 A
80
Then VC (t) = 0.5 × 60 = 30 V
Thus VC (0) = 30 V

b - Fig. (2-17) shows the circuit in position b with


VC (0) = 30 V.
1
Z (s) = R +  S = 0, So Z (s) = ∞ and if = 0
SC
For natural response Z (s) may equal zero, so
1 106
S=- =− =-5
RC 3
400 ×10 × 0.5
∴ in (t) = I e -5t
1
iC (t) = I e -5t V (t) = i (t) . R + ∫ i (t) dt + V (0)
C C
Then at t = 0
120 = i (0) × 400 × 103

400 K Ω

30 V
90 V
0.5 µ F

Fig. 2-17

= 0.3×10 − 3 = 300 μ A
120
∴ i (0) =
400 x 103

so, i (t) = 300 e -5t µ A

46
Chapter 2 The Natural and Forced Responses of Simple Circuit

∴ VC (t) = 90 – 400 × 103 × 300 × 10 – 6 × e -5t = 90 – 120 e -5t


at t = ∞, the voltage VC (t) may be, VC (∞) = 90

c - iC (t) = 300 e - 5 t µ A for t ≥ 0

d -  VC (t) = 90 – 120 e -5t


∴ VC (t) = 0 at 90 = 120 e -5t
t = 57.54 m sec.
−3
Then iC ( 57.54) = 300 e − 5× 57.54×10 = 225 µ A
Fig. (2-18) shows a free sketch for iC (t) and VC (t).

iC(t) VC(t)
300 µ A
225 µ A
VC(t)
90 V

VC(t
iC(t)
57.54) ms t
- 30 V
Fig. 2-18

Example (2-9).
For the circuit shown in Fig. (2-19) the switch is moved from
position 1 to position 2 after one time constant τ. Find i (t) for 0
< t < τ and for t > τ.

1 2 100 Ω
50 V 20 V
50 µ F

Fig. 2-19

47
Chapter 2 The Natural and Forced Responses of Simple Circuit

Fig. (2-20) shows the circuit in position 1.


1 106
Z (s) = R + = 100 +
SC 50 S
 S = 0, then Z (s) = ∞ , So if (t) = 0

50 V 100 Ω

50 µF

Fig. 2-20

For natural response Z (s) = 0


1
Then, R + =0
SC
1 106 -200 t
∴S=- =− = - 200 , in (t) = I e
R C 100 × 50
1
V (t) = i (t) . R + ∫ i (t) dt + V (0)
C C
Then at t = 0, 50 = i (0) × 100
∴ i (0) = 0.5 , So 0.5 = I
-200 t
i (t) = 0.5 e for 0 < t < τ .
-200 t -200 t
VC (t) = V – R. i (t) = 50 – 100 × 0.5 e = 50 (1 - e )
τ = R C = 100 × 50 × 10 = 5000 µ s
–6

-200 × 0.005
VC (τ) = 50 (1- e ) = 31.606 V
Fig. (2-21) shows the circuit in position 2.

100 Ω
20 V
50 µ F
31.606 V

Fig. 2-21

48
Chapter 2 The Natural and Forced Responses of Simple Circuit

-200 ( t - )
For t > τ, i (t) = I e
1
V (t) = i (t) . R + ∫ i (t) dt + V (0)
C C
20 + 31.606
Then, at t = 0 , i (0) = - = - 0.51606 A
100
-200 ( t - )
∴ i (t) = - 0.51606 e for t ≥ τ
-200 t
So, for 0 ≤ t < τ i (t) = 0.5 e A
At t = τ, i (τ) = 0.5 e = 0.184 A
-1

-200 ( - )
Also at t = τ, i (τ) = - 0.51606 e = - 0.51606 A
-200 ( t - )
At t > τ, i (t) = - 0.51606 e A
Fig. (2-22) shows a free sketch for i (t) versus t

i (t)
0.5

0.184 t
5000 µ s

- 0.51606

Fig. 2-22

The following tables summarize the previous items which had


been discussed through this chapter.

49
Chapter 2 The Natural and Forced Responses of Simple Circuit

Table 1 (With step function source)

R i (t) L R i (t) C

V (t) V (t) V (t)

1
Z R+ =∞
R+LS=R Z (s) SC
(s)
V V V
= =0
if (t) Z(s) R if (t) Z(s)
1
R S= − so
S= − so RC
L
in (t) in (t) − t
−R t in (t) = I e RC
in (t) = I e L
L RC
τ τ
R
Determine from V(0) = I R + VC (0)
V the initial V(0) − V (0)
I C
condition I I=
R
V −t V(0)− V (0) − t
C
i (t) + i (0) e τ i (t) e τ
R R

50
Chapter 2 The Natural and Forced Responses of Simple Circuit

Table 2 (With exponential function source)

R i (t) L R i (t) C

V (t) V (t) V (t)

1
Z R+
R+LS Z (s) SC
(s)

V VSC S t
eS t if (t) e
if (t) R + LS 1+ RSC

in (t) − Rt in (t) − t
in (t) = I e L in (t) = I e RC

L
τ RC
τ R
V VSC S t
eS t e
Ve st
R + LS 1+ RSC
it (t) −t it (t) −t
+I e τ +I e τ
V V(0)− V (0)
i (0) + I i (0) C
R + LS R

51
Chapter 2 The Natural and Forced Responses of Simple Circuit

Problems

1- For the circuit shown in Fig. (2-23), the switch has been in
position a for a long time. At t = 0, it moves to position b. Find;
a) i (t) for t ≥ 0. b) The total energy dissipated in the
8 Ω resistor.
30 Ω

a b i (t)
12 A 150 Ω 8 mH 8Ω 2 mH

Fig. 2-23

2- For the circuit shown in Fig. (2-24) at t = 0 each switch has


been in its position for long time. At t = 0 the two switches
moves to their new position for t ≥ 0 find ;
a) Vo (t). b) io (t)
c) The total energy dissipated in the 60 Ω resistor.
d) The energy trapped in the inductors.

t=0 io (t)
1.6 H

Vo (t)
10 Ω 60 Ω
4H t=0 6H
5A

Fig. 2-24

3- For the circuit shown in Fig. (2-25) the switch is moves to the
new position at t = 0 after a long time. Determine;
a) The current in ab branch at t = 0 and at t = ∞.
b) The time at which Iab = 19 A

52
Chapter 2 The Natural and Forced Responses of Simple Circuit

5Ω

5Ω 3Ω
165 V t=0
12.5 mH 3.75 mH

Fig. 2- 25

4- In the two mesh network shown in Fig. (2-26) the switch is


closed at t = 0. Find the currents i1 and i2 as given in the
diagram.
10 Ω

5Ω
150 sin 1000 t
5Ω
0.01 H

Fig. 2-26

5- Find the current in the inductance of the circuit illustrated in


Fig.(2-27-a) for the time interval 0≤ t ≤ 2.5 milliseconds, and
t ≥ 2.5 milliseconds. The waveform of the excitation Voltage
is shown in Fig.(2-27-b).

2Ω V (t)
100 V 1.25 ms
V 5Ω 4H t
2.5 ms
(t -100 V
)
a - The R L circuit b - The voltage wave form

Fig. 2-27

53
Chapter 2 The Natural and Forced Responses of Simple Circuit

6- For the circuit shown in Fig. (2-28) find the magnitude and
the direction of iab(t) after 3 ms.
1
μf
15

t = 0 ms
450 V
a
20 k Ω 30 k Ω
t = 1.6 ms

5 k Ω b 70 k Ω
Fig. 2-28

7- For the circuit shown in Fig. (2-29) calculate;


a) I (t), V1(t), and V2 (t)
b) The energy stored in the capacitors at t = 0.
c) The energy trapped in the circuit and dissipated in the 50
kΩ at t = ∞.

10 kΩ t=0 50 kΩ

i (t)
24 V 40 nf V1 (t) V2 (t)
160 nf

Fig. 2-29

8- For the circuit shown in Fig. (2-30) the two switches have
been closed for a long time. At t = 0 they are opened. Find;
a) io (t) for t ≥ 0.
b) Vo (t) for t ≥ 0.

54
Chapter 2 The Natural and Forced Responses of Simple Circuit

c) The energy trapped in the circuit.

8 kΩ t=0 20 kΩ t=0

io
180 V 120 kΩ 7.5 µf 15 (t)
µf Vo (t)
40 kΩ

Fig. (2-30)

9- For the circuit shown in Fig. (2-31) the capacitor has an initial
charge qo = 25× 10-6 coulombs with a polarity as shown in
the diagram. Obtain the current transient. What initial charge
on the capacitor will cause the current to go directly into the
steady state without a transient when the switch is closed.

t=0
500 Ω
100 sin (1000 t + 30)
qo - 0.5 µf
+

Fig. (2-31)

10 – In the circuit shown in Fig. ( 2- 32) the switch has been


open for a long time and is closed at t = 0. Drive an
expression for V (t) for t > 0.
15 Ω 5Ω

0.4 f V (t)
30 V 0.1 f

Fig. 2-32

55
Chapter 2 The Natural and Forced Responses of Simple Circuit

II - Second order systems:-

Circuit containing resistance, inductance and capacitance:-

Natural response of a parallel RLC circuit.

From the circuit shown in Fig. (2-33)

iR iC iL
I R V C L
t=0

Fig. 2-33

V dV 1 t
+C + V dt + I o
dt L 0∫
I = iR + iC + iL = ………….. 1
R
dI 1 dV V d2 V
= + +C =0
dt R dt L d t2
d2 V 1 d V 1
∴ + + V =0 ………… 2
d t2 R C d t LC
St
Assume V = A e
1 1
∴A S2 eS t + A S eS t + A eS t = 0
RC LC
1 1
∴A eS t ( S2 + S+ ) =0
RC LC
1 1 2 1
S= − ± ( ) −
2R C 2R C LC
1 1 2 1
S1 = − + ( ) −
2R C 2R C LC
1 1 2 1
S2 = − − ( ) −
2R C 2R C LC
S t S t
∴V = A e 1 & V=A e 2
1 2

56
Chapter 2 The Natural and Forced Responses of Simple Circuit

1 1
Let α = and ωo=
2R C LC
Then S1 = - α + α 2 − ωo2 & S2 = - α - α 2 − ωo2
If ω 2 < α 2 then both roots will be real and distinct, the voltage
o
response is said to be over- damped .
While if ω o2 > α 2 both roots S1 and S2 will be complex and
conjugates of each other. In this situation the voltage response is
said to be under- damped.
If ω o2 = α 2 then S1 and S2 are equal and real, the voltage
response is critically- damped.
S t S t
If V = A e 1 & V=A e 2
1 2
The sum of V1 and V2 is also a solution.
S t S t
V=A e 1 + A e 2 …………… 3
1 2
dV S t S t
=A S e 1 +A S e 2
dt 1 1 2 2
d2 V S t S t
= A S2 e 1 + A S2 e 2
d t2 1 1 2 2
According to equation 2,
d2 V 1 d V 1
+ + V =0
dt 2 R C d t L C
S t 1 1 S t 1 1
A e 1 ( S2 + S + ) + A e 2 ( S2 + S + )=0
1 1 R C 1 LC 2 2 R C 2 LC
S t S t
∴ V = A e 1 + A e 2 is a solution.
1 2

Determining the constants A1 and A2for the following cases;

a) Over – damped case.


When ω o2 < α 2 the roots of the characteristic equation are
real and distinct.

57
Chapter 2 The Natural and Forced Responses of Simple Circuit

Equation 3 at zero time may be; Vo = A1 + A2


V dV
Rewriting equation 1 at zero time as; o + C o + I o = 0
R dt
d Vo V I
Then =− o − o …………. 4
dt RC C
d Vo
Also =A S +A S …………. 5
dt 1 1 2 2
Solving equations 4 and 5 for A1 and A2, hence;
1 V 1 V
Vo S + { o + I o } Vo S + { o + I o }
2 C R 1 C R
A1 = & A2 =
S −S S −S
2 1 2 1
Fig. (2- 34) shows a free sketch for the voltage variation
against the time.
V (t)

Fig. 2- 34

b) Under – damped case.


When ω o2 > α 2 the roots of the characteristic equation are
complex then;
Then S1 = - α + α 2 − ωo2 = - α + J ωo2 − α 2 = - α + J ωd
And S2 = - α - J ωd , where ωd = ωo2 − α 2
(− α + J ω ) t −(α + Jω ) t
V (t) = A e d +A e d
1 2
−α t
=e { A1 cos ωd t + J A1 sin ωd t + A2 cos ωd t
- J A2 sin ωd t}
−α t
=e { (A1 + A2 ) cos ωd t + J (A1 – A2 ) sin ωd t }

58
Chapter 2 The Natural and Forced Responses of Simple Circuit

−α t
=e ( B1 cos ωd t + B2 sin ωd t )
Where B1 = A1 + A2 and B2 = J ( A1 – A2 )
For obtaining B1 and B2 the initial conditions may be applied
as follows;
At t = 0, V (0) = B1 and
d V −α t
=e { − ω B sin ω t + ω B cos ω t}
dt d 1 d d 2 d
− α e − α t { B cos ω t + B sin ω t}
1 d 2 d
dV −α t
=e { (ω B − α B ) cos ω t − ( ω B + α B ) sin ω t}
dt d 2 1 d d 1 2 d

d V (0)
At t = 0 =ω B −αB
dt d 2 1
d Vo V I
Also from equation 4 = − o − o Then,
dt RC C
α 1 V α 2 α Vo I
B2 = V (0) − ( o + Io ) = V (0) − − o
ω ω C R ω ω ω C
d d d d d
α 2 R α I o = − α ( V (0) + 2 I R )
B2 = - V (0) − o
ω ω ω
d d d
∴V (t) = V (0) e − α t cos ω t − ( V(0) + 2 I o R) e − α t sin ω t
α
d ω d
d

Fig (2-35) shows a free sketch for the voltage response at this
case.
V (t)

Fig. 2-35

59
Chapter 2 The Natural and Forced Responses of Simple Circuit

c) Critically damped case.


The roots of the characteristic equation are real and equal.
Then S1 = S2 = - α
−α t
∴ V(t) = (A1 + A2) e
The previous equation can not satisfy two initial conditions V(0)
& Io because it has only one arbitrary constant Ao. For this the
solution may be;
−α t
V(t) = ( D1 t + D2 ) e
=D e −α t −αe −α t (D t + D )
d V(t)
dt 1 1 2
= e − α t { (1− α t) D − α D }
1 2
d V(0) 1 V (0)
At t = 0 V(0) = D2 And =D −α D =- ( + Io )
dt 1 2 C R
1 V (0)
∴ D1 = α V(0) − ( + I o ) = α V(0) - 2 α V(0) - 2 α R I o
C R
= - α{ V(0) + 2 R I o }
Fig. (2-36) shows the voltage response for the over-damped case.
V(t)

Fig. 2-36 t

It is possible to obtain the solution relative to the inductor


current iL(t).
S t S t
 V = A e 1 + A e 2 or
1 2
V= e − α t ( B1 cos ωd t + B2 sin ωd t ) or
−α t
V = ( D1 t + D2 ) e
1t
i (t) = ∫ V(t) dt + I
L L0

60
Chapter 2 The Natural and Forced Responses of Simple Circuit

S t S t
Then i (t) = I + A′ e 1 + A′ e 2 or
L 1 2
=I+ e − α t ( B′ cos ω t + B′ sin ω t ) or
1 d 2 d
=I+ e − α t ( D′ t + D′ )
1 2

Example (2- 10).


For the circuit shown in Fig. ( 2-38) Find;
a) The initial value of i .
L
di
b) The initial value of L
dt
c) The roots of the characteristic equation.
d) The numerical expression for i (t) for t ≥ 0.
L
e) For R = 625 Ω & R = 500 Ω find i (t) for t ≥ 0.
L
f) Plot i (t) in each case of the given values of R over the
L
range 0 ≤ t ≥ 220 µ S.

iL
24 mA 25 nf 25 mH
t=0 400 Ω

Fig. 2-38

a- i (0) = 0
L

d i (0)
b- L =0
dt
1 109
c- α = = = 5x 10 4
2 R C 2 x 400 x 25
1 103 x 109
ωo2 = = =16x 108
L C 25 x 25
Then α 2 > ωo2 so the system is overdamped;

61
Chapter 2 The Natural and Forced Responses of Simple Circuit

∴ S1 = − α + α 2 − ωo2 = − 5 x 10 4 + 3 x 10 4 = − 2 x 10 4
& S2 = - 5 × 104 – 3 × 104 = - 8 × 104

S t S t
d- i (t) = I + A′ e 1 + A′ e 2
L 1 2
i (0) = 0
L
0 = I + A′ + A′
1 2
d i (0)
So L = 0 Then 0 = S A′ + S A′
dt 1 1 2 2
Solving for A ′ & A ′ then;
1 2
−S I 4 −3
A′ = 2 = − 8 x 10 x 24 x 10 = − 32 x 10 − 3 A
1 S −S 6 x 10 4
2 1
S I 4 −3
A′ = 1 = 2 x 10 x 24 x 10 = 8 x 10 - 3 A
2 S −S 6 x 10 4
2 1
∴ i (t ) = 24 − 32 e - 20000 t + 8 e − 80000 t mA for t ≥ 0.
L

e- For R = 625 Ω
1 109
α= = = 3.2 x 10 4
2 R C 2 x 625 x 25
∴α 2 < ωo2 So the system is under damped.
S1 = − α + J ωo2 − α 2 = − 3.2 x 10 4 + J 2.4 x 10 4

S2 = − α − J ωo2 − α 2 = − 3.2 x 10 4 − J 2.4 x 10 4


i (t) = I + e − α t ( B′ cos ω t + B′ sin ω t )
L 1 d 2 d

i (0) = I + B = 0
L 1
d i (0)
L = -  B′ + B′ ω = 0
dt 1 2 d
α = 3200 0 , ω = 24000 rad / sec
d
and I = 24 mA

62
Chapter 2 The Natural and Forced Responses of Simple Circuit

∴ B′ = − I = − 24 mA
1
αI 32 x 24
B′ = − =− = − 32 mA
2 ω 24
d
i (t)=24 - e − 32000 t (24 cos 24000 t + 32 sin 24000 t ) mA
L
for t ≥ 0.

For R = 500 Ω
1 109
α= = = 4 x 10 4
2 R C 2 x 500 x 25
Then α 2 = ωo2 So the system is critically damped.
∴S = S
1 2
i (t) = I + e − α t ( D′ t + D′ )
L 1 2

i (0) = I + D = 0
L 2
d i (0)
L = D′ −  D′ = 0
dt 1 2
D′ = α D′ = − α I = − 40000 x 24 = − 960000 mA / S
1 2
D′ = − I = − 24 mA
2
i (t) = 24 - e − 40000 t ( 960000 t + 24) mA
L
for t ≥ 0.

f- Fig. (2-39) shows a free sketch for the changes of i L (t)


against the time t for different values of R for t ≥ 0.

iL (t)
Under damped R = 625 Ω

24 mA
Critically damped R = 500 Ω
Over damped with R = 400 Ω
t µS
63
Chapter 2 The Natural and Forced Responses of Simple Circuit

Fig. 2-39

Natural response of a series RLC circuit with a step function


source.

For the circuit shown in Fig. (2-40) the voltage equation may be;
R L C

t=0
Vo i (t)
V (t)

Fig. 2-40

d i (t) 1 t
V (t) = R i (t) + L + i (t) d t + Vo
d t C 0∫
d i (t) 1 t
For natural response 0 = R i (t) + L + i (t) d t + Vo
d t C 0∫
d i (t) d 2 i (t) 1
Then R +L + i (t) = 0
dt d t2 C
d 2 i (t) R d i (t) 1
+ + i (t) = 0
d t2 L dt LC
St
Letting i (t) = A e so,
St R 1
A e { S2 + S + }= 0
L LC
R R 2 1
S= − ± ( ) −
2L 2L LC
S1,2 = − α ± α 2 − ωo2
S t S t
Thus i (t) = A1 e 1 + A e 2 For over damped case.
2
Or i (t) = e − α t ( B1 cos ωd t + B2 sin ωd t ) For under damped
case.
Or i (t) = ( D1 t + D2 ) e − α t For critically damped case.

64
Chapter 2 The Natural and Forced Responses of Simple Circuit

Once the current natural response is obtained, it's easy to obtain


the voltage natural response across any element. Say the natural
voltage response across the capacitor is needed;
1
VC (t) = ∫ i (t) d t
C
S t S t
VC (t) = Vf + A′ e 1 + A′ e 2 For over damped case.
1 2
VC (t) = Vf + e − α t ( B′ cos ω t + B′ sin ω t ) For under damped
1 d 2 d
case.
VC (t) = Vf + e − α t ( D′ t + D′ ) For over damped case.
1 2

Example (2- 11).


For the circuit shown in Fig. (2- 41) find.
d i (0)
i (0), VC (0), , i (t) for t ≥ 0 and VC (t) for t ≥ 0.
dt

19.2 k Ω a b 100 Ω 10 mH
t=0
50 V 12.8 k Ω 2µF 100 V
i (t)

Fig. 2 – 41

For the switch in position a ;


i (0) = 0
50 ×19.2
VC (0) = 50 - = 20 V
19.2 +12.8
d i (t) 1
R i (t) + L + i (t) d t + V (0) = V (t)
dt C ∫ C
d i (0)
At t = 0, then L + VC (0) = V
dt
d i (0) V − VC (0) 100 − 20
= = = 8000 A / S
dt L 10 x 10 −3

65
Chapter 2 The Natural and Forced Responses of Simple Circuit

1
Z (S) = R + L S + =0
SC
R 1
S2 + S+ =0
L LC
R R 2 1
S= − ± ( ) − = − α ± α 2 − ωo2
2L 2L LC
R
α= = 5000
2L
1
ωo2 = = 5 × 107
LC
Then ωo2 >  2 , So the system is under damped.
ω = ωo2 − α 2 = 5×107 − 25×106 = 5000
d
S1 = - α + J ω = - 5000 + J 5000
d
S2 = - α - J ω = - 5000 - J 5000
d
i (t) = e − α t ( B1 cos ωd t + B2 sin ωd t )
 i (0) = 0 So, B1 = 0
d i (0)
= −α B + ω B = 8000
dt 1 d 2
8000
B2 = = 1 .6
5000
− 5000 t
∴i (t) = 1.6 e sin 5000 t For t ≥ 0
VC (t) = Vf (t) + e − α t ( B′ cos ω t + B′ sin ω t )
1 d 2 d
 VC (0) = Vf (0) + B′
1
B′ = VC (0) – Vf (0) = 20 -100 = - 80
1
d V (0) i (0)
C = =0
dt C
− α B′ + ω B′ = 0
1 d 2
B = B′ = − 80

2 1
− 5000 t
∴VC (t) = 100 – 80 e ( cos 5000 t + sin 5000 t )

66
Chapter 2 The Natural and Forced Responses of Simple Circuit

Example (2- 12).


For the circuit shown in Fig. (2-42) determine the natural current
response for R = 10, 6, 1 and 0, when the switch is closed with
an initial voltage of 35 V on the capacitor.

R 0.5 H 0.0556 F

V(0)=35

t=0

Fig. 2-42

For R = 10 
R 1
α= =10 , ωo2 = = 36
2L LC
ωo2 < α 2 So it's over damped
S t S t
i (t) = A e 1 + A e 2
1 2
S1 = - α + α 2 − ωo2 = - 2
S2 = - α − α 2 − ωo2 = - 18
i (t) = A e - 2 t + A e -18 t
1 2
i (0) = 0 , hence A1 + A2 = 0
d i (0) V 35
=− o = = 70 A / S
dt L 0.5
- 2 A1 – 18 A2 = 70
70 70
A1 = & A2 = -
16 16
70 − 2 t −18 t
i (t) = (e –e )
16

For R = 6 
R 1
α= = 6 , ωo2 = = 36
2L LC

67
Chapter 2 The Natural and Forced Responses of Simple Circuit

ωo2 = α 2 So it's critically damped


i (t) = ( D t + D ) e − α t
1 2
i (0) = D2 = 0
d i (0) V 35
=− o = = 70 A / S
dt L 0.5
70 = D1 - α D2
∴ D1 = 70
−6t
i (t) = 70 t e

For R = 1 
R 1
α= =1 , ωo2 = = 36
2L LC
ωo2 > α 2 So it's under damped case.
i (t) = e − α t ( B1 cos ωd t + B2 sin ωd t )
ω = 36 −1 = 35
d
−t
i (t) = e ( B1 cos 35 t + B2 sin 35 t )
i (0) = 0 , B1 = 0
d i (0) V 35
=− o = = 70 A / S
dt L 0.5
- B1 + 35 B2 = 70
70
B2 = = 2 35
35
i (t) = 2 35 sin 35 t

For R = 0 
R 1
α= = 0 , ωo2 = = 36
2L LC
ωo2 > α 2 , So it's under damped case.
i (t) = e − α t ( B1 cos ωd t + B2 sin ωd t )
ω = 36 − 0 = 6
d
i (t) = ( B1 cos 6 t + B2 sin 6 t )

68
Chapter 2 The Natural and Forced Responses of Simple Circuit

i (0) = 0 , B1 = 0
d i (0) V 35
=− o = = 70 A / S
dt L 0.5
70
6 B2 = 70 , B2 =
6
70
i (t) = sin 6 t
6

Example (2-13).
For the circuit shown in Fig. (2-43) calculate the response of the
currents i1 (t) and i2 (t).
5Ω

t=0 i2 (t) 0.1 H


20 µ F i1 (t)
50 V
5Ω

Fig. 2- 43
106
(5 + 0.1S )
20 S 0.5 S2 + 5025 S + 5 x 105
Z (s) = +5 =
106 0.1S2 + 5 S + 5 x 10 4
5 + 0.1S +
20 S
For forced response S = 0, So;
Z (s) = 10
50
If = =5 A
10
For natural response Z (s) = 0 , 0.5 S2 + 5025 S + 5 x 105 = 0
10050 10050 2
S− ± ( ) −106 = − 5025 ± 4924.49
2 2
S1 = - 100.51 & S2 = - 9949.49
−100.51t − 9949.49 t
in (t) = A1 e + A2 e
−100.51t − 9949.49 t
i (t) = 5 + A1 e + A2 e

69
Chapter 2 The Natural and Forced Responses of Simple Circuit

d i (0)
i (0) = 0 & =0
dt
5 + A1 + A2 = 0
- 100.51 A1 – 9949.49 A2 = 0
A1 = - 5.051 & A2 = 0.051
−100.51t − 9949.49 t
i (t) = 5 – 5.051 e + 0.051 e
−100.51t − 9949.49 t
V (t) = 50 – 5 { 5 – 5.051 e + 0.051 e }
−100.51t − 9949.49 t
V (t) = 25 + 25.255 e - 0.255 e
d V (t) −100.51t
i2 (t) = C = 20 × 10 6 { - 100.51 × 25.255 e
dt
− 9949.49 t
+ 9949.49 × 0.255 e }
−100.51t − 9949.49 t
= - 0.05078 e + 0.05074 e
−100.51t − 9949.49 t
i1 (t) = i (t) – i2 (t) = 5 – 5.051 e + 0.051 e
−100.51t − 9949.49 t
+ 0.05078 e - 0.05074 e
−100.51t − 9949.49 t
= 5 – 5.0002 e + 0.0002 e

Example (2-14).
For the circuit shown in Fig. (2-44) find iL(t) if the switch is
closed for a long time and is opened at t = 0.
t=0
5Ω

i2 (t) 0.1 H
20 µ F i1 (t)
50 V
5Ω

Fig. 2 – 44

At t = 0 iL(0) = 5 , iC (0) = 0 and VC (0) = 25


The circuit after opening the switch may be as shown in
Fig. ( 2-45).

70
Chapter 2 The Natural and Forced Responses of Simple Circuit

5Ω 0.1 H 20µ F

25 V

Fig. 2- 45

1
Z (s) = R + LS + , For S = 0 Z (s) = ∞
SC
So If = 0 and if (t) = 0
For natural response;
R 1
S2 + S + =0
L LC
R 5
α=− =- = − 25
2L 0.2
1
ωo2 = = 5 x 105
LC
So ωo2 >  2 , then its under damped case;
ω = 5 x 105 − 625 = 706.66
d
− 25 t
iL (t) = e ( B1 cos 706.66 t + B2 sin 706.66 t )
d i (0) V 25
At t = 0 iL (0) = 5 and L =− o =− = - 250
dt L 0.1
∴B1 = 5, and - 25 B1 + 706.66 B2 = - 250
125
B2 = - = - 0.177
706.66
− 25 t
∴ iL (t) = e ( 5 cos 706.66 t – 0.177 sin 706.66 t )

71
Chapter 2 The Natural and Forced Responses of Simple Circuit

The Sinusoidal Forcing Function;

The most important forcing function is that which varies with


time according to a sin or a cos function.
V (t) = V cos ω t
Expressing the sinusoidal as exponential;
e ± J ω t = cos ω t ± J sin ω t
cos ω t = Re. e J ω t & sin ω t = Im. e J ω t

Example (2- 15).


For the circuit shown in Fig. (2-46) calculate the current
response if the voltage is applied at the instant when V (t) = 8 V
and increasing. Assume there is no energy stored in the circuit.

1Ω 0.5 H 0.0556 F

10 cos (ω t + θ )
For t ≥ 0

Fig. 2-46

Fig. (2-47) shows the waveform of the supply voltage.

V (t)

10
8
θ
36o 52\

Fig. 2-47

At t = 0 the supply voltage may be 8 = 10 cos θ


θ = - 36o 52\
For forced response;

72
Chapter 2 The Natural and Forced Responses of Simple Circuit

1 18
Z (s) = R + LS + = 1 + J 9 × 0.5 – J = 1 + J 2.5
SC 9
= 2.693 ∠ 68o 12\
V 10 ∠ − 36o 52 \
If = = = 3.7139 ∠ - 105o 4\
Z (s) 2.693 ∠ 68o 12 \
if (t) = 3.7139 cos ( 9 t – 105o 4\ )
For natural response;
R 1
α= =1 & ωo2 = = 36
2L LC
ωo2 > α 2 , So the system is under damped.
in (t) = e − α t ( B1 cos ωd t + B2 sin ωd t )
−t
=e ( B1 cos 35 t + B sin 35 t )
2
i (t) = 3.7139 cos ( 9 t – 105 4\ )o

−t
+e ( B1 cos 35 t + B sin 35 t )
2
o \
i (0) = 3.7139 cos ( – 105 4 ) + B1 = 0
B1 = 0.9654
d i (0) Vo 8
= = =16
dt L 0.5
- B1 + 35 B - 9 × 3.7139 sin ( - 105o 4\ ) = 16
2
16 + 0.9654 − 32.276
B2 = = - 2.588
35
i (t) = 3.7139 cos ( 9 t – 123o 4\ )
+ e - t ( 0.9654 cos 35 t - 2.588 sin 35 t )
i (t) = 3.7139 cos ( 9 t – 105o 4\ )
−t
+ 2.762 e cos ( 35 t + 69o 33\ )

73
Chapter 2 The Natural and Forced Responses of Simple Circuit

The Laplace Transformation

The laplace transformation will be used to determine the total


response. For the circuit shown in Fig. (2-48)
i (t) R

V (t) L

Fig. 2-48

Applying the second Kirchhoff's law on the circuit which had


been shown in Fig (2-48)then;
d i (t)
R i (t) + L = V (t)
dt
For driving the Laplace transform, each element is multiplied by
e – S t (where S is general complex).
Then the equation is integrated over the range 0 → ∞ . The zero
is chosen to include all the initial states in the circuit ( i. e. the
current in inductive branches and the stored charges or initial
voltages on the capacitors).
∞ ∞ d i (t) − S t ∞
∴R ∫ i (t) e − S t d t + L ∫ e d t = ∫ V (t) e − S t d t
0 0 dt 0
∞ d i (t) ∞
L∫ e − S t d t = L [ e − S t i (t) ]∞
0 + L S ∫ i (t) e − S t d t
0 dt 0

= - L i (0) + L S ∫ i (t) e − S t d t
0
∞ ∞
∴ - L i (0) + (L S + R) ∫ i (t) e − S t d t = ∫ V (t) e − S t d t
0 0
∞ − S t d t is called Laplace transform of f (t) and is
∫ f (t) e
0
denoted by F (S) .

74
Chapter 2 The Natural and Forced Responses of Simple Circuit


∴ F (S) = ∫ f (t) = ∫ f (t) e − S t d t
0
Laplace transform is exists only for t > 0 and must converge at
the upper limit for some values of S.
∞ ∞
∴I (S) = ∫ i (t) e − S t d t & V (S) = ∫ V (t) e − S t d t
0 0
∴ (R + L S) I (S) = V (S) + L i (0)
V (S) + L i (0)
I(S) =
R + LS

Some Laplace transforms evaluated;



1- ∫ V = ∫ V e − S t d t = V [ − e − S t ]∞ =
1 V
S 0 S
0
at ∞ ∞
2 - ∫ e = ∫ e a t e − S t d t = ∫ e (a − S ) t d t
0 0
1 − ( S - a) t ∞ 1
=[ e ] = for S > a
a -S 0 S- a
-a t 1
3- ∫ e =
S+ a
4 - ∫ [ f1 (t) + f2 (t) ] = ∫ f1 (t) + ∫ f2 (t)
±a t 1 1 a
5- ∫ (1 – e )= − =
S S a S(S a )
e ωt +e −ωt 1 1 1 S
6- ∫ cosh ω t = ∫ = [ + ]=
2 2 S − ω S + ω S2 − ω 2

e Jω t + e −J ω t 1 1 1 S
7- ∫cosω t =∫ = [ + ]=
2 2 S − J ω S + J ω S2 + ω 2
e Jω t − e −J ω t 1 1 1 
8- ∫sinω t =∫ = [ − ]=
2 J2 S − J ω S + J ω S2 + ω 2
∞ ∞ - e -S t
9- ∫ t = ∫ t e − S t d t = [ − e − S t ]∞ − ∫
t
dt
S 0 S
0 0
- S t
]∞ =
1 -e 1
= [
S S 0 S2

75
Chapter 2 The Natural and Forced Responses of Simple Circuit

∞ ∞
= ∫ t e a t e − S t d t = ∫ t e − ( S − a) t d t =
at 1
10- ∫ t e
0 0 (S - a) 2

f (t) = ∫ f(t) e − a t e − S t d t
-a t
11- ∫ e
0

= ∫ f(t) e − ( S + a) t d t = f (S + a)
0
-a t S+ a
12- ∫ e cos ω t =
( S + a ) 2 + ω2

Expansion by partial fraction.


P (S)
Let F (S) =
Q (S)
Where Q (S) = a ( S + S1) ( S + S2) …………. ( S + Sn)
The roots of Q (S) are one of the following;
1- Simple and non repeated real.
2- Conjugate complex pairs.
3- Repeated.
4- Combination.

1- Simple and non- repeated.


P (S) P (S)
=
Q ( S ) ( S + S ) ( S + S ) ..........( S + Sn )
1 2
A A
= 1 + 2 + ……….. + A n
S+S S+S S + Sn
1 2

2- Conjugate complex.
2S + 5
Let F (S) =
S2 + 6 S + 34
2S + 5 2S + 5
∴F (S) = =
(S + 3 ) 2 + 25 ( S + 3 + J 5) ( S + 3 − J 5)
A A
= 1 + 2
S+ 3+ J 5 S+ 3− J 5
2 S + 5 = A1 (S + 3 – J 5) + A2 (S + 3 + J 5)

76
Chapter 2 The Natural and Forced Responses of Simple Circuit

− 6 + J10 + 5
For S = - 3 + J 5 A2 = =1+ J 0.1
J10
− 6 − J10 + 5
And for S = - 3 - J 5 A1 = =1− J 0.1
- J10
2S + 5 1- J 0.1 1+ J 0.1
∴ = +
S2 + 6 S + 34 S + 3 + J 5 S + 3 − J 5

3- Repeated roots.
P (S) P (S)
F(S) = =
Q (S) (S + S )n
1
A A An
= 1 + 2 + ..........
(S + S ) n (S + S ) n − 1 (S + S )
1
1 1

Example (2- 16).


2 S2 + 3S + 2
If F (S) = , find f (t).
( S +1) 3
A A A
F (S) = 1 + 2 + 3
(S +1)3 (S +1) 2 (S +1)
2 S2 + 3S + 2 = A1 + A2 (S + 1) + A3 (S + 1)2
For S = - 1 , 1 = A1
Equating the coefficients of S2, 2 = A3
Also equating the coefficients of S, 3 = A2 + 2 A3 , then A2 = - 1
1 1 2
∴ F (S) = − +
(S +1)3 (S +1) 2 (S +1)
t2 − t
Thus, f (t) = e − t e − t + 2 e− t
2

The differentiation theorem.


d f (t) ∞ d f (t) − S t -S t ∞
∫ f ′ (t) = ∫ = ∫ e d t = [ f (t) e e ]0
dt 0 d t

+ S ∫ f(t) e − S t d t = - F (0) + S F (S)
0

77
Chapter 2 The Natural and Forced Responses of Simple Circuit

∫ f ′′ (t) = S ∫ f ′ (t) - f ′ (0) = S { S F (S) – F (0) } - F′ (0)


= S2 F (S) – S F (0) - F′ (0)

∫ f n (t) = S n F (S) – S n – 1 F (0) – S n – 2 F′ (0)………


…………… S F n – 2 (0) – F n -1 (0)

The integral theorem.


∞ − S t dt
∫ [ ∫ f (t) dt ] = ∫ ∫ f(t) d t . e
0
1∞
= [ − e − St ∫ f (t) d t ]∞ + ∫ f (t) e − S t d t = N(0) + ∫ f (t)
1 1 1
S 0 S S S
0
Where N (0) is the initial value of ∫ f (t) d t
i. e. N (0) = ∫ f (t) d t at t = 0
1 1
∴ ∫ [∫ f (t) dt ] = ∫ f (t) + N (0)
S S

Circuit elements in the S domain.

The resistance; i (t) R


In time domain;
V (t) = R * i (t) V (t)

In S domain;
V=I*R I R
Where V = ∫ V (t) and I = ∫ i (t) (t)
V
The inductance; i(t) L
In time domain;
d i (t)
V (t) = L V (t)
dt

L Io (V)
SI L
In S domain;
S
V = L { S I – Io }
V

78
Chapter 2 The Natural and Forced Responses of Simple Circuit

Or in other form as follows; V


V + L Io V Io LS
I= = +
SL LS S I

Io
(A)
The capacitance; S
In time domain;
Vo
d V (t) i (t)
i (t) = C
dt
C
V (t)
1
In S domain; SC
I
I = C { S V – Vo }
V

C Vo
Or as; 1 Vo
( V)
SC S
1 V I
V= I+ o
SC S
V
Example (2 -16).
For the circuit shown in Fig. (2-49) if the switch is removed to
position b at t = 0. Find I, V1 and V2 in the S domain, then find
i (t), V1 (t) and V2 (t) in the time domain.
10 K t=0
Ω a b
0.5 µ F V1(t)
60 V
1.0 µ F 3KΩ
V2(t)

Fig. 2- 49

79
Chapter 2 The Natural and Forced Responses of Simple Circuit

Fig. (2-50) shows the previous circuit with the switch in position
a.
10 k Ω

0.5 µ F V1(t)= 40 V
60 V
1.0 µ F V2(t)= 20 V

Fig. 2 – 50

Fig. (2-51) shows the circuit with the switch in position b in the
S domain.

2×106
I
S I
V1
40
S 3kΩ
10 6
V2 S
20
S

Fig. 2- 51
3
V1 + V2 = - 3 × 10 I
40 2 ×106 20 106
+ I+ + I = − 3 × 103 I
S S S S
60 3 × 106
+ I = − 3 × 103 I
S S
60
− − 60 − 20 × 10 − 3
I= S = =
3 × 10 6 3× 103 S + 3 × 106 S +1000
3× 103 +
S

80
Chapter 2 The Natural and Forced Responses of Simple Circuit

40 20 ×10 − 3 2 ×106 40
V1 = − × =
S S +1000 S S +1000
20 20 ×10 − 3 106 20
V2 = − × =
S S +1000 S S +1000
-1000 t
So, i (t) = - 0.02 e A
-1000 t
V1 (t) = 40 e V
-1000 t
V2 (t) = 20 e V

Example (2-17).
For the circuit shown in Fig. (2- 52) find iL (t) if the switch is
opened at t = 0.

iL
24 mA 625 Ω 25 nf 25 mH
t=0

Fig. 2 – 52

Fig. ( 2-53) shows the previous circuit in the S domain.

IL
24x10 −3 40x106 -3
625 25×10 S V
S S

Fig. 2 -53
24 x 10 − 3 VS V V
= + +
S 40 ×106 625 25×10 − 3 S
S 1 1
= V[ + + ]
40 ×106 625 25×10 − 3 S

81
Chapter 2 The Natural and Forced Responses of Simple Circuit

96 ×10 4
V=
S2 + 64 ×103 S +16 ×108

V 96 ×10 4
IL = =
L S 25×10- 3 S (S2 + 64 ×103 S +16 ×108 )

384 ×105
=
S ( S + 32 ×103 − J 24 ×103 ) (S + 32 ×103 + J 24 ×103 )

A A A
= 1+ 2 + 3
S ( S + 32 ×103 − J 24 ×103 ) ( S + 32 ×103 + J 24 ×103 )

So; A1 = 24 × 10 – 3 , A2 = 20 × 10 – 3 ∠ 126.87 o

And A3 = 20 × 10 – 3 ∠ -126.87 o

24 ×10-3 20 ×10- 3 ∠126.87 o


∴IL = +
S ( S + 32 ×103 − J 24 ×103 )
20 ×10- 3 ∠ − 126.87 o
+
( S + 32 ×103 + J 24 ×103 )

- 32000 t J( 24000 t +126.87)


iL (t) = 24 + 20 e {e
- J( 24000 t +126.87)
+e }

- 32000 t
= 24 + 40 e cos (24000t + 126.87) mA

Example (2-18).
For the circuit shown in Fig. (2-54) if the switch is opened at
t = 0, compute;
a) The currents i1 (0) & i2 (0).
b) The currents i1 (t) & i2 (t).
c) The voltage V (t).

82
Chapter 2 The Natural and Forced Responses of Simple Circuit

i1 (t) i2 (t)
4 kΩ 16 kΩ V(t)
5A 0.8 kΩ (t)
8 mH 2 mH
Fig. 2 – 54

a- As t → ∞ the coils may be short circuit , So;


4 x 16
Req = = 3.2 k Ω
20
0.8
i1 (0) + i2 (0) = x 5 =1 A
3.2 + 0.8
16 4
i1 (0) = x 1= 0.8 A & i2 (0) = x 1= 0.2 A
20 20

b- The circuit after opening the switch simulated in the S domain


is shown in Fig. (2-55);

6.4 × 10- 3 A 0.4 x 10- 3 A


I1 I2
V
8 x 10-3 S 2 x 10-3 S

4 × 103 116 x 103

Fig. 2-55

V = I2 (16 × 10 3 + 2 × 10 - 3 S) – 0.4 × 10 - 3
= I1 (4 × 10 3 + 8 × 10 - 3 S) – 6.4 × 10 - 3
 I1 = -I2
3 -3 -3
∴ I1 (20 × 10 + 10 × 10 S) = 6 × 10

83
Chapter 2 The Natural and Forced Responses of Simple Circuit

6 ×10 − 3 0.6
I = =
10 − 2 S + 20 ×10 3 S + 2 ×10 6
1

i1 (t) = 0.6 e − 2 ×10 t & i2 (t) = - 0.6 e − 2 ×10 t


6 6

( 4 ×103 + 8×10 − 3 S ) − 6.4 ×10 − 3


0.6
c- V =
S + 2 ×10 6
7.2 ×10 − 3
= −{1.6 ×10 − 3 + }
S + 2 ×10 6

V (t) = -1.6 × 10 – 3 δ (t) – 7.2 × 10 – 3 e − 2 ×10 t


6

Example (2- 19).


For the circuit shown in Fig. (2-56) calculate the current
response if the voltage is applied at the instant when V (t) = 8 V
and increasing. Assume there is no energy stored in the circuit.

1Ω 0.5 H 0.0556 F

10 cos (9 t + θ )
For t ≥ 0

Fig. 2 – 56

V (t) = 10 cos (9t – 36o 52\ ) = 8 cos 9 t + 6 sin 9 t


8S 6× 9 8 S + 54
V (s) = + =
S 2 + 81 S 2 + 81 S2 + 81
The circuit in the S domain is shown in Fig. (2-57).
8 S + 54 18 S 2 + 2 S + 36
= I ( 1 + 0.5 S + )=I
S2 + 81 S 2S
2 S (8S + 54) 16 S2 +108
I= =
(S 2 + 81) (S2 + 2 S + 36) (S 2 + 81) (S2 + 2 S + 36)

84
Chapter 2 The Natural and Forced Responses of Simple Circuit

18
I 1 0.5 S S

8S + 54
S2 + 81

Fig. 2- 57

AS+ B CS + D
I= +
S2 + 81 S2 + 2 S + 36
− 0.9656 S + 32.276 0.9656 S − 14.3448
= + =I +I
2
S + 81 2
S + 2 S + 36 1 2
S 32.276 9
Where, I1 = − 0.9656 + x
S2 + 81 9 S2 + 81
So, i1 (t) = - 0.9656 cos 9t + 3.586 sin 9t = 3.713 cos (9t – 105o
4\ )
0.9656 S − 14.3448
And I2 =
S2 + 2 S + 36
S +1 15.31 35
= 0.9656 − ×
(S +1) 2 + 35 35 (S +1) 2 + 35
So, i2 (t) = e − t { 0.9656 cos 35 t − 2.588 sin 35 t }
= e − t { 2.7622 cos ( 35 t + 69o 32 \ )}
∴ i (t) = i (t) + i (t) = 3.713 cos ( 9t −105o 4 \ ) +
1 2
2.7622 e − t cos ( 35 t + 69o 32 \ )

Example (2- 20).


For the circuit shown in Fig. (2-58) find the currents i1 (t), i2 (t)
and i (t).

85
Chapter 2 The Natural and Forced Responses of Simple Circuit

i (t) 2 Ω

i2 (t) 0.2 H
V(0)=9 V 0.1 F i1 (t)
12 t V
1Ω

Fig. 2-58

The previous circuit is shown in the S domain as indicated in


Fig. (2-59).
2
I

9
0.2 S
12 S
2 I2 I1
S 10
S 1

Fig. 2 – 59

12
= 2 ( I + I ) + I (1+ 0.2 S ) = I ( 3 + 0.2 S ) + 2 I ……1
S 2 1 2 1 1 2
10 9
Also, I ( ) + = I (1+ 0.2 S ) …….2
2 S S 1
Solving equations 1 & 2 for I1 and I2then;
45 S2 + 300 A B CS + D
I1 = = + +
S2 ( S2 +10 S + 75 ) S2 S S2 +10 S + 75
4 0.5333 S+5
I = − + 0.5333
1 S2 S (S + 5) 2 + (7.071) 2
46.333 7.071
+ ×
7.071 (S + 5) 2 + (7.071) 2

86
Chapter 2 The Natural and Forced Responses of Simple Circuit

−5t
i1(t)=4 t – 0.5333+ e {0.5333 cos 7.071 t + 6.552 sin 7.071 t}
−5t
= 4 t + 6.574 e cos ( 7.071 t – 85o 20\ ) – 0.5333
12 45 S2 + 300
I = − (1.5 + 0.1S )
2 2 S2 S 2 ( S 2 +10 S + 75)
6 4.5 S3 + 67.5 S 2 + 30S + 450
= −
S2 S 2 ( S 2 +10 S + 75)
6 A B C S+ D
I = −{ + + }
2 S2
S 2 S S 2 +10 S + 75

6 6 0.4 4.9 S + 65.5


I = −{ − + }
2 S2 2 S 2
(S + 5 ) + (7.071) 2
S
0.4 S+ 5 41 7.071
I = − 4.9 − ×
2 S (S + 5 ) 2 + (7.071) 2 7.071 (S + 5 ) 2 + (7.071) 2
−5t −5t
i2 (t) = 0.4 – 4.9 e cos 7.071 t – 5.8 e sin 7.071 t
−5t
= 0.4 – e { 7.59 cos ( 7.071 t – 49o 47\ ) }
−5t
i (t) = i1 (t) + i2 (t) = - 0.1333 + 4 t + e { - 4.366 cos (7.071t)
– 0.752 sin (7.071t) }
−5t
= - 0.1333 + 4 t – 4.43 e cos ( 7.071 t + 9o 49\ )

87
Chapter 2 The Natural and Forced Responses of Simple Circuit

Problems

1- The initial voltage on the capacitor in the circuit shown in Fig.


(2-60) is 120 V. The initial current in the inductor is zero. The
voltage response for t ≥ 0 is;
V (t) = - 40 e – 200 t + 160 e – 800 t volt. Find;
a) R, L, α, and ωo.
b) iR (t) , iL (t) and iC (t) for t ≥ 0.

iC iR iL
V (t)
400 n f L
R

Fig. 2-60

2- For the circuit shown in Fig. (2-61) find;


a) V (t) for t ≥ 0 .
d V (t)
b) The three values of t for which = 0.
dt
c) V (t1), V (t2), and V (t3).
d) Sketch V (t) versus t for 0 ≤ t ≤ t2 .

i(0) = - 5
V (0) = 0 0.1 f 10
5Ω H
101

Fig. 2 – 61

3- For the circuit shown in Fig. (2-62) if the switches are


removed to their alternate positions at t = 0 . Find VR (t) for t ≥ 0.

88
Chapter 2 The Natural and Forced Responses of Simple Circuit

10 Ω

20 µ f 500 Ω 31.25 20
8A mA 1 kΩ
H

Fig. 2- 62

4- For the circuit shown in Fig. (2-63) find Va (t) For t ≥ 0.

100 Ω 200 Ω 600 Ω


t=0
300 V 20 µf Va (t) 5H

Fig. 2 -63

5- For the circuit shown in fig. (2- 64) find the current i (t) for
t ≥ 0.
30 Ω
10 Ω 10 Ω
i (t)
8Ω 4Ω
0.01 f
100 V
2H t=0

Fig. 2- 64

6- Find VC (t) for t ≥ 0 for the circuit shown in Fig. (2 – 65).

89
Chapter 2 The Natural and Forced Responses of Simple Circuit

1Ω
2Ω 0.5 H t=0
3Ω
t=0
0.125 f 5Ω
12 V 9V

Fig. 2- 65

7- In the circuit shown in Fig. ( 2-66 ) find;


a) VL (t) for t ≥ 0
b) t for VL (t) may be maximum and its value.
c) Repeat for R = 12 Ω.
0.5 µ f
120 Ω

600 V 5mH
t=0

Fig. 2 – 66

8- For the circuit shown in Fig. ( 2-67 ) the switch is moved


from position 1 to position 2 at t = 0. Find the current in the
resistor for t ≥ 0 .
2 1

10 Ω

50 e -5 t V 0.3 H 100 sin (314 t+60o) V

100 µ f

Fig. 2- 67

90
Chapter 2 The Natural and Forced Responses of Simple Circuit

9- For the circuit shown in Fig. (2 - 68) find the voltage across
the inductor after switch closer.

10 Ω 0.15 H

150 sin(314 t
+ 60o) V 25x10- 5 f
10 Ω

Fig. 2 -68
10- find the current i (t) which may pass through the capacitor
in the circuit illustrated in Fig. 2-69 after opening the switch
at t = 0.0.
t=0

0.3 H 100 µ F
i (t)

20 Ω 30 Ω

V1(t) = 20 e-5t V2(t) = 100 sin(314t


+ 60o)

Fig. 2-69

91
Chapter 3 The sinusoidal Steady State

CHAPTER 3

The sinusoidal Steady State

This chapter considers the steady state "Forced response"


component only. A sinusoidal voltage source produces a
voltage that varies sinusoidal with time. The sinusoidal varying
function can be expressed using either the sine function or the
cosine function. Fig. (3-1) shows a cosine function.
V(t)
Vm

-Vm
Fig. 3-1

V (t) = Vm cos ω t, where ω = 2 π f


f is the number of cycles per second and named " frequency".
1
f = where T is the time period.
T

The sinusoidal response:-


Let Vs (t) = Vm cos (ω t + ϕ)
For the R-L circuit shown in Fig. (3-2) , the voltage law
d i (t)
equation is; R i (t) + L = Vm cos (ω t +  )
dt
The supply voltage may be rewrite in exponential form as
follows;
J ( ω t + )
Vs (t) = R Vm e

92
Chapter 3 The sinusoidal Steady State

i (t) R
Vs (t) L

Fig. 3 - 2

If i (t) = I e S t , then S = J ω So,


d i (t)
L = L × S × I × e S t = L × J ω × i (t)
dt
= J ω L i (t)
R i (t) + J ω L i (t) = Vm cos (ω t +  )
Vm
So, i (t) = cos ( ω t +  − θ )
2
R +(ωL) 2
ωL
Where θ = tan – 1
R

Power due to a sinusoidal varying current:-


For the circuit shown in Fig. (3-3) with Vs (t) = Vm cos ω t the
current
V (t)
i (t) = s = Im cos ω t
R
R

Vs (t) i (t)

Fig. 3 – 3

cos 2 ω t +1
P (t) = { i (t) }2 R = I 2m R cos2 ω t = I 2m R { }
2
Let ωt = θ then the mean value of the power may be;

93
Chapter 3 The sinusoidal Steady State

1 2π 2 cos 2 θ +1 I 2m R 2 π
( cos 2 θ +1) d θ
2π 0∫ m 2 x 2 π 0∫
Pmean = I R dθ =
2
I 2m R sin 2 θ 2 π I 2m R I
= [ +θ] = [ 0 + 2 π ]= ( m )2 R
4π 2 0 4π 2
= ( Ir. m. s.)2 R
Where Ir. m. s. is the root mean square value of the current i (t)
and is defined as ;
It is the value of a direct current which has the same capacity
for developing power in a resistor as the alternating current.
{ Vr. m.s.}2
Also P (t) =
R
V
Where Vr. m. s. = m
2
So, the alternating voltage and current are always referred to
their r. m. s. values, unless if it's other wise stated.
150 J ω t
Thus if V (t) = 150 cos ω t = R e
2
Example (3-1).
Find the r. m. s. value of the periodic voltage shown in
Fig. (3 – 4).
V (t)

100 V 60 90 100
-100 V 10 40 t

Fig. 3- 4

100
V (t) = t =10 t for 0 ≤ t ≤ 10
10
V (t) = 100 for 10 ≤ t ≤ 40

94
Chapter 3 The sinusoidal Steady State

200
V (t) = 100 − (t − 40 ) = 500 −10 t for 40 ≤ t ≤ 60
20
V (t) = - 100 for 60 ≤ t ≤ 90
100
V (t) = -100 + (t − 90 ) = −1000 +10 t for 90 ≤ t ≤ 100
10
1 10 40
Vr. m.s. = [ { ∫ 100 t 2d t + ∫ 10 4 d t
100 0 10
60
+ ∫ ( 25×10 4 −10 4 t +100 t 2 ) d t +
40
1
90 4 100 6 4 2 2
∫ 10 d t + ∫ (10 − 2 ×10 t +100 t ) d t }]
60 90
1 100
=[ { ×103 + 10 4 (40 − 10) + 25 ×10 4 (60 − 40)
100 3
10 4 100 3
− (60 2 − 40 2 ) + (60 − 403 ) + 10 4 (90 − 60)
2 3
1
6 2 ×10 4 2 2 100 3 3
+ 10 (100 − 90) − (100 − 90 ) + (100 − 90 )}] 2
2 3
= 85.6 V

The Phasor diagram:-


The complex number 1.464 + J 3.66 may be converted to a
phasor values as shown;
3.66
(1.464 ) 2 + ( 3.66 ) 2 ∠ tan- 1 = 3.94 ∠ 68o 12\
1.464
These phasor values are shown on a phasor diagram as shown
in Fig. (3- 5).

95
Chapter 3 The sinusoidal Steady State

Imaginary axis
3.66
3.94

o \ Real axis
1.464 68 12
Fig. 3 – 5

Addition and subtraction of phasors.


Let a vector P1 = a + J b and another vector P2 = c + J d then,
b+d
P1+ P2 = (a + c) + J (b + d) = ( a + c) 2 + (b + d) 2 ∠tan-1
a+c
P1 – P2 = P1 + (- P2) = = (a - c) + J (b - d)
b−d
= ( a − c) 2 + (b − d) 2 ∠tan-1
a −c
Fig. (3 - 6) shows the vector diagrams for P1+ P2 and P1- P2 . In
the figure Y is the imaginary axis and X is the real axis.

Y Y
P1+P2
P1 P1-P2
P1
P2
X X
-P2
Fig. 3- 6

Example (3-2).
Let V1 (t) = 30 cos ( ω t + 90) , V2 (t) = 40 sin ( ω t + 90) and
V3 (t) = 50 cos ( ω t + 216.87), Determine their sum following;
a) Calculation in the time domain.
b) Calculation by the phasor method.
c) Calculation by phasor diagram.

96
Chapter 3 The sinusoidal Steady State

30
a- V1 (t) = 30 cos ( ω t + 90) = ∠ 90o
2
40
V2 (t) = 40 sin ( ω t + 90) = 40 cos ω t = ∠ 0o
2
V3 (t) = 50 cos ( ω t + 216.87) = 50 cos ( ω t - 143.13)
50
= ∠- 143.13o
2
V (t) = V1 (t) + V2 (t) + V3 (t) = 30 cos ( ω t + 90) + 40 cos ω t
+ 50 cos ( ω t - 143.13)
= - 30 sin ω t + 40 cos ω t + 50 cos ω t cos 143.13
+ 50 sin ω t sin 143.13 = 0

30 40 50
b- V (t) = ∠ 90o + ∠ 0o + ∠- 143.13o
2 2 2
= 21.213∠ 90 + 28.284 ∠ 0 + 35.355 ∠- 143.13o
o o

= J 21.213 + 28.284 – 28.284 – J 21.213 = 0

c- Fig. (3 -7) shows the vector diagram.

V1(t)
{V1(t) + V2 (t) + V3 (t)}

V2(t) X

V3(t)

Fig. 3- 7

97
Chapter 3 The sinusoidal Steady State

Phasor relationships between voltage and current in the


circuit elements:-

a) Resistance case.

Ohm's law gives the relation between the current and the
voltage as follows;
Let i (t) = Im cos (ω t + ϕ)
∴V (t) = R × Im cos (ω t + ϕ)
R x Im
V = R ( R × Im e J ϕ ) = ∠
2

Fig. (3-8) shows this relation.

V (t) V
I i (t)
ϕ
i (t) t
V (t)
Fig 3 – 8

b) Inductance case.
Let i (t) = Im cos ( ω t + ϕ )
d i (t)
∴V (t) = L = − ω L I m sin (ω t +  )
dt
= − ω L I m cos (ω t +  − 90 ) = ω L I m cos (ω t +  + 90 )
I ω L Im
= R ( ω L e J 90 × m e J ϕ ) = ∠ + 90
2 2
V (t) = ω L ∠ 90o × I r. m.s. ∠
Fig. (3 - 9) Shows this relation.

98
Chapter 3 The sinusoidal Steady State

V I i (t)
ϕ t i L
(t)
V
(t)
V (t)
Fig. 3 – 9
The term ω L is named inductive reactance “XL” while Jω L is
named inductive impedance “ZL” and is measured by ohms.

c) Capacitance case.
Assume V (t) = Vm cos (ω t + ϕ)
d V (t)
∴ i (t) = C = − ω C Vm sin (ω t +  )
dt
= − ω C Vm cos (ω t +  − 90 ) = ω C Vm cos (ω t +  + 90 )
V
I = R ( ω C e J 90 × m e J ϕ ) = ω C Vr. m.s. ∠ ( + 90) o
2
Fig. (3 – 10) shows this relation.

I V
C
ϕ t i
(t)
V
i (t) V (t) (t)

Fig. 3 - 10
1 1
The term is named capacitive reactance “X C” while
ωC J ωC
is named capacitive impedance “Z C” and is measured by ohms.

Example (3-3).
The current in a 75 mH inductor is 4 cos (40000 t – 38) mA
calculates;

99
Chapter 3 The sinusoidal Steady State

a) The inductive reactance XL.


b) The impedance of the reactor.
c) The phasor voltage.
d) The steady state expression for V (t).

a- The inductive reactance of the inductor XL is the ohmic


value of the inductor and it's equal's;
XL = ω L = 40000 ×75 × 10 – 3 = 3000 ohm.

b- The impedance of the reactor ZL is the ohmic value in the


complex plane.
ZL = J XL = J 3000 ohm.

4 x 10 − 3
c- V = I × ZL = ∠ − 38 × 3000 ∠ 90
2
= 8.485∠ 52 o V

d- V (t) = 2 V cos ( ω t + 52 ) =12 cos (40000 t + 52) V

Example (3- 4).


The voltage across the terminals of a 0.2 µ f capacitor is
40 cos (105 t – 50) volt, calculate;
a) The capacitive reactance XC .
b) The impedance of the capacitor ZC.
c) The phasor current I.
d) The steady state current i (t).

a- The capacitive reactance of the capacitor XC is the ohmic


value of the capacitor and it's equal's;
1 106
XC = = = 50 ohm
ω C 105 x 0.2

b- The impedance of the capacitor ZC is the ohmic value in the


complex plane.

100
Chapter 3 The sinusoidal Steady State

Z C = - J X C = - J 50 ohm.

40
∠ − 50o
V
c- I = = 2 o = 0.565 ∠ 40o A
C 50 ∠ − 90
Z

d- i (t) = 2 × 0.565 cos ( 105 t + 40 ) = 0.8 cos (105 t + 40 ) A

Kirchhoff's Laws in The Phasor Domain

a ) Kirchhoff's voltage law in the phasor domain:-


Let v1(t), v2(t), …….. vn(t) represent voltages around a closed
path in a circuit. Assuming the circuit is operating in a
sinusoidal steady state the kirchhoff's voltage law gives;
v1(t) + v2(t) + ………..+ vn(t) = 0
Vm1 cos ( ω t + θ1 ) + Vm2 cos ( ω t + θ2 ) + ………
+ Vmn cos ( ω t + θn ) = 0

R { Vm1 e J ω t e 1 } + R { Vm2 e J ω t e J θ 2 } + ……………

+ R { Vmn e J ω t e Jθn
}= 0

R [ { Vm1 e 1 + Vm2 e J θ 2 + ……+ Vmn e J θ n }eJ ω t ] = 0
Jωt
e ≠ 0 , Then V1 + V2 + ………..+ Vn = 0.
Where V1, V2 ………..Vn are the phasor representations of the
individual voltage v1(t), v2(t), ……..vn(t).

b ) Kirchhoff's current law in the phasor domain:-


Let i1(t), i2(t), …….. in(t) represent currents entering a node in
a circuit. Assuming the circuit is operating in a sinusoidal
steady state the Kirchhoff's current law gives;
i1(t) + i2(t) + ………..+ in(t) = 0

Im1 cos ( ω t + θ1 ) + Im2 cos ( ω t + θ2 ) + ………


+ Imn cos ( ω t + θn ) = 0

R { Im1 e J ω t e 1 } + R { Im2 e J ω t e J θ 2 } + ……………

101
Chapter 3 The sinusoidal Steady State

+ R { Imn e J ω t e J θ n }= 0

R [ { Im1 e 1 + Im2 e J θ 2 + ……+ Imn e J θ n }eJ ω t ] = 0
Jωt
e ≠ 0 , Then I1 + I2 + ………..+ In = 0.
Where I1, I2 ………..In are the phasor representations of the
individual currents i1(t), i2(t), ……..in(t).

Sinusoidal Steady State Power Calculation

Let V (t) = Vm cos ( ω t + θv ) & i (t) = Im cos ( ω t + θi )


P (t) = V (t) ∗ i (t) = Vm Im cos ( ω t + θv ) cos ( ω t + θi )
Taking the current as a reference, this need to shift both the
voltage and the current by θi .
∴ V (t) = Vm cos ( ω t + θv - θi ) & i (t) = Im cos ( ω t )
P (t) = Vm Im cos ( ω t + θv - θi ) cos ( ω t )
cos ( 2 ωt + θ v − θ ) + cos ( θ v − θ )
= Vm I m i i
2
Vm I m
= {cos (2 ω t) cos ( θ v − θ ) − sin (2 ω t)sin (θ v − θ )
2 i i
+ cos (θ v − θ )}
i
1 T
Pav = ∫ P (t) d t
T0
V I 2π
= m m ∫ {cos ( 2δ ) cos( θ v − θ ) − sin ( 2δ ) sin (θ v − θ )
4π 0 i i
+ cos ( θ v − θ ) }dδ
i
Where the angle δ = ω t
V I
∴Pav = m m cos ( θ v − θ )
2 i
If the circuit is purely resistive then θi = θv
V I
∴P (t) = m m {cos (2 ω t) +1}
2

102
Chapter 3 The sinusoidal Steady State

1T
T 0∫
Pav= P (t) d t

V I 2π V I
Pav = m m ∫ {cos ( 2δ ) + 1}dδ = m m = Vr. m.s. ∗ I r. m.s.
4π 0 2
If the circuit is purely reactive then the voltage and the current
will be out of phase by 90o i. e. θi = θv ± 90o
V I
∴ P (t) = ± m m sin (2 ω t) - ve for inductive circuit while +
2
ve for capacitive.
V I 2π
Pav = m m ∫ sin ( 2δ ) dδ = 0
4π 0
Power associated with purely reactive circuit is refereed to as
reactive power Q.
V I V I
Q = m m sin (θ v - θ ) = m m VAR
2 i 2
V I V I
So P = m m cos (θ v - θ ) & Q = m m sin (θ v - θ )
2 i 2 i
cos (θ v - θ ) is named the power factor { P. F. } &
i
sin (θ v - θ ) is named the reactive factor { R. F.}
i

Complex Power

The product V∗ I is called the apparent power and is indicated


by the symbol S. The S unit is the volt ampere (VA), and
S = P ± J Q where;
S is the complex power, P is the average real or active power
and Q is the reactive power.
Let V = V1 + J V2 & I = I1 + J I2
Fig. (3-11) shows both V and I on the complex plane. From this
figure it's possible to have;

103
Chapter 3 The sinusoidal Steady State

Y
V2 V
I2 I

V1 I1 X
Fig. 3 – 11

P = V1 I1 + V2 I2 and Q = V2 I1 – V1 I2
Where V2 and I1 simulates an inductive case while V1 and I2
simulates an capacitive case.
 S = V∗ I = ( V1 + J V2 ) ∗ ( I1 + J I2 )
S = V1 I1 – V2 I2 + J ( V2 I1 + V1 I2 )
The previous equation is a wrong equation. To have the correct
equation then;
S = V ∗ I∗
Where I ∗ is the conjugate value of I.
So S = ( V1 + J V2 ) ∗ ( I1 - J I2 )
= V1 I1 + V2 I2 + J ( V2 I1 - V1 I2 )

Example ( 3-5).
If V (t) = 250 cos ( ω t + 45 ) & i (t) = 12 cos ( ω t – 15 )
Find the active and the reactive powers.
250 ×12
P = Vr. m. s. ∗ Ir. m. s. cos ( θv - θI ) = cos ( 45 +15 ) = 750 W
2
250 ×12
Q = Vr. m. s. ∗ Ir. m. s. sin ( θv - θI ) = sin ( 45 +15 )
2
=1299.04 VAR

Example (3- 6).


If the voltage source with V = 100 ∠ 30o is applied to a load
with impedance Z = 3 + J 4 . Determine P and Q.

Z = 3 + J 4 = 5 ∠ 53.13o

104
Chapter 3 The sinusoidal Steady State

V 100 ∠ 30o
I= = o = 20 ∠ − 23.13o
Z 5 ∠ 53.13
S = V ∗ I∗ =100 ∠ 30 × 20∠ 23.13 = 2000 ∠ 53.13
= 1200 + J 1600
So P =1200 W, and Q = 1600 VAR

Series, Parallel and Delta To Star Simplifications

a) Series combination:-
Impedance’s in series can be combined into single impedance
by adding the individual impedance’s (applying Kirchhoff’s
voltage law). Fig (3-12) shows a series circuit from which its
possible to have;

i (t)
Z1 Z2
V (t)

Z3

Fig 3-12

V (t) = Z1 × i (t) + Z2 × i (t) + Z3 × i (t) = (Z1+ Z2 + Z3) i (t)


= Zeq i (t)
V (t)
∴Z eq = =Z +Z +Z
i (t) 1 2 3

Example (3-7).
For the circuit shown in Fig. (3-13) determine the following;
a) Construct equivalent circuit in the phasor domain.
b) Calculate the steady state current I by the phasor method.

105
Chapter 3 The sinusoidal Steady State

32 mH 5µ f
90Ω

750 cos (5000 t + 30o) V

Fig. 3-13
a- ZL = J ω L = J 5000 × 32 × 10 – 3 = J 160 Ω
− J − J10 6
ZC = = = − J 40 Ω
ω C 5000 x 5
750
V= ∠ 30 = 530.13 ∠ 30o V
2
Fig. (3-14) shows the circuit in the phasor domain.

90Ω J 160 Ω - J 40 Ω

530.33 ∠ 30o V

Fig. 3-14

b- Zeq = 90 + J 160 – J 40 = 90 + J 120 = 150 ∠ 53.13o V


V 530.33 ∠ 30
I= = = 3.535 ∠ − 23.13o A
Zeq 150 ∠ 53.13
i (t) = 5 cos (5000 t – 23.13) A

b) Parallel combination:-
Impedance’s in parallel circuit can be combined into single
impedance by adding the individual admittance’s ( Y ), which is
1
the inverse of the impedance’s (Y= ), by applying Kirchhoff’s
Z
current law. Fig (3-15) shows a parallel circuit by applying
Kirchhoff’s current law on it then;

106
Chapter 3 The sinusoidal Steady State

I
I1 I2 In
Z1 Z2 Zn

Fig. 3- 15

I = I1 + I2 + ………..In
V V V V
∴ = + + ...............
Zeq Z Z Zn
1 2
1 1 1 1
= + + ...............
Zeq Z Z Zn
1 2
So Yeq = Y1 + Y2 + ………Yn Siemens.
Let Y = G + J B where G is named conductance and B is the
susceptance.
Then, Geq+J Beq= G1+ G2+….Gn + J (B1+B2.…Bn) Siemens
As a special case if there are two impedances only connected in
parallel then;
1 1 1 Z +Z Z Z
= + = 1 2 ∴ Zeq = 1 2
Zeq Z Z Z Z Z +Z
1 2 1 2 1 2

Example (3-8).
For the circuit shown in Fig. (3-16) find the following;
a) The equivalent admittance.
b) The equivalent conductance.
c) The equivalent susceptance.
d) The current iC (t).
1 1 1
a- Yeq = Y1 + Y2 + Y3 = + −
3 + J 4 16 − J12 J 4
= 0.2 ∠- 53.13 + 0.05 ∠ 36.87o + 0.25 ∠ 90o
o

Yeq = 0.12 – J 0.16 + 0.04 + J 0.03 + J 0.25 = 0.16 + J 0.12


= 0.2 ∠ 36.87o

107
Chapter 3 The sinusoidal Steady State

iC (t) 3
8 cos ω t 16
A -J4
- J 12 J4

Fig. 3 – 16
b- Geq = 0.16 Siemens

c- B = 0.12 Siemens
8
I 2
d- V = = = 28.284 ∠ − 36.87 o V
Y 0.2 ∠ 36.87
∴ V (t) = 28.284 × 2 cos (ω t – 36.87) = 40 cos (ω t – 36.87)
IC = V×YC = 28.284 ∠ − 36.87 o × 0.25∠ 90o
= 7.071 ∠53.13o A
iC (t) = 7.071 × 2 cos (ω t + 53.13) = 10 cos (ω t + 53.13) A

c) Delta to Star Transformation and vice versa

I) Delta to star transformation:-


For the circuit shown in Fig. (3-17) it’s possible to have;
A

ZB Z1 ZC

Z3 Z2
C B
ZA
Fig. 3 – 17

108
Chapter 3 The sinusoidal Steady State

The equivalent impedance between the terminals AB is;


Z (Z +Z )
ZAB = C A B =Z +Z
Z +Z +Z 1 2
A B C
Also the equivalent impedance between the terminals AC is;
Z (Z +Z )
ZAC = B A C =Z +Z
Z +Z +Z 1 3
A B C
And the equivalent impedance between the terminals BC is;
Z (Z +Z )
ZBC = A B C =Z +Z
Z +Z +Z 2 3
A B C
By adding the two first equations and subtracting the third then;
ZAB + ZAC - ZBC =
Z (Z + Z ) + Z (Z + Z ) − Z (Z + Z )
C A B B A C A B C
Z +Z +Z
A B C

= Z1 + Z2 + Z1 + Z3 - Z2 - Z3
2Z Z
So, C B = 2 Z1
Z +Z +Z
A B C
Z Z Z Z
∴Z1 = C B Similarly Z2 = A C And
Z +Z +Z Z +Z +Z
A B C A B C
Z Z
Z3 = A B
Z +Z +Z
A B C

II) Star to delta transformation:-


By multiplying the impedance value from the star connection to
the other one then;
Z Z2 Z Z2 Z Z
A C B A C B
Z1 × Z2 = , Z2 × Z3 =
(Z + Z + Z ) 2 (Z + Z + Z ) 2
A B C A B C

109
Chapter 3 The sinusoidal Steady State

Z Z Z2
A C B
And Z1 × Z3 =
(Z + Z + Z )2
A B C
By adding the three previous equations then;
Z Z2 Z + Z2 Z Z + Z Z Z2
A C B A C B A C B
Z1×Z2+Z2×Z3+Z1×Z3 =
(Z + Z + Z ) 2
A B C
Z Z Z (Z + Z + Z ) Z Z Z
A C B C A B A C B
= =
(Z + Z + Z ) 2 (Z + Z + Z )
A B C A B C
= ZA Z1 = ZB Z2 = ZC Z3
Z Z Z Z
So ZA = Z2 + Z3 + 2 3 , ZB = Z1 + Z3 + 1 3
Z Z
1 2
Z Z
And ZC = Z1 + Z2 + 1 2
Z
3
Example (3-9).
For the circuit shown in Fig. (3-18) using ∆ - Υ transformation
find; Io, I1, I2, I3, I4, I5, V1 and V2.

Io
I1 63.2
-J 4
I2

I3 10 J 2.4
120 ∠0 V

20
V1 I4 -J 20 V2
J 60 I5

Fig. 3 – 18

110
Chapter 3 The sinusoidal Steady State

Transforming one from the two deltas which the circuit of Fig.
(3–18) combine them into star. Fig. (3-19) shows this process
then;

Io
I1 63.2
-J 4
I2
10 J 2.4
120 ∠0 V

20 Z1

Z2 -J 20
J 60 Z3

Fig. 3 – 19
10 ( 20 + J 60) 10 ( − J 20)
Z1 = = 12 + J 4, Z2 = = - 3.2 - J 2.4
30 + J 60 − J 20 30 + J 40
- J20 ( 20 + J 60)
Z3 = = 8 - J 24
30 + J 60 − J 20
Fig. ( 3-20) shows the circuit after the transformation.

Io 63.2
-J 4

12 J 2.4
I2
120 ∠0 V I1 -3.2
J4 V2
V1 -J 2.4

-J 24

Fig. 3 - 20

111
Chapter 3 The sinusoidal Steady State

From the circuit shown in Fig. (3- 20) it's possible to have Zeq
as follows;
(- J4 +12 + J4 )( 63.2 + J 2.4 - 3.2 - J 2.4)
Zeq = 8 – J 24 +
12 + 60
12 × 60
Zeq = 8 – J 24 + = 18 – J 24 = 30 ∠ - 53.13o ohm
12 + 60
V 120
Io = = = 4 ∠ 53.13o = 2.4 + J 3.2 A
Zeq 30 ∠ − 53.13
60
I1 = 4 ∠ 53.13 × = 3.333 ∠ 53.13 o = 2 + J 2.666 A
72
12
I2 = 4 ∠ 53.13 × = 0.667 ∠ 53.13 o = 0.4 + J 0.534 A
72
V1 = 120 – 3.333 ∠ 53.13 × 4 ∠ - 90 = 120 – 13.332 ∠ - 36.87
= 109.336 + J 8 = 109.626 ∠ 4.184 o V
V2 =120 – 0.667 ∠53.13 × 63.246 ∠2.175
=120– 42.185 ∠55.305 = 96 - J 34.684
= 102.062 ∠ - 19.867 o V
After calculating the voltages V1 and V2, returning to the
circuit which had been shown in Fig. (3 - 17) for calculating the
currents I3, I4 and I5 so;
V
1 109.626 ∠4.184
I4 = = =1.733 ∠ − 67.381
20 + J 60 63.245 ∠71.565
= 0.666 – J1.6 A
2 = 102.062 ∠ −19.867 = 5.103 ∠70.133
V
I5 =
- J 20 20 ∠ - 90
= 1.734 + J 4.8 A
Then, I3 = I1 – I4 = 2 + J 2.666 – 0.666 + J 1.6
= 1.334 + J 4.266A

112
Chapter 3 The sinusoidal Steady State

Problems

1- A voltage whose waveform is triangle as shown in Fig. (3-


21) having a peak value of 240 V is applied to a 100 ohm
resistor, calculate;
a) The peak value of the power.
b) The mean value of the power.

V (t)
240

1 2 t sec
Fig. 3 – 21

2- A 40 Ω resistor and a 10 µ f capacitor are connected in series


across a voltage source. A current with r. m. s. value = 2 A and
leads the voltage by 40 o is passed through the circuit determine,
the frequency f and the potential difference at the source
terminals.

3- Two coils are connected in parallel. When a constant voltage


source is applied on them, the currents in the coils are 2 A and 3
A. With 50 V at 35 Hz applied to these coils the corresponding
currents are 1 A and 1.6 A. Calculate the total current, power
and power factor when the coils are connected in series across
240 V, 50 Hz. Supply.

4- Convert the following phasors to time domain expression;


a- 15 ∠ 53 o with ω = 5000. b- 3 – J 4 , with ω = 280.
− J 30 − 1+ J 2
c- 20 e , with ω = 50. d- , with ω = 60.
−1 − J 2

113
Chapter 3 The sinusoidal Steady State

5- For the circuit shown in Fig. ( 3-22) find the impedance


between the terminals A and B.
- J 20 Ω
100 Ω J 50 Ω

A B
50 Ω J 100 Ω 60 Ω
80 Ω
- J 60 Ω
- J 50 Ω

Fig. 3 - 22

6- For the circuit shown in Fig. (3 – 23) calculate;


a) The total conductance.
b) The total susceptance.
c) The admittance for each branch and for the whole circuit.
d) The total current, Power, and power factor for the whole
circuit.
- J 10 Ω

10 Ω 15 Ω
400 V
J 20 Ω J 15 Ω

Fig. 3 – 23

7- A parallel circuit with one branch of R = 5 ohm and a single


unknown element in the other branch has the following applied
voltage and total current;
V (t) = 10 cos (50t + 60o ) V,
i (t) = 5.38 cos (50t – 8.23o ) A. Find the unknown element.

114
Chapter 3 The sinusoidal Steady State

8- In the parallel circuit shown in Fig. (3 -24) the applied


voltage V (t) = 100 cos (5000t + 45o) volt find;
a) The total current it (t).
b) What two elements in a series connection would result in
the same current and would therefore be equivalent to the
parallel circuit for the same frequency.
it (t)

5Ω 3Ω
V (t)
1.73 mH 50µf

Fig. 3 – 24

9- In a series of R = 5 ohms and L = 0.06 henrys the voltage


across the inductance is VL (t) = 15 sin 200t. Find the
total voltage, the current, the phase shift between the
current and the voltage and the magnitude of the
impedance.

10- An RLC series circuit has a current which lags the


applied voltage by 30o. The voltage across the
inductance has a maximum value which is twice the
maximum value of the voltage across the capacitor and
VL (t) = 10 sin 1000 t V.
If R = 20 ohms, determine the values of L and C.

115
Chapter 3 The sinusoidal Steady State

Parallel and Series Resonance

The previous part from this chapter is concerned with finding


the steady state currents and voltages with constant frequency
power supplies. But the following section may concern with the
frequency response of the circuits which contain a power
supplies with variable frequency.

a) Parallel Resonance:-
For the circuit shown in Fig. (3 - 25), for supply with very low
frequencies, the inductive reactance will be very small. Thus
the output voltage will be very small. But for supply at very
high frequency, the capacitive reactance will be very small,
therefore the output voltage will be very small.

Im cos ω t C R L Vom cos(ω t+θ)

Fig. 3 – 25
The output voltage;
I I I
Vo = S = S = S
Y 1 + 1 + J ωC 1
+ J (ω C −
1
)
R JωL R ωL
1
I ∠0 ωC −
where = tan −1
S ωL
Vo =
1 1 2 1
( )2 + ( ω C − ) ∠ R
R ωL
I
Vo = S
1 1 2
( )2 + ( ω C − )
R ωL
Where IS is the current source magnitude.
So, Vo (t) = Vo m cos (ω t - ϕ) = Vo m cos (ω t + θ )

116
Chapter 3 The sinusoidal Steady State

With IS , R, L, and C are fixed the previous equation tells us


how the amplitude and phase of Vo (t) changes as ω changes.
Fig. (3-26) shows how the output voltage Vo (t) changes as ω
changes.
Vo

Vom = Im R

ω1 ωo ω2 ω
Fig. 3 – 26

At resonance case YC = YL (i. e. the total admittance is


conductance).
1 1
So, ωo C = then ωo =
ωL LC

Band width and quality factor:-


The band width is the range of frequencies in which the
amplitude of the output voltage is equal the r. m. s. value
Vo m
( i. e. Vo = ).
2
Vo m 1 1 2 2
For Vo = then, ( ) 2 + ( ω C − ) =
2 R ωL R2
1 1 2
∴ ( )2 = ( ω C − )
R ωL
So ω may have four values two from them are rejected while
the others may be;
1 1
= ± (ωC− )
R ωL

117
Chapter 3 The sinusoidal Steady State

ω 1 ω 1
∴ω 2 + − =0 Or ω 2 − − =0
R C LC R C LC
1 1 2 1
Then ω = − ± ( ) +
2R C 2R C LC
1 1 2 1
Or ω = ± ( ) +
2R C 2R C LC
The negative values of the previous equations are rejected then;
1 1 2 1
ω =− + ( ) +
1 2R C 2R C LC
1 1 2 1
And ω = + ( ) +
2 2R C 2R C LC
The band width β is the difference between ω2 and ω1 so;
1
β = ω 2 - ω1 =
RC
Thus the change in L may control the value of ωo while the
change in R may control the width of β ( i. e. For large values
from R gives small β “narrow band with “ and vice versa ).
The quality factor of the frequency response Q is described
quantitatively as the ratio of the resonance frequency ωo to the
band width β .
ω R 1 C
Q = o = ωo R C = = ×R C= R×
β ωo L LC L
Example (3-10).
For the circuit shown in Fig. ( 3- 27) Find;
a) Q, ωo, ω1 and ω2.
b) At what value of R the band width β = 500 Rad./ sec.
c) Find the magnitude of Q for the case that discussed in b

50 mA 0.25 µ f 2kΩ 40 mH

Fig. 3 – 27

118
Chapter 3 The sinusoidal Steady State

1 109
a- ωo = = =10 4 rad. / sec
LC 0.25× 40
Q = ωo R C = 104 × 2 × 103 × 0.25 × 10 – 6 = 5
1 1 2 1
ω =− + ( ) +
1 2R C 2R C LC
= - 1000 + 106 +108 = 9049.88 Rad. / sec.
1 1 2 1
ω = + ( ) +
2 2R C 2R C LC
= 1000 + 106 +108 = 11049.88 Rad. / sec.

1 1 10 6
b-  β = , So R = = = 8000 Ω
RC β C 500 × 0.25

ωo 10 4
c- Q = = = 20
β 500

Describing the circuit behavior in terms of ωo , β and Q

1 1 ωo
β = & ωo = and Q =
RC LC β
1 1 2 1 β β
ω =− + ( ) + = − + ( ) 2 + ωo2
1 2R C 2R C LC 2 2
ωo ω 1 1 2
ω1 = - + ( o ) 2 + ωo2 = ωo { − + ( ) +1 }
2Q 2Q 2Q 2Q
1 1 2
Also ω = ωo { + ( ) +1 }
2 2Q 2Q
1 2 1 β
For 1 >> ( ) Then ω = ωo {1− } = ωo −
2Q 1 2Q 2

119
Chapter 3 The sinusoidal Steady State

1 β
Also ω = ωo {1+ } = ωo +
2 2Q 2
The total admittance Y for the parallel circuit may be;
1 1 1 R
Y = + J (ωC− ) = {1+ J ( ω C R − )}
R ωL R ωL
1 ω ω C R ωo R 1 ω ωo
= {1+ J ( o − ) } = {1+ J Q ( − )}
R ωo ωo ω L R ωo ω
1 ωo ω 2 − ωo2 1 ( ω − ωo ) ( ω + ωo )
= {1+ J ( )} = {1+ J }
R β ωωo R βω
As ω → ωo then, the admittance value in approximate form
1 2 ( ω − ωo )
may be;Y ≅ {1+ J }
R β

Example (3-11).
For the circuit shown in Fig. (3- 28) find;
a) Vo at ω = 10500 Rad/sec. and at 9500 Rad/sec. following
the exact formula for Y.
b) Repeat the previous following the approximate
expression for Y.
c) Find the error percentage in the magnitude of Vo due to
following the approximate expression.

50 mA 0.25 µ f Vo 2kΩ 40 mH

Fig. 3 – 28
a- Following the exact formula;
1 ( ω − ωo ) ( ω + ωo )
Y = {1+ J }
R βω
1 109
ωo = = =10 4 Rad. / sec
LC 0.25× 40

120
Chapter 3 The sinusoidal Steady State

106
β= = 2000
3
2 × 10 × 0.25
For ω = 10500 Rad. / sec.
1 500 × 20500
Y= {1+ J } = 0.5564 × 10 - 3 ∠ 26.02 o mho
2000 2000 ×10500
I 50 ×10 − 3
Vo = = = 89.87 ∠ − 26.02 o V
Y 0.5564 ×10 − 3 ∠ 26.02
At ω = 9500 Rad. / sec.
1 - 500 × 19500
Y= {1+ J } = 0.562 × 10 - 3 ∠-27.16 o mho
2000 2000 × 9500
I 50 ×10 − 3
Vo = = = 88.97 ∠ 27.16 o V
Y 0.562 ×10 − 3 ∠ − 27.16

b- Following the approximate expression;


1 2 ( ω − ωo )
Y = {1+ J }
R β
For ω = 10500 Rad. / sec.
1 2 × 500
Y= {1+ J } = 0.559 × 10 – 3 ∠ 26.57 o mho
2000 2000
I 50 ×10 − 3
Vo = = = 89.44 ∠ − 26.57 o V
Y 0.559 ×10 − 3 ∠ 26.57
At ω = 9500 Rad. / sec.
1 2 × − 500
Y= {1+ J } = 0.559 × 10 - 3 ∠ − 26.57 o mho
2000 2000
I 50 x 10 − 3
Vo = = = 89.44 ∠ 26.57 o V
Y 0.559 x 10 − 3 ∠ − 26.57

c- The error due to due to following the approximate


expression may be;

121
Chapter 3 The sinusoidal Steady State

For ω = 10500 Rad. / sec.


89.87 − 89.44
The error percentage = ×100 = 0.48 %
89.87
For ω = 9500 Rad. / sec.
88.97 − 89.44
The error percentage = ×100 = − 0.53 %
88.97

Effect of frequency variation on the phase shift:-


I
Vo = = Vo ∠ θ where θ = - ϕ
Y ∠
( ω − ωo ) ( ω + ωo )
  = tan – 1
βω
− ωo2
From the previous equation at ω = 0  = tan –1
=-∞
0
 = - 90 o , then θ = 90 o
And as ω → ∞  = tan – 1 ∞ = 90 o hence θ = - 90 o.
At ω = ωo  = tan – 1 0 = 0 θ = 0
( ω − ωo ) ( ω + ωo )
At ω = ω1  = tan – 1 1 1
βω
1
β
As ω1 tends to ωo then; ω1 + ωo = 2 ω1 and ω1 - ωo = -
2
β
(- )( 2ω )
1
 = tan –1 2 = - 45 o
βω
1
β
Also as ω2 tends to ωo then; ω2 + ωo = 2 ω2 and ω2 - ωo =
2
β
( )( 2ω )
2
 = tan – 1 2 = 45 o
βω
2
Fig. (3 – 29) shows the θ variation against the frequency
response.

122
Chapter 3 The sinusoidal Steady State

θ
90
45
ω1 ωo ω2 ω
-45
-90
Fig. 3 – 29

b) Series Resonance:-

For the circuit shown in Fig. (3- 30) the amplitude of the
current approach zero at both very small and very high
values of ω.
The amplitude will be at peak at V when XL = XC.
R
R L C

Vm cos ω t Im cos (ω t + θ )

Fig. 3 – 30

1 1
So ωo L = , then ωo =
ωo C LC
V ∠0
I ∠θ =
Z ∠
1
Where Z = R + J ( ω L − )
ωC

123
Chapter 3 The sinusoidal Steady State

1
ωL−
) ∠ tan −1
1 2 ωC
Z = (R) 2 + (ω L −
ωC R
1
ωL−
∠ − tan −1
Vm ωC
Im =
1 2 R
(R) 2 + (ω L − )
ωC
Fig. (3-31) shows a free sketch for the current amplitude as a
function of ω.
I
V
V R
2R

ω1 ωo ω2 ω
Fig. 3 – 31
The bandwidth of the series circuit is defined as the range of
frequencies in which the amplitude of the current is equal to
1 V
times the maximum amplitude which is .
2 R
Im V V
= =
2 2R 1 2
(R) 2 + (ω L − )
ωC
1 2
∴ 2 R = (R) 2 + (ω L − )
ωC
1 2 1
R2 = ( ω L − ) So, R = ± ( ω L − )
ωC ωC
R 1 R R 1
Hence ω 2 − ω− = 0 , So ω = ± ( ) 2 +
L LC 2L 2L LC

124
Chapter 3 The sinusoidal Steady State

R 1 R R 1
Or ω 2 + ω− = 0 , Then ω = − ± ( )2 +
L LC 2L 2L LC
The negative sign in the two previous equations are rejected
so;
R R 1 R R 1
ω =− + ( )2 + and ω = + ( ) 2 +
1 2L 2L LC 2 2L 2L LC
R
Thus the band width β = ω2 - ω1 = and ωo = ω ω
L 1 2
ω ω L 1 L 1 L
Also Q = o = o = = =
β R ωo R C R L C R C

Describing the circuit behavior in terms of ωo , β and Q


R R 1 ωo R R 2
ω =− + ( )2 + = − + ωo ( ) +1
1 2L 2L LC 2 ωo L 2 ωo L
1 1
= ωo { − + ( ) 2 +1 }
2Q 2Q
1 1
Also ω = ωo { + ( ) 2 +1 }
2 2Q 2Q
1 2 ω β
For 1 >> ( ) Then, ω = ωo - o = ωo -
2Q 1 2Q 2
ω β
Also ω = ωo + o = ωo +
2 2Q 2
V V 1
At ωo VL = ωo L = V × Q and VC = × =V×Q
R R ωo C
R 1 ω
β = , ωo = and Q = o
L LC β
1 ωL 1
Z = R + J (ωL− )=R{1+J( − )}
ωC R ωCR
ω ωL ωo
Z =R{1+J( o − )}
ωo R ωo ω C R

125
Chapter 3 The sinusoidal Steady State

ω ωo ω 2 − ωo2
Z=R{1+JQ( − )}=R{1+JQ( )}
ωo ω ωωo
1 (ω − ωo ) ( ω + ωo )
=R{1+J ( )}
β ω
As ω → ω o then, the impedance value in approximate form
2 ( ω − ωo )
may be; Z ≅ R { 1 + J }
β
Example (3-12).
For the circuit shown in Fig. ( 3-32) Calculate;
Q, β, ωo and ω at which i (t) = 0.8 Im and ω1& ω2.
10 Ω 0.2 mH 5 nf

Vm cos ω t

Fig. 3 – 32
1 1
ωo = = =106 Rad./sec.
LC 0.2 ×10 − 3 × 5×10 − 9
ω L 106 × 0.2 ×10 − 3
Q= o = = 20
R 10
ωo 106
β= = = 5×10 4 Rad./sec.
Q 20
V V
I= & Im =
1 2 R
R 2 + (ω L − )
ωC
V V
So 0.8 × =
R 1 2
R 2 + (ω L − )
ωC
1 2
0.64 { R2 + (ω L − ) } = R2
ωC

126
Chapter 3 The sinusoidal Steady State

1
ωL− = ±0.75 R
ωC
0.75 R 1
ω2  ω− =0
L LC
0.75 R 0.75 R 2 1
ω = ±{ ± ( ) + }
2L 2L LC
0.75 R 0.75 R 2 1
ω =− + ( ) +
1 2L 2L LC
0.75 ×10
=- + (1.875 ×10 4 ) 2 +1012 = 98.1×10 4 Rad./sec
2 × 0.2 ×10 − 3

0.75 R 0.75 R 2 1
ω = + ( ) +
2 2L 2L LC
0.75 ×10
= + (1.875 ×10 4 ) 2 +1012 = 101.89 ×10 4 Rad./sec
2 × 0.2 ×10 − 3

More in parallel resonance:-


For the circuit shown in Fig. (3-33) the admittance Y may be;
1 1
Y= + + J ωC
Rg R +JωL

1 R ωL
Y= + + J (ω C − )
R g R 2 + (ω L) 2 2
R + (ω L) 2

R
Rg C Vo
Ig
L

Fig. 3- 33

127
Chapter 3 The sinusoidal Steady State

At resonance the imaginary part may equal zero, assume this


may be happened at ω equals ωo then;
ωo L
ωo C = So ωo2 L2 C + C R 2 = L
2
R + (ωo L) 2
1 R
ωo = − ( )2
LC L
So the admittance value at resonance case may be;
1 R 1 R C L+Rg R C
Y ( ωo ) = + = + =
Rg R2 + L −R2 Rg L LRg
C
LRg
Z ( ωo ) =
L+ Rg R C
LRg
Vo = × Ig
L+ Rg R C

Power Factor Improvement

In ordinary and industrial applications the loads appear


inductive. The required improvement in the power factor (P. F.)
is obtained by connecting a capacitor in parallel with the load.
If the equivalent circuit is resonating, the P. F. in this case is
raised to unity.

Example ( 3- 13).
A 500 kVA transformer is at full load with an overall P. F. 0.6
Lag.. The P. F. is improved by adding a capacitor to raise the p.
F. to 0.9 Lag.. Determine the % of the load which the
transformer carrying after raising the P. F..
P = V I cos ϕ = 500 × 0.6 = 300 kW
Q = V I sin ϕ = 500 × 0.8 = 400 kVAR
At P. F. = 0.9 then, θ = cos – 1 0.9 = 26 o
300
S= = 333 kVA, Thus Q\ = 333 sin 26 = 146 kVAR
0 .9

128
Chapter 3 The sinusoidal Steady State

∴ QC = 400 – 146 = 254 kVAR


333
% of full load = ×100 = 66.7 %
500
Fig. (3- 34) shows the equivalent circuit and the power diagram
of the problem.

333 kVA 500 kVA


254 kVAR
300 KW 300 KW
146 kVAR 400 kVAR
0.9 P.F. 0.6 P.F.
Lag. Lag.
300 KW
θ
ϕ 146 KVAR
333 KVA
500 KVA 254 KVAR

Fig. 3 – 34

Example (3-14).
A certain load may be considered to be equivalent to a
resistance of 35 ohms in series with an inductance of 0.1 H .
Calculate the capacitance required in parallel with the load to
raise the effective power factor to unity. The system voltage is
300
500 volts and its frequency is Hz.
2
The capacitance current must cancel the susceptive component
of the inductive branch current.
R −JωL 35 − J 30
The load admittance Y = =
R 2 + (ω L ) 2 (35) 2 + (30) 2
30
∴ × V = 300 × C × V
(35) 2 + (30) 2
30 1
C= × = 47 µ f
(35) 2 + (30) 2 300

129
Chapter 3 The sinusoidal Steady State

Example ( 3-15).
An industrial load consists of ten induction motors each
absorbing 40 kW at a lagging P. F. of 0.8 and an efficiency of
88 %. To improve the power factor of the plant, three of the
motors are replaced by synchronous motors having a leading
power factor of 0.707 and an efficiency of 92 %. The
mechanical loads on all motors remain unchanged. Calculate
the overall power factor and the total power taken from the
supply under the new conditions.

The induction motor is merely an inductive- resistive load and a


synchronous motor is a capacitive – resistive load.
Before correction the load = 10 × 40 = 400 kW at 0.8 P. F. lag.
88
After correction the load = 7 × 40 + 3 × 40 × = 395 kW
92
KVAR taken by each part of the load;
280
1- Induction motors kVA= = 350 kVA
0.8
KVAR = 350 × sin (cos – 1 0.8) = 350 × 0.6 = 210 kVAR
115
2- Synchronous motors kVA = = 163 kVA
0.707
KVAR = - 163 × sin (cos – 1 0.707) = - 163 × 0.707
= - 115 kVAR
Total kVAR = 210 - 115 = 95 kVAR
Then kVA = ( 395 ) 2 + ( 95 ) 2 = 406.3 kVA
395
So the power factor = = 0.972 Lag.
406.3

Example (3- 16).


A group of induction motors with 500 kW total rating at 0.8 P.
F. Lag. Is to be partially replaced with synchronous motors of
the same efficiency but with 0.707 lead P. F. . The over all P. F.
reaches 0.9 Lag.. Find the % of the load will have been
replaced.

130
Chapter 3 The sinusoidal Steady State

500
S= = 625 kVA
0 .8
ϕ = cos – 1 0.8 = 36.9 o
Q = 625 sin 36.9 = 375 kVAR
For a P. F. = 0.9 Lag. , then θ = cos – 1 0.9 = 26 o
500
Then S\ = = 556 kVA and Q\ = 556 sin 26 = 243 kVAR
0 .9
Fig. (3 – 35) shows a power diagram for the system.
500 kW

θ
ϕ PS 243 kVAR
\
S
S SS
θ 132 kVAR
S

Fig. 3 – 35
θS = cos 0.707 = 45
-1 o

ϕ + θS = 36.9 + 45 = 81.9 o
S
S = 132
sin 53.1 sin 81.9
Then SS = 106.5 kVA
PS = 106.5 cos 45 = 75.3 kW
75.3
So the load percentage which has been replaced = ×100
500
= 15 %

131
Chapter 3 The sinusoidal Steady State

Z and Y Locus

If the impedance of a circuit is plotted as a point on the


complex Z plane the line which joining all such points when
one or more components of Z are vary is named the Z locus. If
only one component of Z is varied the Z locus is either a
straight line or a circle. For the Y locus the same rule may be
followed.
Case 1 Series circuit with variable X constant R.
Z = R ± J X = Z ∠±
The Z locus for this case is a straight line parallel to the X axis
at distance R from the imaginary axis Fig. (3–36) shows the Z
locus.
X
A
ϕ
B R
R
C

Fig. 3 – 36
The line ABC is the Z locus if X varies from ∞ to - ∞.
1
Y = So Y Z = 1 then the Y locus may be a circle as shown
Z
in Fig. (3 -37).
J
D A

0 ϕ F E
• R
X = ∞ and ϕ B X=0
ω = 0 or ∞
C
G

Fig. 3 – 37

132
Chapter 3 The sinusoidal Steady State

0B
From the figure 0D × 0A = 0E cosϕ × = 0E × 0B
cos
If the diameter of the circle is chosen so that 0E × 0B = 1 then
0D × 0A = 1. So if 0A = Z then 0D = Y .
If Z = Z ∠ ϕ then Y = Y ∠ - ϕ
Hence the Y locus corresponding to A is G. So 0 G E is the part
of Y locus corresponding to the Z locus from B and upward to
∞ and 0 D E is the part corresponding from B and downward to
- ∞. The point E represents the resonance value of Y.

Case 2 Series circuit with variable R constant X.


Fig. (3 – 38), (a) shows the Z locus and Fig. (3 – 38), (b) shows
the Y locus.
B
X
R=0 A\
A B Z locus Y locus for
for X +ve X -ve
\
B
R R=∞ G
R=∞
Z locus for B
\ Y locus for
A\ B X -ve X +ve
\
R=0 A

a b
Fig. 3 - 38
Case 3 Parallel circuit with variable B constant G.
Fig. (3 - 39) shows a wiring diagram for the circuit.

R L C

Fig. 3 – 39

133
Chapter 3 The sinusoidal Steady State

1 1 1
Y= + + J ωC = G + J ( ω C - )=G±JB
R JωL ωL
Fig. (3-40(a)) shows the Y locus and (b) shows the Z locus.

B X
ω increase

ϕ ω=0 R R
G G 1
ωL=
ωC

a-Y Locus b-Z Locus


Fig. 3 – 40

Example (3-17).
For the circuit shown in Fig. (3- 41) Find RL which result in
resonance. Draw the Y locus.

RL 4

-J5
J 10

Fig. 3 – 41

1 1 R
Y= + = L + 4 + J ( 5 − 10 )
R + J10 4 − J 5 R 2 +100 41 41 R 2 +100
L L L
At resonance the imaginary part may equal zero then;
5 10
=
41 R 2 +100
L

134
Chapter 3 The sinusoidal Steady State

∴ RL = - 18 So there is no resonance.
1
 = 0.0976 + J 0.122
4−J5
For the other branch at RL = 0 Y = - J 0.1 and at RL = ∞ Y = 0
Fig. (3-42) shows the Y locus.

B
0.122 RL = ∞

0.022
RL = 0

0.097 G
6
Fig. 3 – 42

Example (3-18).
Obtain the current locus for the circuit shown in Fig. (3-43).
Find RC which results in a phase angle of 45o between V and I.

RC
10
-J4

Fig 3 – 43
I=VY
So if the voltage source has a phase angle equals zero then the
current may has a phase shift from the voltage equals the
admittance angle, So;
1 R 4
Y= + C +J
10 R 2 +16 R 2 +16
C C

135
Chapter 3 The sinusoidal Steady State

1 R 4
For a phase shift ϕ = 45o then, + C =
10 R 2 +16 R 2 +16
C C
from which RC = 2 Ω
Fig. (3 -44) (a) Shows the Y locus while Fig. (3-44) (b) shows
the current locus.

0 .25 RC = ∞

RC = 0 45o
0 .1 G 0.1 V V
a b
Fig 3 – 44

Example ( 3- 19).
For the parallel resonance circuit shown in Fig. (3- 45) plot the
admittance locus for variable ω and determine the resonance
frequency, the frequency for minimum admittance, the
minimum admittance, and the upper and lower frequency limits
of band width.

400 Ω
1 nf
1 mH

Fig 3 – 45
400 X 1
YL = −J L And YC = J
2
400 + X 2 400 + X 2
2 X
L L C

136
Chapter 3 The sinusoidal Steady State

1
At ω = 0 YL = & YC = 0
400
And at ω = ∞ YL = 0 & YC = ∞
Fig. (3- 46) shows the admittance locus for the previous circuit.
1
At ω = 0 YL = & YC = 0
400
And at ω = ∞ YL = 0 & YC = ∞
From the locus YR = 0.4 × 10 – 3 S at ωR = 920 k Rad. / Sec.
YMin. = (0.34 + J 0.13) × 10 – 3 S at ω = 1000 k Rad. / Sec.
The band width is defined by the intersection of a circle with a
radius = 2 Y = 2 × ( 0.34) 2 + ( 0.13) 2 × 10 – 3
Min.
= 0.5148 × 10 - 3
So ω1 = 805 k Rad. / Sec. And ω2 = 1205 k Rad. / Sec.
β = ω2 - ω1 = 1205 – 805 = 400 k Rad. / Sec.

YCat ω = 900 Y Locus


k
2 Y Min at ω1

2Y
YCat ω = 700 Min.
k YRes
YCat ω = 500 YMin . 2 Y Min at ω1
k Cat ω = 400
Y
k
YLat ω = 400
YLat ω = 900 k ω = 500
YLat
k YLat ω = 700 k
k
Fig. 3- 46

137
Chapter 3 The sinusoidal Steady State

Problems

1- A load of 300 kW with P. F. of 0.65 lag. The power factor is


improved to 0.9 lag. by adding a parallel capacitor. What kVAR
of capacitors is required and what is the percent reduction in
kVA results?

2- An induction motor load of 1500 w with P. F. of 0.75 lag is


combined with 500 VA of synchronous motors with a P. F. of
0.65 leading. What kVAR of capacitors is required to correct
the overall P. F. of the two groups of the motors to 0.95 lag?
What percent reduction in VA results?

3- A 100 kVA transformer is at 80% of full load with a P. F. of


0.85 lag. What kVA of load at 0.6 P. F. lag. may be added
without exceeding the full load rating of the transformer?

4-A coil of inductance 50µ H and resistance RL is to be tuned


by series capacitance to resonance at 1000 kHz, and bandwidth
of 20 kHz.
Calculate the capacitance required and the necessary Q factor of
the coil. Determine also the frequencies of the half power
points.

5- Show that for a series resonance circuit in which the


inductance and the frequency are fixed, the values of
Q Co
capacitance at the half power points are given by, C =
Q ±1
where Co is the capacitance for resonance and Q is the Q factor
of the circuit. Such a circuit having a coil of inductance 100 µ
H and a Q factor of 10 is tuned to resonance at 1000 kHz.
Calculate the required tuning capacitance and the two values at
the half power points.

138
Chapter 3 The sinusoidal Steady State

6- A coil of inductance L and resistance RL is connected in


parallel with a capacitor of capacitance C and equivalent series
resistance RC.
Show that the frequency of resonance is given by;
2
ωo L − C R L
fo = where ω 2 L C =1.
2π L−CR 2 O
C
L
Show also if RL = RC = the circuit is non resonant.
C

7- For the parallel circuit shown in Fig. (3 - 47) is to be made


resonant by varying L. Construct the admittance locus diagram
and determine L for resonance if ω = 5000 Rad. / Sec.

2 5

L 20 µ f

Fig. 3 – 47

8- For the circuit shown in Fig. (3 - 48) find the value of R


which result in parallel resonance and draw the locus diagram.

R 5

J4 -J8

Fig. 3 – 48

9- For each of Fig. (3 – 49) (a & b) a locus diagram is given for


the total current taken by the circuit which contains one

139
Chapter 3 The sinusoidal Steady State

variable element. Describe a circuit that corresponding to each


locus.

Locus of It
Locus of It
V
V
a b
Fig. 3 – 49

10- A fixed inductive reactance of 10 Ω is connected in series


with a resistance R, variable over the range 0 → ∞ . The two
are connected in parallel with a resistance of 10 Ω to a 200 V,
fixed frequency supply.
Draw the locus of the supply current phasor and hence show
that the total range of its phase angle variation relative to the

applied voltage, as R is varied, is 0 → − .
4

11- The parallel circuit shown in Fig. (3-50) It lags the applied
voltage by 53.1o when R = 0. Then if R = ∞ It leads the voltage
by the same angle.
Construct the admittance locus diagram to illustrate this
condition. At what value of R is the circuit resonant?

It
R 6
100 ∠0o V
J 6.25 -J8

Fig.3 – 50

140
Chapter 4 Circuit analysis

CHAPTER 4

Circuit analysis
Mesh current: -
Closed loop of current called mesh current or loop current as
shown in Fig. (4-1). The circuit showed in this figure has three
loops, then write three equations in the unknowns I1, I2 and I3
and solve them. Now the current in any branch is given either
directly by one of the mesh currents or by a combination of
them.
ZA ZC ZE

I1 ZB I2 ZD I3
VA VB

Fig. 4 – 1
To obtain the set of three equations apply Kirchoff’s voltage
law for each current loop.
I1 ZA + (I1 – I2)ZB = VA
I1(ZA + ZB) – I2 ZB = VA ………..1
(I2 – I1) ZB + I2 ZC + (I2 – I3)ZD = 0
- I1 ZB + I2(ZB + ZC + ZD) – I3 ZD = 0 ………...2
(I3 – I2) ZD + I3 ZE = - VB
- I2 ZD + I3 (ZD + ZE) = - VB ………...3
How to determine the required number of mesh current?
Fig. (4-2) shows an electric circuit that it’s required to
determine the required number of mesh current for it.

Fig. 4 – 2

141
Chapter 4 Circuit analysis

To determine the required number of current loops for any


circuit the following steps are followed;
1- Name the nodes (the node is the point to which three or more
elements are connected).
2- Determine the solid branches (the solid branch is the
element, which connect between two nodes and not form a
closed path’s), the solid branches are called tree branches. The
tree branches contain all nodes and not form closed paths.
3- Determine the link branches (the link branch is the element,
which connect between two nodes and form a closed loop) and
is simulated by a doted line.
4- The number of mesh currents required by the network is the
number of the link branches.
So the electric circuit that had been shown in Fig. (4-2) has the
following;
Four nodes, three branches and four links, so the number of
meshes current required are four. Fig. (4-3) shows the tree.
1 2
1 • • • 3
• • 3 •
2
4

4

Fig. 4 – 3
Example (4-1).
For the circuit shown in Fig. (4-4) evaluates the branches
current.

The two parallel branches may combine as equivalent one


branch.
4x5
Req = = 2.22 Ω
9

142
Chapter 4 Circuit analysis

1 2

4 5
2 3

5A

Fig. 4 – 4
Fig. (4 – 5) shows the circuit after the combination.
1 2

2 2.22 3

5A

Fig. 4 – 5

Fig. (4 – 6) shows the network tree, from which it’s possible to


deduce;
4
• 3
I2 2 I3
1 • 1 • 3
I1

2
5A
4
Fig. 4 – 6

143
Chapter 4 Circuit analysis

From the previous figure it’s possible to deduce that;


The number of nodes is four. The links number is four. So the
number of equations is four.
From the tree network shown in Fig. (4-6) it’s possible to write
the following equations;
I1 (4) + (I1 + I3 – 5) (3) + (I1 + I2 -5) (2) = 0
I1 (4 + 3 + 2) + I2 (2) + I3 (3) = 25
9 I1 + 2 I2 + 3 I3 = 25 ……….1
(I2 – I3) (2.22) + (I1 + I2 -5) (2) + I2 (1) = 0
I2 (2.22 + 2 + 1) + I1 (2) – I3 (2.22) = 10
2 I1 + 5.22 I2 – 2.22 I3 = 10 ……….2
I3 (2) + (I1 + I3 – 5) (3) + (I3 – I2) (2.22) = 0
I3 (2 + 3 + 2.22) + I1 (3) – I2 (2.22) = 15
3 I1 – 2.22 I2 + 7.22 I3 = 15 ……….3
Rewriting equations from 1 to 3 in matrix form as;

9 2 3   I1  25
2 5.22 − 2.22 * I  = 10 
   2  
3 − 2.22 7.22   I 3  15 
Solving the previous matrix equation for I1, I2 and I3 then;
25 2 3
10 5.22 − 2.22
Δ 15 − 2.22 7.22 306.5
I1 = 1 = = =1.593 A
Δ 9 2 3 192.34
2 5.22 − 2.22
3 − 2.22 7.22
9 25 3
2 10 − 2.22
Δ 3 15 7.22 422
I2 = 2 = = = 2.;194A
Δ 9 2 3 192.34
2 5.22 − 2.22
3 − 2.22 7.22

144
Chapter 4 Circuit analysis

9 2 25
2 5.22 10
Δ 3 − 2.22 15 402
And I3 = 3 = = = 2.09 A
Δ 9 2 3 192.34
2 5.22 − 2.22
3 − 2.22 7.22
Fig. (4- 7) shows the circuit with the branches current values
indicated on it.
1 2.194 A 2.09 A 2

0.058 0.046 A
4 1.593 A A
4 5
2 3

1.213 A 1.317 A
5A

Fig. 4 – 7
Example (4-2).
For the circuit shown in Fig. (4-8) determine the loops current.
J 4Ω

-J 8Ω 10 Ω
5Ω 8Ω 5 ∠30o V

10 ∠0o V
3Ω J4Ω

Fig. 4 – 8

145
Chapter 4 Circuit analysis

Fig. (4- 9) shows the tree from which the number of nodes is
four, the number of links is three and so the number of
unknowns current is three.
1

I1 I2
2 4
3
I3

Fig. 4 – 9
Writing the loop equation for the unknown currents then;
I1 (5 + 10 – J 8) – I2 (10) – I3 (5) = 0
(17 ∠ - 28.07o) I1 – 10 I2 – 5 I3 = 0 ……………..1
I2 ( 8 + 10 + J 4) – I1 (10) – I3 (8) = - 5 ∠ 30 o

- 10 I1 + (18.44 ∠ 12.53o) I2 – 8 I3 = - 5 ∠ 30o ……………. 2


I3 (5 + 8 + J 4 + 3) – 5 I1 – 8 I2 = - 10
- 5 I1 – 8 I2 + (16.292 ∠ 14.036o) I3 = - 10 ……………..3
Rewriting the previous equations in matrix form as;

17 ∠ − 28.07 − 10 −5   I1   0 
 −10 18.44 ∠12.53 −8  * I  = − 5 ∠ 30
   2  
 −5 −8 16.292 ∠14.036  I 3   −10 

Then;
0 − 10 −5
− 5 ∠ 30 18.44 ∠12.53 −8
Δ1 −10 −8 16.292 ∠14.036
I1 = =
Δ 17 ∠ − 28.07 − 10 −5
−10 18.44 ∠12.53 −8
−5 −8 16.292 ∠14.036

146
Chapter 4 Circuit analysis

2616.06 ∠ −160.5
I1 = = 1.92 ∠- 155.3o A
1363.11∠ − 5.2
17 ∠ - 28.07 0 −5
−10 - 5 ∠ 30 −8
Δ2 −5 −10 16.292 ∠14.036
I2 = =
Δ 17 ∠ − 28.07 − 10 −5
−10 18.44 ∠12.53 −8
−5 −8 16.292 ∠14.036
1115.06 ∠ 102.64
= = 0.818 ∠ 107.84o A
1363.11∠ − 5.2
17 ∠ - 28.07 - 10 0
−10 18.44∠12.53 − 5 ∠ 30
Δ2 −5 −8 −10
I3 = =
Δ 17 ∠ − 28.07 − 10 −5
−10 18.44 ∠12.53 −8
−5 −8 16.292 ∠14.036
2997.13 ∠ 166.65
= = 2.199 ∠ 171.85o A
1363.11∠ − 5.2

Example (4-3).
If the two equal capacitors C and the resistor R in Fig. (4- 10)
are adjusted until the current in ZD is zero. Find RX and LX in
terms of R, C, and ω.

The previous circuit has four loops, so four currents may be


assumed as shown in Fig. (4-11).

147
Chapter 4 Circuit analysis

C
C

ZG
ZD
VG RX
LX

Fig. 4 – 10
R

I2 C
C

ZG
ZD
VG RX
I3 LX
I1 I4

Fig. 4 – 11

Writing the loops equation in matrix form then;

148
Chapter 4 Circuit analysis

 1 1 
 Z G + R X + Jω C −
Jω C
−R
X
0 
 
 1 2 1 
− R+ 0 −
 Jω C Jω C Jω C 
 − R 0 R + Jω L − Jω L 
 X X X X 
 1 1 
 0 − − Jω L Z + J(ω L − )
 Jω C X D X ω C 
 I1 
  V 
I   G 
0
*   =  
2
0
 I3   
   0 
I 4 
When the current I4 equals zero then the current through ZD is
zero. So for I4 = 0 then ∆4 may equals zero. Thus;

1 1
ZG + R X + − − RX VG
Jω C Jω C
1 2
− R+ 0 0
∆4 = Jω C Jω C =0
− RX 0 R X + Jω L X 0
1
0 − − Jω L X 0
Jω C

 1  1   2 
- VG −  (R X + Jω L X ) + R X − Jω L X (R + )  =0
 Jω C  Jω C   Jω C 
1 1
RX = 2 2 & LX =
ω C R 2ω 2 C

Node Voltage:-

A node is a point in a network to which three or more circuit


elements are connected. A node voltage is the voltage of a

149
Chapter 4 Circuit analysis

given node with respect to one node called the reference node.
For the circuit which has been shown in Fig. (4 - 12) if node
three is chosen as the reference node, then V13 is the voltage
between nodes 1 and 3 . Also V23 is the voltage between nodes
2 and 3.

1 2
ZA ZC ZE

Vm ZB ZD
Vn
3

Fig. 4 – 12

Applying Kirchoff’s current law at nodes 1 and 2 then;


V − Vm V V − V
1 + 1 + 1 2 =0
Z Z Z
A B C
1 1 1 V V
V1 ( + + )− 2 = m
Z Z Z Z Z
A B C C A
V1 (YA + YB + YC ) – V2 YC = Vm YA …………1
V − Vn V V −V
2 + 2 + 1 2 =0
Z Z Z
E D C
1 1 1 V V
V2 ( + + )− 1 = n
Z Z Z Z Z
E D C C E
V2 (YE + YD + YC ) – V1 YC = Vn YE ………….2
Or in matrix form as follows;
(Y + Y + Y ) −Y   V  Vm Y 
 A B C C * 1  =  A
 −Y (Y + Y + Y )  V   Vn Y 
C E D C   2  E
Solving the previous matrix equation for V1 and V2 then;

150
Chapter 4 Circuit analysis

Vm Y −Y
A C
Vn Y (Y + Y + Y )
V1 = E E D C
(Y + Y + Y ) −Y
A B C C
−Y (Y + Y + Y )
C E D C

(Y + Y + Y ) Vm Y
A B C A
−Y Vn Y
And V2 = C E
(Y + Y + Y ) −Y
A B C C
−Y (Y + Y + Y )
C E D C

Example (4 – 4).
For the circuit shown in Fig. (4-13) find the branches current.
1 B 2

4
A 4 C
D 5
2 3

5A

Fig. 4 - 13

The circuit has four nodes A, B, C, and D. choosing node C as a


reference node, then the circuit may have three nodes A, B, and
D.
At node A
VA ( 1 + 0.5 + 0.25) – VB – 0.5 VD = - 5
1.75 VA – VB – 0.5 VD = - 5 ………………..1
and node B
VB (1 + 0.25 + 0.5 + 0.2) – VA – VD (0.25 + 0.2) = 0

151
Chapter 4 Circuit analysis

1.95 VB – VA – 0.45 VD = 0 ………………….2


At node D
VD ( 0.5 + 0.25 + 0.333 + 0.2) – 0.5 VA – VB (0.25 + 0.2) = 0
1.283 VD – 0.5 VA – 0.45 VB = 0 ………………….3
Rewriting the three previous equations in matrix form as
follows;
 1.75 −1 − 0.5  VA  − 5
 −1 1.95 − 0.45 *  VB  =  0 
     
− 0.5 − 0.45 1.283  VD   0 
Solving the previous matrix equation for VA, VB and VD then;
- 5 −1 − 0.5 
 0 1.95 − 0.45
 
 0 − 0.45 1.283  −11.4968
VA = = = − 6.375 V
 1.75 −1 − 0.5  1.8034
 −1 1.95 − 0.45
 
− 0.5 − 0.45 1.283 
1.75 − 5 − 0.5 
 -1 0 − 0.45
 
- 0.5 0 1.283  − 7.54
VB = = = − 4.181 V
 1.75 −1 − 0.5  1.8034
 −1 1.95 − 0.45
 
− 0.5 − 0.45 1.283 
1.75 −1 − 5
 -1 1.95 0
 
- 0.5 − 0.45 0  − 7.125
VD =  = = − 3.951 V
 1.75 −1 − 0.5  1.8034
 −1 1.95 − 0.45
 
− 0.5 − 0.45 1.283 

152
Chapter 4 Circuit analysis

Fig. (4-14) shows the previous circuit with node voltage and
branches current magnitudes.

1 2.19 - 4.181 V 2 2.09


4
4 0.058 0.046 1.593

- 6.375 V 4 5
3
2
1.213 - 3.951 V 1.317
5A

Fig. 4 – 14

Example (4- 5).


For the circuit shown in Fig. (4–15) find the line currents IA, IB
and IC.
IA

150 ∠ 120o V 10∠45


IB
10∠45

150 ∠ 0o V 10∠45
IC

Fig 4 – 15

The circuit which had been shown in Fig (4–15) has three
nodes. Taking one of them as a reference remains two nodes,
these nodes are shown in Fig. ( 4-16).

153
Chapter 4 Circuit analysis

IA 1

150 ∠ 120o V 10∠45


IB
10∠45

150 ∠ 0o V 10∠45
IC
2
Fig 4 – 16

Applying Kirchoff’s law at nodes 1 and 2;


V1 = 150 ∠ 120o V & V2 = 150 ∠ 180o V
V1 V − V2 2 V1 − V2
IA = + 1 =
10 ∠45 10 ∠45 10 ∠ 45
300 ∠120 −150 ∠180
= = 26 ∠45o A
10 ∠ 45
V1 V2
IB = - { + }
10 ∠45 10 ∠45
150 ∠120 +150 ∠180
=− = 26 ∠ − 75o A
10 ∠ 45
V2 V − V1 2 V2 − V1
IC = + 2 =
10 ∠45 10 ∠45 10 ∠ 45
300 ∠180 −150 ∠120
= = 26 ∠ 165o A
10 ∠ 45

154
Chapter 4 Circuit analysis

Problems

1- In the two-mesh network of Fig. (4-17), find the power


supplied by the source and the power in each resistor.

2Ω - J2 Ω 3 Ω - J5 Ω

10 ∠0o V J2 Ω
1Ω

Fig. 4 – 17

2- For the circuit of Fig. (4-18) find the power which each
source supplies to the network.
J2Ω 5Ω -J2Ω

4Ω
2Ω
10∠90o V 10∠90o V

Fig. 4 – 18

3- In the circuit of Fig. (4-19) find V1 which causes the current


I1 to be zero.

155
Chapter 4 Circuit analysis

50 ∠0o V 5 Ω
I1

10 Ω J5Ω

2Ω V1
-J 4 Ω -J 4 Ω

Fig. 4 – 19

4- For the circuit of Fig. (4-20) determine the voltage V such


that no current flows through the 18 ohm resistor.

J5 -J20 10 18 J 30

J20 -J10 J 20
V 10∠60o V
4 88 6

-J30

Fig. 4 - 20

5- Determine the voltage across the source and the power


delivered by the source in the circuit of Fig. (4-21).
10 Ω J20 -J5 Ω

10 ∠30 A 10 Ω 5Ω 10 Ω

Fig. 4 – 21

156
Chapter 4 Circuit analysis

6- In the network of Fig. (4–22), find the power in the 6 ohm


resistor by the nodal method.

5Ω 2 Ω J3 Ω 4Ω

30∠0 V J5 Ω 6Ω 20∠0 V

Fig. 4 – 22

7- In the network of Fig. (4-23), find the voltage V such that the
current in the 4 ohm resistor is zero. Select one end of the
capacitor as the reference node.
5Ω 4Ω 2Ω

V J2 Ω -J2 Ω 50 ∠90oV

Fig. 4 – 23

8- In the network of Fig. (4-24), find the ratio of the two node
V
voltages 1 .
V
2
5 Ω -J2 Ω J5 Ω 2Ω

10 ∠30oV 3Ω V1 V2 5Ω -J2 Ω

Fig. 4 – 24

157
Chapter 4 Circuit analysis

9- In the network of Fig. (4-25), find the voltage V such that it’s
current will be zero.
J5 Ω J2 Ω 5Ω

10 ∠ 0 V 3Ω -J2 Ω V

Fig. 4 – 25

10- In the network of Fig. (4-26), find the driving current I


which results in a voltage VAB of 5 ∠ 30o V.
10 Ω A 2Ω

2Ω
I J5 Ω J5 Ω VAB - J2 Ω
J2 Ω

Fig. 4 – 26

158
Chapter 4 Circuit analysis

Mutual inductance

Self inductance:-
When a current is changing in a magnetic circuit as shown in
Fig. (4- 27), the magnetic flux linking the same circuit changes
and an Electro Motive Force (E. M. F.) is induced in it.
ϕ

L
i (t)

V (t)
Fig 4 – 27

d i (t)
VL = L where L is the self inductance of the circuit and
dt
its unit is the hennery (H).
For a coil of N turns the induced E. M. F. is given by
d
VL = N
dt
d i (t) d
∴L =N
dt dt
d
L=N
d i (t)

Mutual inductance:-

Consider a magnetic circuit as shown in Fig. (4–28) the


changing in the current i1 (t) establishes a magnetic flux ϕ1 .
Part of this flux links coil 1 only and is called leakage flux ϕ11
and the

159
Chapter 4 Circuit analysis

i1 (t)
ϕ11
1 N1 N2 2

ϕ12

Fig. 4 - 28

remaining flux links coil 2 and is named ϕ12. The induced


voltage in coil 2 is given by;
d 12
V2 = N2
dt
 ϕ12 is related to the current i1 (t) then V2 is proportional to the
d i1 (t) d i1 (t)
rate of change of i1 (t) i. e. V2 α . Then V2 = M
dt dt
where M is the constant of proportionality and is called the
mutual inductance between the two coils. The unit of M is the
same as the unit of L (the Henry).
d 12 d i (t)
∴ V2 = N2 =M 1
dt dt
d 12
M = N2
d i1 (t)
If the flux and current are linearly related then;
12
M = N2
i1 (t)
If i2 (t) is introduced in coil 2 then the linking fluxes are ϕ2, ϕ21
d i 2 (t)
and ϕ22 then the induced voltage in coil 1 is V1 = M
dt
d  21 
∴ M = N1 = N1 21
d i 2 ( t) i 2 (t)

160
Chapter 4 Circuit analysis

Coupling coefficient K :-

The fraction of the total flux that links the coils is called the
coefficient of coupling K.
12  21
Then K = =
1 2
 ϕ12 ≤ ϕ1 and ϕ21 ≤ ϕ2 hence the maximum value of K = 1.
12  21 K 1 K2
M2 = ( N2 ) * (N1 ) = (N2 ) * (N1 )
i1 (t) i 2 (t) i1 (t) i 2 (t)
K 1 K2 1 2
= (N1 ) * (N2 ) = K2 (N1 ) * (N2 )
i1 (t) i 2 (t) i1 (t) i 2 (t)
= K L1 L2
M = K L1 L 2

Analysis of coupling circuits:-


Fig. (4 – 29) shows a coupling circuit from which it’s possible
to deduce.
ϕ1 ϕ2
I1 R1 R2 I2

V1 L1 M L2 V2

Fig. 4 -29

d i1 (t) d i (t)
R 1 i1 (t) + L1 ± M 2 = V1
dt dt
d i (t) d i (t)
And R 2 i 2 (t) + L 2 2 ± M 1 = V2
dt dt
The voltage induced due to the mutual inductance may be of
either polarity depending on the winding sense. To determine

161
Chapter 4 Circuit analysis

the correct sign apply the right hand rule to each coil, allowing
the fingers to warp in the direction of the assumed current, then
the thumb in the direction of the flux. Thus the + ve direction of
ϕ1 and ϕ2 are as plotted in Fig. (4 – 29).
 ϕ1 and ϕ2 opposes each other, so the –ve sign may be taken
as indicated in the following equations;
d i (t) d i (t)
R 1 i1 (t) + L1 1 − M 2 = V1
dt dt
d i (t) d i (t)
And R 2 i 2 (t) + L 2 2 − M 1 = V2
dt dt
Rewriting the previous equations in the steady state form as;
(R1 + J ω L1) I1 – J ω M I2 = V1
And (R2 + J ω L2) I2 – J ω M I1 = V2

Natural current:-
Due to the mutual coupling between the two circuits shown in a
current I2 is deduced.
ϕ1 ϕ2
I1 R 1 2 1 R2 I2

V1 JωL1 JωL2
JωM

Fig. 4 – 30

From the circuit which had been shown in Fig. (4-30) it’s
possible to have;
(R1 + J ω L1) I1 – J ω M I2 = V1
And (R2 + J ω L2) I2 – J ω M I1 = 0

162
Chapter 4 Circuit analysis

Dot rule for coupled coils:-


On each coil a dot is placed at the terminals which are
instantaneously of the same polarity on the basis of the mutual
inductance alone.
To assign the dots on a pair of coupled coils, select a current
direction in one coil of the pair and place a dot at the terminal
where this current enters the winding. The doted terminal is
instantaneously positive with respect to the other terminal of the
coil, apply right hand rule to find the corresponding flux.
Now on the second coil the flux must oppose the original flux,
according to Lenz’s law. Use the right hand rule to find the
direction of the natural current and since the voltage of the
mutual inductance is positive at the terminal where this natural
current leaves the winding. Place a dot at this terminal. Fig. (4 –
31) shows a circuit this rule is applied on it.
ϕ ϕ
I •

I

Fig. 4 – 31

The dot rule:-


1- When both assumed currents enter or leave a pair of coupled
coils at the dotted terminals the signs of M terms will be the
same as the signs of the L terms.
2- If one current enters at a dotted terminal and the other leaves
the doted terminal the sign of the m terms are opposite to the
sign of the L terms.
Fig. (4 – 32) shows a circuit for indicating an application on
this rule.

163
Chapter 4 Circuit analysis

I1 I2
• •
V1 JωM

Fig. 4 – 32

Z11 I1 – J ω M I2 = V1 and Z22 I2 – J ω M I1 = 0


Conductivity coupled equivalent circuit:-
For the circuit shown in Fig. (4–33) the following matrix
equation may represent the loop voltage equation across the two
loops.
R1 I1 JωM R2 I2
• •
V1 J ωL1 J ωL2 V2

Fig. 4 – 33

(R 1 + J ω L1 ) − J ω M   I1   V1 
 −JωM =
(R 2 + J ω L 2 ) I 2  V2 
*

The circuit in Fig. (4 – 34) shows an equivalent electric circuit


for the previous magnetic circuit.

R1 Jω (L1-M) Jω (L2-M) R2
I1 I2
V1 JωM V2

Fig. 4 – 34

164
Chapter 4 Circuit analysis

Fig. (4 – 35) (a) shows a magnetic circuit which is simplified in


Fig. (4 – 35) (b).
( R1 + J ω L1 ) I – J ω M I + J ω L2 I – J ω M I = V
I {R1 + J ω ( L1 + L2 – 2 M )} = V

ϕ ϕ

I R1 J ωL1 J ωL2
J ωM
V
(a)

I1 R1 J ω L1 J ω L2
• •
JωM
V
(b)

Fig 4 – 35

The circuit which is shown in Fig. (4–36) simulates the


equivalent circuit which has been shown in Fig. (4-35).

I R1 Jω (L1+L2-2M)

V
Fig 4 – 36
Example (4-6).
For the magnetic circuit which is shown in Fig. (4-37) draw the
equivalent circuit of it.

165
Chapter 4 Circuit analysis

I
• MC •
L1 MA L3
MB

V
L2
R
C

Fig 4 – 37

Fig. (4-38) shows the electric circuit which simulate the


magnetic circuit shown in Fig. (4 - 37), from this circuit its
possible to deduce that.
1
{ R + J ω ( L1 + L2 + L3 + 2 MA – 2 MB – 2 MC) + }I=V
J ωC
I Zeq = V
Where Zeq = R + J ω ( L1 + L2 + L3 + 2 MA – 2 MB – 2 MC) -
1
J
ωC
JωMC
JωMA JωMB
• • •
JωL1 R JωL2 1 JωL3
Jω C
I V
Fig. 4–38

So the equivalent circuit which represents the previous


magnetic circuit is shown in Fig. (4–39).

166
Chapter 4 Circuit analysis

Zeq

I
V
Fig. 4 – 39

Example (4-7).
In the coupled circuit shown in Fig. (4–40) find the potential
difference across the 5 Ω terminals. Then reverse the polarity in
one coil and repeat.
J XM = J K X L X L = J 0.8 5 x 10 = J 5.66 Ω
1 2

I1 • K= 0.8

J5 J 10 I2
3
50∠0 5
-J4

Fig. 4 – 40

Righting the loop equations for this circuit in matrix form as


follows;

 3 + J1 − (3 − J4 + J 5.66)  I1  50 ∠0


− 3 − J1.66 +  * I  =  0 
 8 J 6   2  
3 + J1 50 ∠0
− 3 − J1.66 0 171∠ 29
I2 = = = 8.6 ∠- 24.8oA
3 + J1 − (3 − J4 + J 5.66) 19.9 ∠ 53.8
− 3 − J1.66 8+ J 6

167
Chapter 4 Circuit analysis

Then the voltage asross the 5 ohm resistor = 5 × 8.6 ∠- 24.8o


= 43 ∠- 24.8o V
When changing the polarity of one coil then;

 3 + J1 − (3 − J4 − J 5.66)  I1  50 ∠0


 *  =  
− 3 + J 9.66 8+ J 6  I 2   0 

3 + J1 50 ∠0
− 3 + J 9.66 0 505 ∠ − 72.7
I2 = = = 3.83 ∠ - 112.1o A
3 + J1 − 3 + J9.66 132 ∠ 39.4
− 3 + J 9.66 8+ J 6
So, V = 5 × 3.83 ∠ - 112.1o = 19.15 ∠ - 112.1 V

Example (4 – 8).

Obtain the conductively coupled equivalent circuit for the


mutually coupled network shown in Fig. (4–41).
5Ω

2 4Ω
Ω I2
V I1 •
J8 J4
J3
Ω Ω
• Ω

Fig. 4 – 41

For loop 1
5 I1 + (2 + J 8) (I1 – I2) – J 4 I2 = 0
(7 + J 8) I1 – (2 + J 12) I2 = V
For loop 2

168
Chapter 4 Circuit analysis

(I1 – I2) (2 + J 8) – (4 + J 3) I2 – J 4 I2 + (I1 – I2) J 4 = 0


(2 + J 12) I1 – (6 + J 19) I2 = 0
 (7 + J 8) − (2 + J12)  I1  V 
− (2 + J12) (6 + J19)  * I  =  0 
   2  
The conductively coupled equivalent circuit is shown in Fig.
(4-42).
I1 I2
5–J4 4+J7

V1 2 + J 12

Fig. 4 – 42

Example (4-9).
For the circuit shown in Fig. (4–43) calculate the source and
load currents for;
a) The winding as shown.
b) One winding reversed. Taking ω = 100 Rad. / Sec.

5 Ω M = 0.1 H
• •
10 Ω, 15 Ω,
0.1 H 0.2 H

100 J5Ω
∠0 10 Ω
V I1
5Ω I2

Fig 4 – 43

169
Chapter 4 Circuit analysis

Zm = J 100 × 0.1 = J 10 Ω
a- The loops current is assumed on the circuit which is shown
in Fig. (4- 43) then;
For loop 1;
I1 (5 + 10 + J 10 +J 5 + 5) – I2 ( 5 + J 5 + J 10) = 100 ∠ 0
I1 ( 20 + J 15) – I2 (5 + J 15) = 100 ∠ 0 ……….(1
And for loop 2;
- I1 × J 10 – I1(J 5 + 5) + I2 (5 + J 5 + 15 + J 20 + 10) = 0
- I1 ( 5 + J 15) + I2 ( 30 + J 25) = 0 ……. (2
Solving equations 1 & 2 for I1 and I2 So,
I1 = 4. 311 ∠ - 22o 13\ A and I2 = 1.745 ∠ 9o 33\ A

b – By reversing one of the two winding then;


For loop one;
I1 (20 + J 15) + I2 × J 10 – I2 (5 + J 5) = 100 ∠ 0
I1 (20 + J 15) – I2 ( 5 – J 5) = 100 ……(1
For loop two;
- I1 (5 + J 5) + I1 × J 10 + I2 ( 30 + J 25) = 0
- I1 ( 5 – J 5) + I2 (30 + J 25) = 0 …….(2
From equations 1 and 2 then;
I1 = 3.81∠ - 37o 31\ A and I2 = 0.69∠ - 122o 19\ A

170
Chapter 4 Circuit analysis

Problems

1- For the coupled circuit shown in Fig. (4-44), find the voltage
V
ratio 2 which results in zero current I1 . Repeat for zero
V1
current I2.
5Ω 2Ω
J2 Ω
• •
V1 J8 Ω J2 Ω V2
I1 I2

Fig. 4 -44

2- For the series circuit shown in Fig. (4-45), find K and place
the dots such that the circuit is in series resonance.
K
18 Ω J8 Ω J10 Ω -J20 Ω

Fig. 4 – 45

3- For the circuit shown in Fig. (4-46) find K and place the dots
so that the power output from the source is 168 watt.
10 J5 Ω K J8 Ω

50∠ 0 V
Fig 4 – 46

4- Determine the value of K in the coupled circuit of Fig. (4-47)


if the power in the 10 Ω resistor is 32 watts

171
Chapter 4 Circuit analysis

J5 Ω

K

20 ∠0 V J8 Ω 10 Ω

Fig. 4 - 47

5- Referring to the circuit of Fig. (4-48) find the equivalent


impedance at the terminals of the source.
5Ω J4 Ω
• •
J3 Ω
J5 Ω
V -J8 Ω

Fig. 4- 48

6- Referring to the circuit of Fig. (4-48) find the voltage across


the J 5 Ω reactance if the source V = 50 ∠ 45o V.

172
Chapter 4 Circuit analysis

Circuit theorems

1- Superposition theorem:-

The response in any element of a linear network containing two


or more sources is the vector sum of the responses obtained by
each source acting separately and with all the other sources set
equal zero.

Example (4 – 10).
Apply the superposition theorem to the network shown in
Fig. (4-49) for obtaining the current in the (3 + J 4) ohm
impedance.
5 J5

3
V1= 50 ∠90o V2= 50 ∠0o
J4

Fig. 4 - 49

Firstly letting V2 = 0 then the circuit may be as shown in Fig.


(4 – 50).
I1 5 J5

3
V1 = 50∠90 Ω I\
J4

Fig 4 – 50

173
Chapter 4 Circuit analysis

J 5 (3 + J 4)
Zeq = 5 + = 5.83 + J 2.5 = 6.35 ∠ 23.2o Ω
3+ J 9
V 50 ∠ 90
I1 = 1 = = 7.87 ∠ 66.8 o A
Z eq 6.35 ∠ 23.2
J5 J5
I \ = I1 × = 7.87 ∠ 66.8 × = 4.15 ∠ 85.3o A
3+ J 9 3+ J 9
Secondly let V1 = 0, the circuit may be as shown in
Fig. (4 – 51).
5Ω J5

3Ω
I\\ V2 = 50∠ 0
J4
I2

Fig. 4 – 51

5 (3 + J 4)
Zeq = J 5 + = 2.5 + J 6.25 = 6.74 ∠ 68.2o Ω
8+ J 4
V 50 ∠ 0
I2 = 2 = = 7.42 ∠ − 68.2 o A
Z eq 6.74 ∠ 68.2
5 5
I\\ = I2 × = 7.42 ∠- 68.2 × = 4.148 ∠- 94.765o A
8+ J 4 8+ J 4
So I = I - I = 4.15 ∠ 85.3 - 4.148 ∠- 94.765
\ \\

= 8.298 ∠ 85.268o A

2- Reciprocity theorem:-
In a linear, single source network the ratio of excitation to
response is constant when position and response are
interchanged.

174
Chapter 4 Circuit analysis

Example (4- 11).


For the circuit shown in Fig. (4–52), verify the reciprocity
theorem. I1

2
5 Ω I
V Ω
X J5 -J 2 5 ∠90o A

Fig. 4 – 52

From the circuit shown in Fig. (4-52)


−J2 −J2
I1 = I × = 5 ∠ 90 × = 1.31 ∠ - 23.2o A
7+J3 7+J3
So VX = I1 × (5 + J 5) = 1.31 ∠ - 23.2 × 7.071 ∠ 45
= 9.263 ∠ 21.8o V
Applying the reciprocity theorem then the circuit may be as
shown in Fig. (4- 53).
I I2

2Ω
5Ω
5 ∠ 90 A
o

J5 -J2 VX

Fig. 4 – 53

5+ J5 5+ J5
I2 = I × = 5 ∠ 90 × = 4.63 ∠ 111.8o A
7+J3 7+J3
So VX = I2 × (- J 2) = 4.63 ∠ 111.8 × 2 ∠- 90 = 9.26 ∠ 21.8o V
Thus the reciprocity theorem is verified.

175
Chapter 4 Circuit analysis

3- Maximum power transfer theorem:-

This theorem determines the values of the load impedance


which results in maximum power transfer across its terminals
when connect to an active network.

Case 1:The load is a variable resistor RL:-


Consider the circuit as shown in Fig. (4 – 54).

Rg JXg
Vg RL
I

Fig. 4 – 54
Vg
I=
R L + R g + J Xg
Vg
I=
(R L + R g ) 2 + X g2
The transfer power to RL is,
2 Vg2
P = I × RL = × RL
(R L + R g ) 2 + X g2
dP
For maximum power Pmax. then =0
dRL

dP 2 { (R + R g ) 2 + X g2 } − 2 × R (R + R g )
∴ = Vg [ L L L ] =0
dR 2
{ (R + R g ) + X g } 2 2
L L
So R g + 2 R L R g + R L + X g − 2R L R g − 2 R 2L = 0
2 2 2

R g2 + X g2 − R 2L = 0
RL = R g2 + X g2 = Z g

176
Chapter 4 Circuit analysis

Case 2:The load is a variable impedance ZL:-


The circuit shown in Fig. ( 4 – 55) simulates this case.

Rg J Xg

Vg J XL

I RL

Fig. 4 – 55
Vg
I=
R L + R g + J ( XL + Xg )
Vg
I=
(R + R g ) 2 + (X + X g ) 2
L L
The transmitted power to ZL is,
Vg2 × R
2
P = I × RL = L
(R + R g ) 2 + (X + X g ) 2
L L
Assuming RL is held constant while XL is variable, then the
Vg2 × R
value of P is maximum when Xg = - XL so Pmax = L
(R + R g ) 2
L
Consider now RL is variable as indicated in case 1, so the
maximum power is delivered to the load when RL = Rg.
Vg2
∴ Pmax =
2Rg
Thus the maximum transferred power to the variable load
impedance ZL is happened when XL = - Xg and RL = Rg
( i. e. ZL = Z*g )

177
Chapter 4 Circuit analysis

Case 3:The load impedance is a variable resistance RL and


fixed reactance XL:-
This case is simulated in the circuit of Fig. ( 4 – 56).
Rg J Xg

Vg J XL

I RL

Fig. 4 – 56
Vg
I=
R L + R g + J ( XL + Xg )
Vg
I=
(R L + R g ) 2 + (X L + X g ) 2
Vg2 × R
P= L
(R + R g ) + (X + X g ) 2
2
L L
dP
For maximum power then =0
dRL
Thus R g2 + (X g + X L ) 2 = R 2L
So, RL = Z g + J X L
Table (4-1) summarizes the obtained results for obtaining
maximum power with the different cases.

Table (4-1)
Case number Load type The result
1 Variable resistor RL = Z g
2 Variable impedance ZL = Z*g

3
variable resistor RL = Z g + J X L
fixed reactor

178
Chapter 4 Circuit analysis

Example (4-12).
In the circuit which is shown in Fig. (4-57) XC changes
between 2 → 8 Ω, RL variable. Determine RL and XC which
result in maximum power, then calculate the maximum power
delivered to the load.

J 0.72 2.64
Ω Ω
- J XC
45.6 ∠ 60.3o
V RL

Fig. 4 – 57

For maximum power delivered ZL = Z*g = 2.64 – J 0.72 Ω


But the minimum value of XC is 2, then;
XC = 2 Ω
Thus XC became fixed at 2 Ω, hence
RL = Z g − J X C = 2.64 + J 0.72 - J 2 = 2.64 - J 1.28
= 2.93 Ω
∴ Zeq = 2.64 + 2.93 + J ( 0.72 – 2) = 5.57 – J 1.28
= 5.7 ∠ - 13o Ω
V 45.6 ∠ 60.3
∴I = = = 8 ∠ 73.3o A
Z eq 5.7 ∠ −13
So the maximum delivered power may be;
P = I 2 × RL = 8 2 × 2.93 = 187.5 W

Example (4-13).
In the circuit which is shown in Fig. (4- 58) the supply
resistance Rg is variable from 2 to 55 Ω. What value of R g
results in maximum power transfer across the terminals AB.
What is this value?

179
Chapter 4 Circuit analysis

Rg J5 A

100 ∠ 0 V 10 Ω

Fig. 4 – 58 B
The maximum power may be transferred when passing a
maximum current and the maximum may pass at a minimum
value of Rg so,
Rg = 2 Ω So, Zeq = 2 + J 5 + 10 = 12 + J 5 = 13 ∠ 22.6o Ω
100
∴I = = 7.7 ∠ − 22.6 A
13 ∠ 22.6
∴PAB (Max.) = (7.7)2 × 10 = 593 W

4- Thevenin’s and Norton’s theorems:-

4-1) Thevenin’s theorem:-


Thevenin’s theorem states that in any linear active network with
an output terminals AB as shown in Fig. ( 4-59) can be replaced
by a single voltage source Vo.c. in series with single equivalent
impedance Zeq.. The voltage source Vo.c. is the measured voltage
at the terminals AB. While Zeq is the equivalent impedance of
the network at the terminals AB when all internal sources are
set equal to zero.
Zeq •A
•A
Linear
active Vo.c.
network •B
•B

The equivalent Thevenin’s


Fig. 4 – 59 circuit

180
Chapter 4 Circuit analysis

Example (4-14).
Determine Thevenin’s equivalent circuit for the network which
is shown in Fig. (4-60) with respect to terminals AB. Then Use
the result to find the current in impedance (5 – J5) connected to
it, and then determines the power delivered to it.

•A
-J5 5Ω
50 ∠ 0 V
J5
•B

Fig. 4 – 60
50 ∠0
I= = 10 ∠ 0 A
5
Vo.c. = 10 × (5 + J5) = 50 + J 50 = 70.7 ∠ 45o V
( 5 + J 5) ( − J 5)
Zeq = =5− J5
5
Fig. (4 – 61) shows the Thevenin’s equivalent circuit

-J5 5
• A

70.7 ∠ 45o V

• B
Fig. 4 – 61

Fig. (4-62) shows the equivalent circuit after connecting the


impedance (5 - J5).

181
Chapter 4 Circuit analysis

-J5 5

5
70.7 ∠ 45 V
o

I -J5

Fig. 4 – 62
70.7 ∠ 45
I= = 5 ∠ 90 o A
10 − J10
P = I2 × R = 52 × 5 = 125 W

4-2) Norton’s theorem:-

Norton’s theorem states that any linear active network with


output terminals AB can be replaced by a single current source
IS.C. in parallel with a single impedance Zeq. as shown in
Fig. (4-63). Where IS.C. is the short circuit current through a
short circuit applied to the terminals of the active network. Zeq.
of the Norton’s and Thevenin’s equivalent circuits are identical.

•A
•A
Linear
IS.C. Zeq
active
network •B
• B
The equivalent Norton’s circuit

Fig. 4 – 63

182
Chapter 4 Circuit analysis

Example (4-15).
Determine Norton’s equivalent circuit for the network which is
shown in Fig. (4-64) with respect to terminals AB. Then Use
the result to find the power delivered to the impedance (5–J5)
connected to it.
• A
-J5 5Ω
50 ∠ 0 V
J5
• B
Fig. 4 – 64

Short circuit the terminals A,B for calculating IS.C. Fig. (4-65)
shows the equivalent circuit.

A
-J5 5Ω
50 ∠ 0 V
IS.C
J5
.
B
Fig. 4 – 65

50 ∠ 0
IS.C. = =10 ∠ 90 o A
−J5
For calculating Zeq. short circuit the voltage source and then
calculate the equivalent impedance between the terminals A, B.
− J 5 ( 5 + J 5)
Zeq. = =5− J5
5
The circuit which is shown in Fig. (4–66) represents the
equivalent Norton’s circuit after connecting the load
impedance.

183
Chapter 4 Circuit analysis

I
5Ω 5Ω
10 ∠ 90 A
o

-J 5 -J 5
Ω Ω
Fig. 4 – 66

10 ∠ 90 x (5 − J 5)
I= = 5 ∠ 90o A
10 − J10
Then P = I2 R = 52 × 5 = 125 W
(N. P. Both Thevenin’s and Norton’s equivalent circuits are
alternative to each other).
V
Then, IS.C. = o.c. and Vo.c. = IS.C. × Zeq applying this
Z eq
conclusion on the previous example .
Vo.c. = 10 ∠ 90 × (5 – J 5) = 70.7 ∠ 45o V
V 70.7 ∠45
Also from example (4-14) IS. C. = o. c. = =10 ∠ 90 o A
Z eq 5− J5
Fig. (4-67) shows both Thevenin’s and Norton’s equivalent
circuits.
5Ω -J5 • ِ◌
Ω• A A
5
70.7∠45o 10∠90o Ω
V A
•B -J5
Thevenin’s Ω • B
equivalent Norton’s
circuit equivalent
circuit
Fig. 4 – 67

184
Chapter 4 Circuit analysis

Example (4-16).
Calculate using a) Thevenin’s theorem, b) Norton’s theorem,
and the current in branch AB of the circuit represented in
Fig.(4- 68).
3Ω J 6 A

J5 10 Ω
100∠0o V 12∠-90o A

3Ω -J 20

B

Fig. 4 – 68

Remove the branch AB , Short circuit the voltage source and


open circuit the current source. The internal impedance of the
circuit viewed from AB from AB is that of (3 + J 6) and
(10 – J 20) in parallel so;
( 3 + J6) (10 - J20)
Zeq = = 5.342 + J 5.753 = 7.851∠47.7 o Ω
13 − J14

a) By Thevenin’s theorem.
First change the current source shunted by (10 – J 20) into a
voltage source of voltage;
V = -J 12 × (10 – J 20) = - (240 + J 120) V
in series with (10 – J 20) Ω. The circuit is shown in Fig. (4-69).
3Ω J 6 Ω A - J 20 10 Ω
• Ω

100 ∠0 V -(240 + J 120) V


B

185
Chapter 4 Circuit analysis

Fig. 4 – 69

100 + 240 + J120


The circulating current I =
13 − J14
= (7.507 + J 17.315) A
So the open circuit voltage at AB is;
Vo. c. = 100 – (7.507 + J 17.315) × (3 + J 6)
= (181.37 – J 96.986) V
Hence, the current through the branch AB is;
181.37 − J 96.986
I= = 15.112 ∠ - 80o 20\ A
3 + J 5 + 5.342 + J 5.753

b) By Norton’s theorem.
First change the voltage source in series with (3 + J6) Ω to a
current source of generated current IS shunted by (3 + J 6) Ω, as
shown in Fig. (4 – 70).
100
IS = = 6.667 − J13.33 A
3 + J6
A

3Ω 10 Ω
-J12 A
(6.667–J 13.333)A
J6 - J20 Ω


B
Fig. 4 – 70

If terminals AB are short circuited, the short circuit current IS. C


may be;
IS. C. = 6.667 – J 13.333 + ( - J 12) = (6.667 – J 25.333) A
The equivalent impedance for Norton’s circuit is the same as
Thevenin’s circuit so;
Zeq = (5.342+ J 5.753 ) Ω

186
Chapter 4 Circuit analysis

Hence, the current through the branch AB, is:


5.342 + J 5.753
I = ( 6.667 − J 25.333) x =15.112 ∠ − 80 o 20 \ A
8.342 + J10.753
The same result as was obtained using Thevenin’s theorem.

Example (4-17).
Obtain the Norton equivalent circuit at terminals A B for the
circuit shown in Fig. (4-71).
J5Ω
4Ω J8Ω J8Ω 4Ω

• • •
A
10∠0 V 10∠90 V
• B

Fig. 4 – 71

Short circuit the two voltage sources for obtaining the


equivalent impedance. The equivalent circuit from which the
equivalent impedance may be calculated is shown in Fig.(4-72).
4 J8

D J5
• C C
D D
4 J8

1A
Fig. 4 - 72

From the circuit shown in Fig. (4-72) the current 1A may be


divide equally between the two branches, then the potential
difference VCD = 0.5 × (4 + J8) + 0.5 × J5 = (2 + J 6.5) V

187
Chapter 4 Circuit analysis

VC D
So the equivalent impedance Zeq = = ( 2 + J 6.5) Ω
1
The second step in applying Norton’s theorem is to calculate
the short circuit current. Fig. (4-73) shows the circuit after the
terminals A,B is shorted together.
J5Ω
4Ω •
J8Ω J8Ω • 4Ω

I1 I2
10∠0 V IS.C 10∠90 V
.

Fig. 4 -73

From the previous circuit;


I1 (4 + J 8) + I2 (J 5) = 10 ∠0

I2 (4 + J 8) + I1 (J 5) = 10 ∠90
Rewriting the previous equations in a matrix form as follows;
(4 + J 8) J 5   I1   10 ∠0 
=
 J5
 (4 + J 8) I 2  10 ∠90

∆ = (4 + J 8)2 + 25 = - 23 + J 64
∆1 = 10 (4 + J 8) – J 10 × J 5 = 90 + J 80
90 + J 80 120.41∠ 41.63
I1 = = =1.77 ∠ − 68.137 o A
− 23 + J 64 68 ∠109.767
∆2 = J10 (4 + J 8) – 10 × J 5 = -80 - J 10
- 80 − J10 80.62 ∠ −172.87
I2 = = =1.185 ∠ 77.358o A
− 23 + J 64 68 ∠109.767
IS. C. = I1 + I2 =1.77 ∠ − 68.137 o + 1.185∠ 77.358 o
= .659 – J 1.643 + .259 + J 1.156
= 0.918 – J 0.487 = 1.04 ∠ - 28o A

188
Chapter 4 Circuit analysis

The circuit which is shown in Fig. (4-74) shows the Norton’s


equivalent circuit of the magnetic circuit which had been shown
in Fig. ( 4 – 71).
• A

2Ω
1.04 ∠ - 28o A
J 6.5 Ω
• B
Fig. 4 - 74

189
Chapter 4 Circuit analysis

Problems

1- In the network shown in Fig. (4-75) determine the


components of node voltage V2 due to each current source.

1 4.3 J2.5 2
3
10∠0 A 5 2 J10 5∠0 A

Fig. 4 – 75

2- In the network shown in Fig. (4-76) find the current in the 4


Ω resistor due to each of the voltage sources.
5 4 2

50∠0V J2 -J 2 50∠90oV

Fig. 4 – 76

3- In the network shown in Fig. (4-77) let the voltage source act
separately on the circuit. If the corresponding currents in the 10
V
Ω resistor are equal, what is the value of the ratio 1 ?
V
2
5 J5 J 10

V1 10 V2

Fig. 4 – 77

190
Chapter 4 Circuit analysis

4- In the circuit shown in Fig. (4- 78) find the current in the
(3 – J 4) ohm impedance. Apply the reciprocity theorem and
compare the two currents.
5 3

3
25∠0 V -J4

J4

Fig 4 – 78

5- In the network shown in Fig. (4-79) find the voltage Vx.


Interchange the position of the current source and the voltage
Vx and verify the reciprocity theorem.
3

V
5 ∠-90o A 2 x
10
J4

J5

Fig 4 – 79

6- In the network shown in Fig. (4-80) determine the value of


RL for which the power transferred is a maximum. Then
determine the value of the maximum power.

191
Chapter 4 Circuit analysis

10 J 10

-J 15
100∠0 V
RL

Fig. 4 – 80

7- In the network shown in fig. (4-81) if this load is variable in


both reactance and resistance, what load ZL will receive
maximum power? What is the value of the maximum power?
5 J5 3 -J 4

ZL 50∠90o V
50∠0 V

Fig. 4-81

8- Using Thevenin’s theorem, calculate the current through the


10 ohm resistor in the circuit of Fig. (4-82).

4Ω 6Ω

15 Ω 12 Ω 10 Ω
12 V
5Ω 2Ω

Fig. 4 – 82

192
Chapter 4 Circuit analysis

9- Determine the Norton’s equivalent generator at the terminals


AB for the network represented by Fig.(4-83) and hence the
current in the 10 ohm resistor. Then apply a source
transformation to obtain the Thevenin’s equivalent generator at
the terminals AB. Then calculate the current in the 10 ohm
resistor. Compare the results.
- J5 Ω
J20 Ω 10 Ω A

10 Ω 5Ω 10 Ω
10∠30 V


B
Fig. 4- 83

10- Determined the Thevenin’s equivalent generator for the


network represented by Fig. (4-84). Then apply source
transformation to obtain the Norton’s equivalent generator.

- J10 Ω 10 Ω

100 ∠0 A 10 Ω J10 Ω

Fig 4 - 84

193
Chapter 5 Poly phase Systems

CHAPTER 5

Poly phase Systems

Principles of star and mesh connections:-


It's a generator load pair in which the generator instead of
producing single phase voltage produces two or more voltages
of the same magnitude and frequency but differ from one
another in phase. With the exception of the two-phase system in
π
which the two voltages differ in phase by only radian, the
2
generated voltages in an n phase system form a symmetrical

array of phasors radian apart. Each equivalent source of the
n
generator feeds the corresponding branch of the load as shown
in Fig. (5-1).
n lines
Generator load

En E3 E2 E1 1 2 3 n

n lines
Fig. 5-1

From Fig. (5-1) 2n connecting wires are needed to supply the


load. However, by commoning one of each terminal pair of
both generator and load into one terminal, as shown in Fig. (5-
2), only (n + 1) wires are needed, and this can be reduced to n
in the special case of a balanced load, (i. e. a load in which all
branches have the same impedance).

194
Chapter 5 Poly phase Systems

Generator n lines
load

En E3 E2 E1 1 2 3 n

Neutral
line

Star point of generator Star point of load


Fig. 5 -2

This is the so-called STAR system, the common terminal being


referred to as the star point or neutral point, sea Fig. (5-3).

En
E1

E2 Star point of load

E3

Star point of generator

Neutral wire
Not required for a balanced load

Fig. 5-3

195
Chapter 5 Poly phase Systems

An alternative arrangement of the generator and the load phases


except for two phase system is to connect them end to end in
cyclic order to form a closed mesh, as shown in Fig. (5-4).

En E3 E2 E1 1 2 3 n

Generator Load

Fig. 5 – 4

The wires connecting load to generator are then taken to the n


junctions of the n sources, so that here, also, n wires are needed.
There is no node corresponding to the star point and hence the
mesh system on its own cannot be symmetrically located with
respect to earth, Fig. (5-5) shows an alternative representation
of mesh connected system..

En n

E1 1
Generator Load
E2 2
E3 3
Fig. 5 - 5

Applying voltage Kirchhoff's law on the circuit shown in Fig.


(5-5) then;

196
Chapter 5 Poly phase Systems

E1 + E2 + E3 + --------- + En = 0
The wire connecting the generator to the load are called the
lines, where each branch of the generator or load is called a
phase so that the words line and phase may be used to describe
the voltages and currents as appropriate.

Advantage of polyphase systems:-


1- In rotating machines the same available for the winding
is used for the winding is used more efficiently for a
given size the poly phase machine output power is
greater.
2- For the same overall cross sectional area of conductors
and the same voltage to earth the transmission efficiency
(ηtran.) of the same power is greater.

Example (5-1).
a) A single phase transmission line delivers power P to a load
at a voltage V with 10% voltage drop from a supply to load.
Assuming that is all due to resistance in the line calculate the
transmission efficiency.
b) For the same overall conductor cross section and the same
transmitted power, calculate the transmission efficiency for a
symmetrical n phase system having a voltage V to earth on each
line.

a- For simplicity assume a unity power factor. R is the load


resistance and r is the resistance of each line then the load
power P = I2 R where I is the line current and the line loss = I2
(2r).
I2 R R
η= 2 =
I R + I (2r) R + 2r
2

Total voltage drop in the line = I (2 r)


Voltage drop across the load = I R
2Ir 2r 1
Ratio of voltage drop = = =
IR + 2Ir R + 2r 10

197
Chapter 5 Poly phase Systems

R
Then 20 r = R + 2 r , 18 r = R r =
18
R 9
∴η= = = 90%
R 10
R+
9
b-The same amount of conductor material must divide to n lines
plus neutral.
Assuming A is the cross section area of the single phase
conductor circuit and a is the cross section of the conductor
from the n phase system.
1 2
∴a(n+ )=2A , a= A
2 1
n+
2
1 1
r α & r \ α Where r\ is the line resistance of n phase
A a
system.
1
\ n+
r A 2
∴ = =
r a 2
 The same power is transmitted by n phases then the current in
I
each line is .
n
1
n+
I 2 \ I 2 2 )r
∴ Power loss per line = ( ) * r = ( ) * (
n n 2
1
I 2 n+ 2
Total power losses for the n lines = n * ( ) * ( )r
n 2
1
n+
= I2 r ( 2)
2n
1
n+
Power transmitted = I2 R + I2 r ( 2)
2n

198
Chapter 5 Poly phase Systems

R
r=
18
I2 R 36 n 36
So, η = = ≅ = 97.3 %
1 1 37
n+ 37n +
2 R 2 2
I R +I
2
( )
18 2 n

Example (5-2).
Prove that the transmitted power by a symmetrical n phase
supply to a balanced load is constant.
Let the voltage for the n phases supply is V1(t) = 2 V cos ω t
2 4
V2 (t) = 2 V cos (ω t - ) , V3 (t) = 2 V cos (ω t - )
n n
2 ( m −1 ) π
Vm (t) = 2 V cos (ω t - ) where m = 1, 2, 3, …….n
n
For a power factor = 1
2 ( m −1 ) π
im (t) = 2 I cos (ω t - ) For m =1, 2, 3, …….n
n
n n
2 (m −1) π
∴ P (t) = ∑ Vm (t) *i m (t) = 2 ∑ V I cos 2 (ω t − )
m =1 m =1 n
2 (m −1) π
n
{ cos 2 (ω t − ) +1}
= 2 ∑VI n
m =1 2
n
2 (m −1) π
= n V I + ∑ V I cos 2 (ω t − )= n V I
m =1 n
∴ P (t) is independent of time (i. e. the instantaneous power
is equal to the mean power) .
When the voltage and current are out of phase by angle ϕ the
result is P = n V I cos ϕ.

199
Chapter 5 Poly phase Systems

Two and three phase's systems:


Relationships between line and phase values of voltage and
current which are named (VL, IL) for line values and (Vph , Iph)
for phase values.

a) Two phase star connected system:-


For this system n = 2 Fig. (5- 6-a) shows the generating system,
while Fig. (5- 6 -b) shows the vector diagram.

Line 1
E1-E2=EL
I1 E1
Neutral
E1 45
- (I1+I2) o
Line 2
-E2 E2
I2
E2
a b
Fig. 5-6

E1 – E2 = 2 EP ∠ 45o EP is the phase voltage.


And IN = - (I1 + I2)
For a balanced load (i. e. each phase having the same
impedance)
Then, IN = 2 IP where IP is the phase current.
∴ P = 2 EP IP cos ϕ
Thus VL = 2 VP and IL = IP, where VL and IL are the line
values. So, P = 2 VL IL cos ϕ

b) Three phase star connected system:-


A three phase star connected power supply is shown in
fig. (5-7).

200
Chapter 5 Poly phase Systems

IL1
I1
V1
V31 V12
IL2

V2
I2
I3 V3 V23

IL3
Fig. 5-7

Fig. (5-8) shows the vector diagram of the voltage (phases and
lines).
V12 V1

30o

120o 120o
V23
V3 120o
V2

V32

Fig. 5 – 8

From the vector diagram which is shown in Fig. (5-8) it's


possible to have;
V12 = V1 – V2 = 3 V∠ 30o, V23 = V2 – V3 = 3 V∠-90o
V31 = V3 – V1 = 3 V∠ 150o

201
Chapter 5 Poly phase Systems

Where V1, V2 and V3 are the phase's voltage with magnitude


value equals V. While V12, V23 and V31 are the lines voltage,
which is leading the set representing the phase voltage by 30o in
phase and 3 greater in magnitude, so VL = 3 V ∠ 30o.
If a load is connected to the generator then;
I1 = IL1 , I2 = IL2 and I3 = IL3
Also P = 3 V I cos ϕ
Where I is the magnitude of the phase current and ϕ is the
phase shift between the phase voltage and the phase current.
V = 3 V
L
The magnitude of line current IL is equals the magnitude of
phase current then (i.e. IL = Iph)
V
∴ P = 3 x L × IL × cos ϕ = 3 VL IL cos ϕ
3

c) Three phase mesh connected system:-


A three phase mesh connected power supply is shown in
Fig. (5-9–a) and a current vector diagram is shown in
Fig. (5-9-b).
IL1 IL1
I1
E1 E2
IL2 V1 I1
I2
2 V3
I3
E3 IL3 V2 1 I3
3 IL2
I2

(a) IL3 (b)

Fig. 5 -9

From the shown figure its assumed that;


I1 = I ∠ 0o then I2 = I ∠ - 120o and I3 = I ∠ 120o
where I is the current the phase value .

202
Chapter 5 Poly phase Systems

So IL1 = I1 – I2 = 3 I ∠ 30o, IL2 = I2 – I3 = 3 I ∠ - 90o


IL3 = I3 – I1 = 3 I ∠ 150o
Thus its possible to say V = V while I = 3 I
L ph L ph
 P = 3 Vph Iph cos ϕ
Where ϕ is the phase shift between Vph and Iph.
I
∴ P = 3 VL × L × cos ϕ = 3 VL IL cos ϕ
3
For a balanced load the phases and lines voltage and currents
are equal in magnitudes while shifted from each other by 120o
or – 120o corresponding to its sequence.

Example (5-3).
A 60 KW three phase mesh connected induction motor has a
full load power factor of 0.819 Lag. And an efficiency η = 87
% with VL = 415 V. Calculate the line and phase current at full
load.
P 60
 Pout = 60 KW , then Pin = out = = 68.965 KW
η 0.87
 Pin = 3 x VL × IL × cos ϕ = 3 × 415 × IL × 0.819
68965
∴ IL = = 117.15 A
3 x 415 x 0.819
I 117.15
Iph = L = = 67.64 A
3 3
P 60
Or Pout per phase = out = = 20 KW
3 3
P per phase 20
Pin per phase = out = = 22.988 KW
η 0.87
 Pin per phase = Vph × Iph × cos ϕ
∴ 22988 = 410 × Iph × 0.819
Iph = 67.64 A

203
Chapter 5 Poly phase Systems

And IL = Iph × 3 = 67.64 × 3 = 117.15 A.

Example (5-4).
Three coils each coil have 5 Ω resistance and 10 Ω reactance.
They are connected (a) In star, b) In mesh) across a 440 V three
phase supply. If each coil is shunted by a capacitor of reactance
20Ω . For each case calculate the line and phase currents and
the total power absorbed.

The total admittance per phase Y may be;


1 1
Y= +J = 0.04 – J 0.08 + J 0.05 = 0.04 – J 0.03
5 + J10 20
= 0.05 ∠ - 36o 52\
1 1
Z= = = 20 ∠ 36 o 52 \ Ω
Y 0.05 ∠ − 36 52
o \

a) For star connection;


440
Vph = = 254 V
3
V 254
Iph = ph = = 12.7 ∠- 36o 52\ A
Z 20 ∠ 36 52 o \

 IL = Iph , ∴ IL = 12.7 ∠ - 36 52 A
o \

P = 3 Vph Iph cos ϕ = 3 x 254 × 12.7 × cos - 36o 52\ = 7742 W


Or P = 3 VL IL cos ϕ = 3 × 440 × 12.7 × cos - 36o 52\
= 7742 W

b) For mesh connection;


Vph = VL = 440
V 440
ph
Iph = = = 22 ∠ − 36 o 52 \
Z 20 ∠ 36 52
o \

I = 3 I ∠  + 30o
L ph

204
Chapter 5 Poly phase Systems

∴I = 3 x 22 ∠( − 36 o 52 \ + 30 o ) = 38.105 ∠ − 6o 52 \ A
L
P = 3 Vph Iph cos ϕ = 3 × 440 × 22 × cos – 36o 52\ = 23232 W
Or P = 3 VL IL cos ϕ = 3 × 440 × 38.105 × cos - 36o 52\
= 23232 W

Three phase power system with unbalanced load.


This is still easy for a mesh and for a four wire star systems.
But when the neutral wire is removed ( i. e. three wire star
connected) one of the circuit analysis methods must be used.

Example (5-5).
For the load system shown in Fig. (5-10) calculate the line and
phase currents then calculate the total power absorbed and the
total power factor.

1
IL1
I1
Z1 -J 20 5 Z2
400 V J 10
J 10
5
3 IL3 5 J 10 I2

2 IL2 I3 -J 20
Z3
Fig. 5 – 10

− J 20 ( 5 + J10 )
Z1 = Z3 = = 20 ∠ 36o 52\ Ω
5 − J10
Z2 = 5 + J 10 = 11.18 ∠ 63o 26\ Ω
Assume V13 = 400 ∠ 0o V, V21 = 400 ∠ - 120o V and
V32 = 400 ∠ 120o V

205
Chapter 5 Poly phase Systems

V13 400
I1 = = = 20 ∠ − 36 o 52 \ A
Z1 20 ∠ 36 52 o \

V21 400 ∠ -120 o


I2 = = = 35.778 ∠ −183o 26 \ A
Z 2 11.18 ∠ 63 26o \

V32 400 ∠ 120 o


I3 = = = 20∠ 83o 8 \ A
Z 3 20 ∠ 36 52o \

IL1 = I1 – I2 = 20 ∠ - 36o 52\ - 35.778 ∠ - 183o 26\


=16 – J 12 + 35.713 – J 2.14 = 51.713 – J 14.14
= 53.611 ∠ - 15o 17\ A
IL2 = I2 – I3 = 35.778 ∠ - 183o 26\ - 20 ∠ 83o 8\
= -35.713 + J 2.142 - 2.391 – J 19.856 = - 38.104 – J 17.714
= 42.02 ∠ 204o 56\ A
IL3 = I3 – I1 = 20 ∠ 83o 8\ - 20 ∠ - 36o 52\
=2.391 + J 19.565 - 16.0 + J 11.999 = - 13.609 + J 31.564
= 34.373 ∠ 113o 19\ A
there are different methods for calculating the power absorbed
through each phase, the first one as;
P1 = V13 I1 cos (  −  ) = 400 × 20 × cos ( 0o – 36o 52\ )
V I
13 1
= 6400.27 W.
P2 = V21 I2 cos (  − )
V I
21 2
= 400 × 35.778 × cos (-120o +183o 26\) = 6400.524 W
P3 = V32 I3 cos (  −  ) = 400 × 20 × cos ( 120 – 83o 8\ )
V I
32 3
= 6400.27 W
Pt =P1 + P2 + P3 = 6400.27 + 6400.524 + 6400.27
= 19201.064 W
While the second is; as the resistances is the only elements
which consumes power then,
Z1 = Z3 = 20 ∠ 36o 52\ = 16 + J 12
and Z2 = 5 + J 10
Thus P1 = I12 R1 = (20)2 × 16 = 6400 W
P2 = I 22 R2 = (35.778)2 × 5 = 6400.326 W

206
Chapter 5 Poly phase Systems

P3 = I 32 R3 = (20)2 × 16 = 6400 W
Pt =P1 + P2 + P3 = 6400 + 6400.326 + 6400 = 19200.326 W
The third method is;
For phase 1 S1 = V13 I1* = 400 × 20 ∠ 36o 52\ = 6400 +J 4800
Phase 2 S2 = V21 I *2 = 400 ∠ - 120o × 35.778∠183o 26 \
= 6400 + J 12800
Phase 3 S3 = V32 I *3 = 400 ∠ 120o × 20∠ − 83o 8 \
= 6400 + J 4800
Total S = S1 + S2 + S3 = 19200 + J 22400
= (19200) 2 + ( 22400 ) 2 = 29500 VA
So Pt = 19200 W
P 19200
P. F. = = = 0.651 lag.
S 29500

Example ( 5-6).
For the circuit shown in fig. (5-11) calculate the line currents,
the neutral current and the total power absorbed. The phase
sequence is 1- 2- 3.
The phases voltage are symmetrical and of magnitude 254 V.

IL1
Z1=10∠30o
V13 = 440 V
IN
Z2=15∠- 45o
IL3
Z3=20∠60o
IL2
Fig. 5 – 11
Taking V1 as the reference phasor:
V1 = 254 ∠ 0o V , V2 = 254 ∠ -120o V and V3 = 254 ∠ 120o V
V 254 ∠ 0
IL1 = I1 = 1 = = 25.4 ∠ − 30 o = 21.997 – J 12.7 A
1 10 ∠ 30
Z

207
Chapter 5 Poly phase Systems

V 254 ∠ − 120
IL2=I2= 2 = =16..933 ∠ − 75o = 4.383 – J 16.356 A
Z
2 15 ∠ − 45
V 254 ∠120
IL3 = I3 = 3 = =12.70 ∠ 60o = 6.35 + J 10.999 A
Z 20 ∠ 60
3
The neutral current ;
IN = - ( I1 + I2 + I3 ) = - ( 21.997 – J 12.7 + 4.383 – J 16.356 +
6.35 + J 10.999 ) = - 32.73 + J 18.058 = 37.381 ∠ 151o 7\ A
The power into each phase is calculated separately;
P1 = V1 × I1 × cos (  −  )
V I
1 1
= 254 × 25.4 × cos (30) = 5587 W
P2 =V2 × I2 × cos (  − )
V I
2 2
= 254 ×16.933 × cos (45) = 3041 W
P3 = V3 × I3 × cos (  −  )
V I
3 3
= 254 × 12.7 × cos (60) = 1613 W
Pt = P1 + P2 + P3 = 5587 + 3041 + 1613 = 10241 W

Example (5-7).
The neutral wire is removed from the circuit of the previous
example as shown in Fig. (5-12). Recalculate the line currents
and determine the change in potential of the star point.

IL1
Z1=10∠30o
V13 = 440 V IA
0
Z2=15∠- 45o
IL3
Z3=20∠60o
IB
IL2

Fig. 5 – 12

208
Chapter 5 Poly phase Systems

As the neutral wire is removed then the star point potential


changes and the three line to neutral voltages are neither equal
nor at 120o to one another. Referring to Fig. (5-12) let IA and IB
be two currents placed as shown.
The mesh-current rule gives;
(Z1 + Z3) IA – Z3 IB = V1 – V3 = V13
And (Z2 + Z3) IB – Z3 IA = V3 – V2 = V32
Now V13 = 254 – 254 ∠ 120o = 381 – J 220 V
And V32 = V3 – V2 = 254 ∠ 120o – 254 ∠- 120o = J 440 V
So IA (18.66 + J 22.32) – IB (10 + J 17.32) = 381 – J 220
And – IA (10 + J 17.32) + IB (20.61 + J 6.71) = J 440
Hence IA = 6.01 ∠ - 26o 15\ A & IB = 25.8 ∠ 71o 4\ A
Thus I1 = IA = 6.01 ∠ - 26o 15\
I2 = - IB = 25.8 ∠ -108o 56\ A
And I3 = IB – IA = 21.95 ∠ 82o 12\ A
Fig (5-13) shows a vector diagram for the voltage sequence of
the previous example. From this figure its possible to have;
V10 = I1 Z1 = 6.01 ∠ - 26o 15\ × 10 ∠ 30o = 50.1 ∠ 56o 15\
= 33.39 + J 49.97
Thus V0N =V1 – V10 =254 ∠0o –33.39 –J49.97
= 220.61 – J49.97 = 226.2 ∠ - 12o 46\ V

V1
V12 0

V31 V3 V2
V23

Fig. 5 -13

209
Chapter 5 Poly phase Systems

The measurement of power and power factor in a three


phase circuits.

Connection of the wattmeter in a three phase system

Case 1 : Four wires system ( Star point available).


a ) Unbalanced load.
The phase power is measured separately.
Fig. ( 5-14) shows a wiring diagram for the system.

W1 Z1∠ ϕ1
N
Z2∠ ϕ2

3
Z3∠ ϕ3

2 W3

W2
Fig. 5 – 14

Fig. (5-15) shows a vector diagram for the voltages and the
currents of the previous system.
P1 = V1 I1 cos ϕ1 , P2 = V2 I2 cos ϕ2 and P3 = V3 I3 cos ϕ3
So Pt = P1 + P2 + P3
Q = S2 − P 2
P P
And Power factor P. F. = =
S P + Q2
2

210
Chapter 5 Poly phase Systems

V1
ϕ1
I3 I1
ϕ3
ϕ2 V2
I2
V3
Fig. 5 - 15

b) Balanced load.
For balanced load P1 = P2 = P3 So Pt = 3 × P1
Hence only one wattmeter in any phase is needed. The overall
P. F. is the same as that in any one phase.
P
P. F. = 1
V1 I1

Case 2 : Three wires system (Star point not available).


The circuit shown in Fig. (5-16) shows a wiring diagram for the
system.
V1
1 I1

W1 Z1∠ ϕ1
V
Z2∠ ϕ2

3
V3 Z3∠ ϕ3
I3 V2
2 W3

I2
W2
Fig 5 – 16

211
Chapter 5 Poly phase Systems

From this circuit;


P1 = Re ( I* ( V − V )) , P2 = Re ( I* ( V − V ))
1 1 2 2
*
And P3 = Re ( I ( V − V ))
3 3
So Pt = P1 + P2 + P3
= Re ( I* V − I* V + I* V − I* V + I* V − I* V )
1 1 1 2 2 2 3 3 3
= Re {( I* V + I* V + I* V ) - V ( I* + I* + I* ) }
1 1 2 2 3 3 1 2 3
 There is no neutral wire So I* + I* + I* = 0
1 2 3
∴ Pt = Re ( I* V + I* V + I* V )
1 1 2 2 3 3
This result is valid whatever the magnitude and phase of V.

Case 3 : Two wattmeter method.


Fig. (5-17) shows the circuit arrangement for the two wattmeter
method.

V1
1 I1

W1 Z1

Z2

3
V3 Z3
I3 V2
2 I2 W2

Fig. 5 -17
Fig. (5-18) shows a phasor diagram for the voltage and the
current of the system.

212
Chapter 5 Poly phase Systems

V13

V1
30o
ϕ
I3 I1 V23
ϕ 30o
ϕ V2
V3 I2
Fig 5 – 18

From the circuit shown in Fig. 5.17


W1 = Re { I∗ (V − V ) } = Re { I∗ V } = VL IL cos ( ϕ - 30 )
1 1 3 1 13
W2 = Re { I (V − V ) } = Re { I∗ V } = VL IL cos ( ϕ + 30 )

2 2 3 2 23
 I = I = I = I and V = V = V
1 2 3 L 13 23 L
∴W1 + W2 = VL IL cos (ϕ - 30) + VL IL cos ( ϕ + 30 )
= VL IL {cos ϕ . cos 30 + sin ϕ . sin 30 + cos ϕ .
cos 30 - sin ϕ . sin 30 } = VL IL {2 cos ϕ . cos
30}
= 3 VL IL cos ϕ or W1 + W2 = 3 V I cos ϕ
V
Where V is the voltage phase value = L and I is the phase
3
current = IL

Example (5-8).
The current in each line of a 440 v three wire supply feeding a
balanced load is 25 A and two wattmeter's are used for power
measurement. Show how the total power and the individual
meter indications vary as the load P.F. changes from zero P.F.
lag to zero P.F. lead.

213
Chapter 5 Poly phase Systems

P1 = V I cos ( ϕ - 30)
P2 = V I cos ( ϕ + 30)
P1 = 25 × 440 cos ( ϕ - 30) = 11 cos (ϕ - 30) KW
P2 = 25 × 440 cos ( ϕ + 30) = 11 cos (ϕ + 30) KW
P = P1 + P2 = 3 × 11 × cos ϕ = 19.053 cos ϕ KW
1- For P. F. = 1 ϕ = 0 P1 = P2 = 11 cos 30 = 9.526 KW
and P = 19.052 KW

2- For P. F. = 0.866 lag. ϕ = 30o P1 = 11 cos 0 = 11 KW


P2 = 11 cos 60 = 5.5 KW and P = 19.052 KW

3- For P. F. = 0.5 lag ϕ = 60o P1 = 11 cos 30 = 9.526 KW


P2 = 11 cos 90 = 0 and P = 9.526 KW

4- For P. F. = 0 ϕ = 90o P1 = 11 cos 60 = 5.5 KW


P2 = 11 cos 120 = - 5.5 KW and P = 19.052 KW
And so on. Fig. (5-19) shows how P1, P2 and P vary as the load
P. F. changes from zero lag to zero lead.
P

P2 P1

ϕ
• • • • • •
Lead P. F. o 60o 30o 30o 60o 90o Lag P. F.
90

Fig. 5 – 19

Power factor measurement for balanced loads.


 P1= V × I × cos (  - 30)
And P2 = V × I × cos (  + 30)
P cos ( − 30 )
∴ 1= =r
P cos (  + 30 )
2

214
Chapter 5 Poly phase Systems

cos  cos 30 + sin  sin 30 3 cos  + sin 


r= =
cos  cos 30 − sin  sin 30 3 cos  − sin 
3 + tan 
=
3 − tan 
3 r − r tan  = 3 + tan 
( 1 + r ) tan ϕ = 3 (r–1)
3 ( r −1 )
tan  =
( r + 1)
Fig (5 – 20 ) shows a triangle with angle ϕ.

S
3 (r - 1)

(r + 1)
Fig. 5 – 20

From the previous figure its possible to deduce;


S = 3 ( r −1 ) 2 + ( r + 1 ) 2 = 3 r 2 − 6 r + 3 + r 2 + 2 r + 1
= 4 r 2 − 4 r + 4 = 2 r 2 − r +1
r +1 r +1
Thus cos  = =
S 2 r 2 − r +1
This expression applies only to the balanced loads.

Example (5-9).
Two wattmeter's, connected to measure the power delivered to
a three wire, 415 V balanced load, gave readings of 48.2 KW
and – 32.7 KW. Calculate the total power, the line current and
the power factor.

P = P1 + P2 = 48.2 – 32.7 = 15.5 KW

215
Chapter 5 Poly phase Systems

P 48.2
r = 1 =− = −1.474
P 32.7
2
So cos  = 0.1099
32.7
Alternatively: r = − = - 0.684 which gives cos  = 0.1099
48.2
P = 3 V I cos  = 15500 W
15500
So I = = 196.5 A
3 × 415× 0.1099

216
Chapter 5 Poly phase Systems

Problems

1- Three loads are connected in star across a three phase four


wire 415 V, 50 Hz mains supply. Each load consists of a coil of
inductance 0.5 H and resistance 40Ω. Shunted with a capacitor
16 µ f. Calculate;
a- The total power supplied to the load.
b- The line currents.
c- The power factor.
d- The total load on the supply.

2- Three loads of 3 KW, 4 KW and 5 KW all of unity power


factor are star connected to lines 1, 2 and 3 respectively, of a
440 V three phase four wire supply. Calculate; the line currents
and the neutral current.

3- Three impedances 20 ∠ 0o Ω, 16 ∠ 30o Ω and 10 ∠- 60o Ω


are connected in mesh across lines 1 and 2, 2 and 3, 3 and1,
respectively, of 400 V three phase supply. Calculate the phase
currents and the line currents.

4- A 440 V, three phase power system carries the following


loads connected in mesh; across lines 1 and 2 (10 + J 10) Ω,
across lines 2 and 3 (20 – J 15) Ω and across lines 3 and 1 (15 +
J 5) Ω.
a- Calculate the line currents and the power supplied to the
load.
b- Given that the load between lines 2 and 3 is now
disconnected. Calculate the new line currents and the total
power supplied.

5- Three equal loads of 5 KVA at 0.8 P. F. Lag. are connected


in star to a 415 V three phase three wire supply system.
Calculate the total power supplied and the line currents.

217
Chapter 5 Poly phase Systems

The load connected to line 3 is then short circuited. Calculate


the total power supplied and the three currents under these
conditions. The phase sequence is 1, 2, and 3.

6- Three non reactive resistors are connected to a 415 V three


phase three wire balanced supply and the voltages across two of
them are 260 V. Determine the magnitude of the voltage across
the third resistor and the phase relationships between all the
three phase voltages.

7- The power supplied to a balanced three phase load is


measured by two wattmeters connected in the usual way.
Calculate the power and power factor of the load;
a- When the readings are 3 KW and 700 KW.
b- When the readings are 3 KW and – 700 KW

218

You might also like